Sunteți pe pagina 1din 169

QUESTION ANSWERS

OCT 19
1. How many days IRCTC’s Special Buddha Circuit train will take to complete
its journey?
a. Eight
b. Nine
c. Ten
d. Eleven
2. Which of the following player has made a debut for third and last match of
India vs South Africa Series at the age of 30?
a. Ajay Punia
b. Sanjay Singh
c. Shahbaz Nadeem
d. Rashid Faizan
3. How many women astronauts did a spacewalk, without a male astronaut,
for the first time in the history of Space?
a. Two
b. Three
c. Four
d. Five
4. In which school has Union Defence Minister Rajnath Singh approved the
proposal to give admission to girls from the session 2021-22?
a. Netarhat School
b. Sainik School
c. Navoday School
d. NDMC School
5. The culture department of which state government is going to organize
the country's first training and demonstration program for the world-famous
'Khon Ramlila'?
a. Bihar
b. Punjab
c. Karnatak
d. Uttar Pradesh
6. Which of the following country will organise 91st Interpol General
Assembly in 2022?
a. China
b. India
c. Nepal
d. Pakistan
7. Which two states will witness assembly elections on October 21, 2019?
a. Karnataka & Maharashtra
b. Maharashtra and Haryana
c. Haryana & Rajasthan
d. Rajasthan & Karnataka
8. Who was appointed as the Director-General of the National Security
Guard (NSG)?
a. Anup Kumar Singh
b. Rakesh Kumar Singh
c. Harinder Singh
d. MM Narwane
9. How many agreements were signed between India and Philippines during
President Ram Nath Kovind’s visit to Manila?
a. Six
b. Five
c. Four
d. Three
10. What is the name of Oracle’s CEO who was recently passed away?
a. Jerem Dumini
b. Mark Hurd
c. Joseph Ilnis
d. Robert Dwen
Answer
1. (a) Eight
IRCTC has started Special Buddha Circuit Train to connect all Buddha
pilgrimages. This train is equipped with modern facilities and will travel to 26
places in eight days.
2. (c) Shahbaz Nadeem
Shahbaz Nadeem is a left-arm spinner who has made a debut in third test-
match of India vs South Africa series. He is 30 years old. He is assisting R
Ashwin and Ravindra Jadeja in the match.
3. (a) Two
NASA’s Christina Koch and Jessica Meir have made a world record with their
spacewalk which was organized without a male companion. It was the fourth
spacewalk of Christina Koch while first of Jessica Meer.
4. (b) Sainik School
India's first Sainik School was established in the year 1961 in Maharashtra.
After that, Sainik Schools were opened in Kunjpura in Haryana, Kapurthala
in Punjab, Balachadi in Gujarat and Chittorgarh in Rajasthan. It will be the
first time when girl students will get admission in Sainik School.
5. (d) Uttar Pradesh
Thailand's Khon Ramleela has been included in UNESCO's list of intangible
cultural heritage. It is a masked dance depicting scenes from Ramlila. Uttar
Pradesh Government will prepare some artists from the state for this art.
6. (b) India
India will organise Interpol’s 91st General Assembly on the occasion of 75th
Independence Day of India. It was established in 1923 and India was
included in the organization in 1949. Earlier, India had organized general
assembly in 1997.
7. (b) Maharashtra and Haryana
Maharashtra and Haryana assemblies will be going to polls on October 21,
2019 (Monday). Along with the assembly elections, by-polls for two Lok
Sabha constituencies and 51 Assembly seats will also take place on the
same day.
8. (a) Anup Kumar Singh
Senior IPS officer Anup Kumar Singh was appointed as the Director-General
of the National Security Guard (NSG). Singh is a 1985-batch IPS officer of
Gujarat Cadre.
9. (c) Four
The President of India Ram Nath Kovind met his Philippines counterpart
President Duterte and signed four agreements. These agreements are
covering domains like security, maritime, science and technology and
tourism.
10. (b) Mark Hurd
Oracle co-CEO Mark Hurd died at the age of 62. He was also the CEO of HP
before this position. During his tenure, there was a 37% increase in annual
profits of Oracle shares.

OCT 18
1. Which State’s Legislative Council was abolished recently on October 18,
2019?
a. Manipur
b. Jammu & Kashmir
c. Himachal Pradesh
d. Assam
2. Who has been recommended to be appointed as the 47th Chief Justice of
India?
a. Justice SA Bobde
b. Justice Indu Malhotra
c. Justice Arun Kumar Mishra
d. Justice RF Nariman
3. Which State topped the India Innovation Index 2019 of NITI Aayog?
a. Karnataka
b. Kerala
c. Tamil Nadu
d. Gujarat
4. Which organization recently retained Pakistan on its Grey List for its
failure to combat terror funding?
a. Afghanistan
b. Syria
c. Pakistan
d. Yemen
5. Which Union Territory got the first rank in India Innovation Index 2019?
a. Goa
b. Daman and Diu
c. Puducherry
d. Delhi
6. What percentage of Indian Children are underweight as per the recently
released UNICEF report?
a. 33 per cent
b. 28 per cent
c. 20 per cent
d. 13 per cent
7. According to the report released by which international organization,
India's economic growth rate for the year 2019 and 2020 will be 6.1% and
7% respectively?
a. World Bank
b. IMF
c. United Nations
d. Asian Bank of Development
8. How many agreements have been signed between China and Nepal during
a bilateral meeting?
a. 16
b. 18
c. 20
d. 22
9. Which state is celebrating Shirui Lily Festival 2019?
a. Manipur
b. Assam
c. Nagaland
d. Arunachal Pradesh
10. Union Minister for Road Transport Nitin Gadkari recently announced the
construction of new National Highway NH- 703AA connecting Gobindwal
Sahib and Taran Tarana in Punjab. The Road is named after which Guru?
a. Guru Nanak Dev
b. Swami Vivekananda
c. Guru Ram Das
d. Guru Tegh Bahadur
Answers
1. (b) Jammu & Kashmir
The Legislative Council of Jammu & Kashmir got abolished on October 18,
2019, making the way for the creation of two Union Territories - Jammu &
Kashmir and Ladakh. The Council was dissolved under Section 57 of the J&K
Reorganisation Bill, 2019.
2. (a) Justice SA Bobde
CJI Ranjan Gogoi on October 18, 2019, recommended Justice Sharad Arvind
Bobde for appointment as the 47th Chief Justice of India. Chief Justice Gogoi
is set to retire on November 17, 2019.
3. (a) Karnataka
Karnataka has topped the India Innovation Index, the first-ever index
launched by NITI Aayog to promote competitiveness among Indian states.
Karnataka is followed by Tamil Nadu and the Maharashtra States that are
ranked second and third, respectively.
4. (c) Pakistan
FATF decided to retain Pakistan on its ‘Grey List’ till February 2020. The
Financial Action Task Force (FATF) has directed Pakistan to take extra
measures in combating money laundering and funding of terrorists.
5. (d) Delhi
Delhi has got the first place in the list of Union Territories and small states in
the India Innovation Index 2019. Delhi is at the first place with 42.98 points
while Chandigarh is in second place with 27.97 points.
6. (a) 33 per cent
According to UNICEF’s The State of the World’s Children 35% of children in
India are suffering from stunting, 2% of children are overweight while 33%
of children are underweight.
7. (b) IMF
International Monetary Fund (IMF) has recently released its report – World
Economic Outlook Report. According to this report, India’s economic growth
rate will be 6.1% in 2019 and 7 in 2020. Earlier, the IMF estimated that
India’s economic growth rate will be 7.3% in 2020.
8. (c) 20
The president of China Xi Jinping recently visited Nepal and signed 20
agreements for the development for the country. Nepal and China agreed to
build a network worth Rs. 2.75 crore to connect Nepal with China’s Belt and
Road Initiative (BRI).
9. (a) Manipur
The annual Shirui Lily Festival 2019 is being organized at Ukhrul in Manipur.
This four-day state festival was inaugurated by Union Tourism Minister
Prahlad Singh Patel. Folk songs and folk dances are performed by different
communities in the festival.
10. (a) Guru Nanak Dev
Nitin Gadkari, Union Minister for Road Transport and Highways, announced
the construction of new National Highway NH- 703AA starting from
Kapurthala and ending at Taran Tarana in Punjab. The highway is named as
‘Shri Guru Nanak Dev Ji Marg’ to mark 550th Birth Anniversary of Guru
Nanak Dev.
OCT 17
1. What percentage of growth was recorded in the recently concluded
Livestock Census in India as compared to the year 2012?
a. 4.63 percent
b. 3.96 percent
c. 2.34 percent
d. 1.71 percent
2. When International Day for the Eradication of Poverty is observed?
a. 16 October
b. 17 October
c. 18 October
d. 19 October
3. Which of the following footballer has won this year's Golden Shoe title?
a. Harry Ken
b. Cristiano Ronaldo
c. Vergil Van
d. Lionel Messi
4. From how many days the Supreme Court was hearing the case of
Ayodhya land dispute?
a. 40
b. 50
c. 60
d. 70
5. How much fine has recently imposed on SBM Bank (India) by the Reserve
Bank of India?
a. 5 cr
b. 4 cr
c. 6 cr
d. 3 cr
6. How many UAE players have been suspended by the ICC recently on
corruption charges?
a. 3
b. 4
c. 7
d. 8
7. Which scheme has been launched by the Central Government to
encourage girl students to pursue STEM education?
a. Jyoti Anand Yojana
b. Vigyan Jyoti Yojana
c. Naari Kalyan Yojana
8. Which of the following actress has made a world record of getting more
than 1 million followers on Instagram in 5 hours 16 minutes?
a. Priyanka Chopra
b. Kylie Minogue
c. Sandra Bullock
d. Jennifer Aniston
9. Which Chief Minister of an Indian state has recently biked 122 kilometres
to promote adventure tourism in his state?
a. Jaganmohan Reddy
b. Arvind Kejriwal
c. Pema Khandu
d. Nitish Kumar
10. Which State/UT Chief Minister has announced the Odd-Even scheme to
curb pollution starting from November 4 to 15?
a. Haryana
b. Delhi
c. Rajasthan
d. Bihar
Answers
1. (a) 4.63 percent
The livestock population in India has increased by 4.63% from 512 million to
535 million. UP has recorded as the highest number of livestock at 67.8
million. In India 192 million Cattle and 109 million buffalos are present.
2. (b) 17 October
The main objective of this day is to create awareness about the efforts being
made to eradicate poverty from the world community. The International
Poverty Eradication Day was announced by the United Nations on 22 October
1992.
3. (d) Lionel Messi
Argentine footballer Lionel Messi was awarded the Golden Shoe Award for
the sixth time for scoring 36 goals for Barcelona. He is the first player to win
this award for three consecutive years.
4. (a) 40
The Supreme Court has kept its verdict reserved after completion of
Ayodhya Case hearing. The final verdict will come after 23 days. The
Supreme Court heard the Ram Janmabhoomi and Babri Masjid case on 16
October 2019 which was the 40th day of the case.
5. (d) 3 cr
This penalty has been imposed on SBM Bank (India) for not complying with
the regulatory rules. SBM Bank (Mauritius) has been merged with SBM Bank
(India).
6. (a) 3
The ICC has accused national team batsman Sheman Anwar, Captain
Mohammad Naveed and right-arm fast bowler Qadir Ahmed. Another
cricketer Mehrdip Chhayakar, who took part in cricket through the Ajman
League, has been accused of refusing to cooperate to the ICC.
7. (b) Vigyan Jyoti Yojana
Through this scheme, 100 girls from 550 districts will be trained by the year
2020-2025. The qualified students will be chosen according to their marks in
the exams. Girls from class 9 to 12 will be included under this scheme.
8. (d) Jennifer Aniston
More than one million people started following Jennifer Aniston’s Instagram
account in 5 hours 16 minutes. She joined Instagram on October 16 and
shared her first picture on the account.
9. (c) Pema Khandu
The chief minister of Arunachal Pradesh Pema Khandu rode a motorcycle for
more than 122 kilometres to promote adventure tourism in his state. He also
gave a message through this trip that Arunachal Pradesh is the safest place
for biking.
10. (b) Delhi
Chief Minister of Delhi Arvind Kejriwal announced that vehicles carrying
school children (in uniform) will be exempted from the Odd-Even restrictions
in Delhi that will kick in from November 4 to 15. Women who drive alone, or
with a child up to 12 years old will also be exempted.

OCT 16
1. How many matches in international cricket Mithali Raj has won as Indian
women team’s captain?
a. 100
b. 150
c. 200
d. 250
2. The International Monetary Fund (IMF) has reduced India's GDP growth
forecast for the year 2019-20 from 7 percent to what percentage?
a. 6.9 percent
b. 6.1 percent
c. 6.8 percent
d. 6.5 percent
3. What is the rank of India in Global Hunger Index 2019?
a. 94
b. 99
c. 102
d. 105
4. Who among the following has won Booker Prize 2019?
a. Margret Atwood
b. Nilima Riddhi
c. Jenifer Williamson
d. Pedro Roberto
5. How many PCOs to be established in the Kashmir region to improve
phone connectivity in the area?
a. 40
b. 50
c. 60
d. 70
6. Which scheme was recently launched by FSSAI on the occasion of World
Food Day?
a. Food Quality Scheme
b. Health and Food Relation Scheme
c. Food Supply Gram
d. Food Safety Mitra
7. Who has been recently selected as president of Indian Pharmaceutical
Alliance?
a. Jatin Prasad
b. Vivek Tomar
c. Satish Reddy
d. Arjun Patel
8. India and which country is organizing a joint military exercise ‘Shinyuu
Maitri’?
a. Nepal
b. China
c. Russia
d. Japan
9. Who has been recently appointed as new law secretary?
a. Anoop Kumar Mendiratta
b. Abhijit Banerjee
c. Dev Patel Sahay
d. Vivek Goyal
10. Which Indian Armed Forces recently organized the Sangam Youth
Festival?
a. Indian Air Force
b. Indian Military
c. Indian Navy
d. Para Military Force
Answers
1. (a) 100
Mithali Raj has achieved this feat after winning the ODI series 3–0 against
South Africa. Mithali Raj has captained 167 matches. She is only the second
female captain to win 100 matches in women's cricket after England's
Charlett Edwards.
2. (b) 6.1 percent
The International Monetary Fund (IMF) has reduced global economic growth
forecast to 3 percent during 2019-20. IMF estimated India’s GDP growth for
the year 2019-20 to 6.1 percent.
3. (c) 102
India is ranked 102 in the Global Hunger Index of 117 countries. Its aim is
to save the world from the food crisis and improve the quality of existing
food production. It also helps to know about the policies issued by various
governments around the world.
4. (a) Margret Atwood
The Booker Prize for the year 2019 has been given jointly to Margaret
Atwood and Bernardine Evaristo. Canada’s 79-year-old Atwood won this
prize for her novel ‘The Testament’ while Bernardine Evaristo was selected
for her work ‘Girl, Women and Other.’ Evaristo is the first black women
winner of this award.
5. (b) 50
Jammu and Kashmir government has announced to set up 50 PCOs to
provide free of cost phone facility in Kashmir region. These PCOs will be set
up by BSNL and people will be able to talk from these designated places at
no charge.
6. (d) Food Safety Mitra
Union Minister for Health and Family Welfare Dr Harsh Vardhan launched
Food Safety Mitra Scheme to engage motivated individuals with the food
safety ecosystem at ground level. He also launched Eat Right Jacket and Eat
Right Jhola.
7. (c) Satish Reddy
Dr. Reddy's Chairman K.K. Satish Reddy has been appointed as the new
President of the Indian Pharmaceutical Alliance for 2019-21. Panacea Biotec
Limited’s MD will continue as Vice President.
8. (d) Japan
Japan and India are jointly organizing ‘Shinyuu Maitri’ military exercise to
extend bilateral relations. Japanese Air Force’s C2 aircraft, air crew and
observers are participating in this exercise. The Indian Air Force is taking
part in this exercise along with C17 and N-32 aircraft, air crew and
observers.
9. (a) Anoop Kumar Mendiratta
Serving District and Sessions Judge Anoop Kumar Mendiratta was appointed
as Union Law Secretary. He has been appointed till 30 March 2023 on a
contractual basis.
10. (b) Indian Military
The Indian Army organized Sangam Youth Festival at Bhaderwah in Jammu
and Kashmir. About 1500 students of 32 different colleges and universities
participated in the Sangam Youth Festival held in Jammu and Kashmir.
OCT16

1. How many matches in international cricket Mithali Raj has won as Indian
women team’s captain?
a. 100
b. 150
c. 200
d. 250
2. The International Monetary Fund (IMF) has reduced India's GDP growth
forecast for the year 2019-20 from 7 percent to what percentage?
a. 6.9 percent
b. 6.1 percent
c. 6.8 percent
d. 6.5 percent
3. What is the rank of India in Global Hunger Index 2019?
a. 94
b. 99
c. 102
d. 105
4. Who among the following has won Booker Prize 2019?
a. Margret Atwood
b. Nilima Riddhi
c. Jenifer Williamson
d. Pedro Roberto
5. How many PCOs to be established in the Kashmir region to improve
phone connectivity in the area?
a. 40
b. 50
c. 60
d. 70
6. Which scheme was recently launched by FSSAI on the occasion of World
Food Day?
a. Food Quality Scheme
b. Health and Food Relation Scheme
c. Food Supply Gram
d. Food Safety Mitra
7. Who has been recently selected as president of Indian Pharmaceutical
Alliance?
a. Jatin Prasad
b. Vivek Tomar
c. Satish Reddy
d. Arjun Patel
8. India and which country is organizing a joint military exercise ‘Shinyuu
Maitri’?
a. Nepal
b. China
c. Russia
d. Japan
9. Who has been recently appointed as new law secretary?
a. Anoop Kumar Mendiratta
b. Abhijit Banerjee
c. Dev Patel Sahay
d. Vivek Goyal
10. Which Indian Armed Forces recently organized the Sangam Youth
Festival?
a. Indian Air Force
b. Indian Military
c. Indian Navy
d. Para Military Force
Answers
1. (a) 100
Mithali Raj has achieved this feat after winning the ODI series 3–0 against
South Africa. Mithali Raj has captained 167 matches. She is only the second
female captain to win 100 matches in women's cricket after England's
Charlett Edwards.
2. (b) 6.1 percent
The International Monetary Fund (IMF) has reduced global economic growth
forecast to 3 percent during 2019-20. IMF estimated India’s GDP growth for
the year 2019-20 to 6.1 percent.
3. (c) 102
India is ranked 102 in the Global Hunger Index of 117 countries. Its aim is
to save the world from the food crisis and improve the quality of existing
food production. It also helps to know about the policies issued by various
governments around the world.
4. (a) Margret Atwood
The Booker Prize for the year 2019 has been given jointly to Margaret
Atwood and Bernardine Evaristo. Canada’s 79-year-old Atwood won this
prize for her novel ‘The Testament’ while Bernardine Evaristo was selected
for her work ‘Girl, Women and Other.’ Evaristo is the first black women
winner of this award.
5. (b) 50
Jammu and Kashmir government has announced to set up 50 PCOs to
provide free of cost phone facility in Kashmir region. These PCOs will be set
up by BSNL and people will be able to talk from these designated places at
no charge.
6. (d) Food Safety Mitra
Union Minister for Health and Family Welfare Dr Harsh Vardhan launched
Food Safety Mitra Scheme to engage motivated individuals with the food
safety ecosystem at ground level. He also launched Eat Right Jacket and Eat
Right Jhola.
7. (c) Satish Reddy
Dr. Reddy's Chairman K.K. Satish Reddy has been appointed as the new
President of the Indian Pharmaceutical Alliance for 2019-21. Panacea Biotec
Limited’s MD will continue as Vice President.
8. (d) Japan
Japan and India are jointly organizing ‘Shinyuu Maitri’ military exercise to
extend bilateral relations. Japanese Air Force’s C2 aircraft, air crew and
observers are participating in this exercise. The Indian Air Force is taking
part in this exercise along with C17 and N-32 aircraft, air crew and
observers.
9. (a) Anoop Kumar Mendiratta
Serving District and Sessions Judge Anoop Kumar Mendiratta was appointed
as Union Law Secretary. He has been appointed till 30 March 2023 on a
contractual basis.
10. (b) Indian Military
The Indian Army organized Sangam Youth Festival at Bhaderwah in Jammu
and Kashmir. About 1500 students of 32 different colleges and universities
participated in the Sangam Youth Festival held in Jammu and Kashmir.

OCT 15
1. When International Rural Women Day is observed?
a. 12 October
b. 13 October
c. 14 October
d. 15 October
2. Which rule will decide a winner during a tie match as per the new ICC
rule?
a. Super Over
b. Duckworth Luis
c. Knock Out
d. First Run
3. What is the name of Indian Origin economics who recently declared a
winner of Nobel Prize 2019 for economics?
a. Vishwajit Chatterjee
b. Abhijit Banerjee
c. SK Burman
d. Anil Joseph Mukharjee
4. Who among the following was recently elected as president of Tunisia?
a. Mohammad Albaruni
b. Altaf Hussain
c. Kais Saied
d. Iftikhar Khan
5. When World Standard Day is observed?
a. 14 October
b. 13 October
c. 12 October
d. 11 October
6. What is the name of joint military exercise that will be held between India
and Japan at Mizoram?
a. Vishva Vijeta-2019
b. Raksha Parv-2019
c. Ultra Battle-2019
d. Dharm Guardian-2019
7. The United States recently imposed sanctions on which country over its
incursion into Syria?
a. Iraq
b. Iran
c. Turkey
d. Afghanistan
8. Which Indian Airlines became the first in the world to use Taxibot on an
A-320 aircraft with passengers on board?
a. SpiceJet
b. Air India
c. IndiGo
d. Go Air
9. Justice Arup Kumar Goswami was recently sworn-in as the Chief Justice of
which High Court?
a. Sikkim High Court
b. Manipur High Court
c. Guwahati High Court
d. Chandigarh High Court
10. Who won the 2019 Booker Prize?
a. Lucy Ellmann
b. Margaret Atwood
c. Chigozie Obioma
d. Elif Shafak
Answers
1. (d) 15 October
This day is observed every day on 15 October with an objective to improve
the economic and social condition of rural women. This day also highlights
the importance of rural women in the modern-day economy and campaign
against climate change.
2. (a) Super Over
As per the ICC’s new rule, if both teams score equal runs then a super over
will decide the winner. It was also announced by ICC that super over will
continue until one team emerged as a winner.
3. (b) Abhijit Banerjee
Abhijit Banerjee, along with his wife, Esther Duflo and Michael Kramer will be
awarded the Nobel Prize for their research on 'Experiments to ending global
poverty'. Amartya Sen was given this honour in the year 1998 prior to
Abhijit Banerjee.
4. (c) Kais Saied
Kais Saied, a former law professor, won the presidential election in Tunisia
by a huge margin. He received about 77% of votes in his favour while
opposite candidate Nabil Karoui got 23% of votes. Large-scale rallies and
door-to-door campaigns were conducted during his election campaign.
5. (a) 14 October
Every year World Standard Day is observed on October 14. The day is
celebrated with the aim of creating awareness among regulators, industry
and consumers. It also raises its importance of standardization within the
global economy.
6. (d) Dharm Guardian-2019
India and Japan are organizing Dharm Guardian-2019, a joint military
exercise, at Mizoram to strengthen the military bond. It was first organized
in the year 2018. Last joint military exercise was also held at Mizoram. Both
countries share their war-related expertise.
7. (c) Turkey
The United States recently imposed new sanctions against Turkey over its
invasion into northern Syria. The sanctions call for a ceasefire, increase in
steel tariffs and cancellations of the negotiations over a trade deal with
Turkey.
8. (b) Air India
Air India became the first airline globally to use a Taxibot on an A-320
aircraft that too with the passengers on board. Taxibot is a robot-accessed
aircraft tractor that is used for taxiing the aircraft from parking bay to
runway.
9. (a) Sikkim High Court
Justice Arup Kumar Goswami was sworn in as the Chief Justice of Sikkim
High Court, succeeding Justice Vijai Kumar Bist. He was administered the
oath by Governor Ganga Prasad.
10. (b) Margaret Atwood
Margaret Atwood and Bernardine Evaristo won the 2019 Booker Prize jointly.
The Award is awarded every year for the best novel written in English
language and published in the United Kingdom.
OCT 14
1. Which medal was won by Indian boxer Manju Rani in the 48-kilogram
weight category of the Women's World Boxing Championships?
a. Gold
b. Silver
c. Bronze
d. None of the Above
2. What percent of India’s growth rate has been projected by the World
Bank for the current financial year?
a. 8%
b. 7%
c. 6%
d. 5%
3. When is World Day Against the Death Penalty observed?
a. 10 October
b. 11 October
c. 12 October
d. 13 October
4. Which country's energy company ‘Total’ has announced to buy 37.4
percent stake in Adani Gas?
a. Nepal
b. France
c. China
d. Russia
5. Which international organization has recently published a report about the
violation of labour rights in tea gardens of an Indian state?
a. Oxfam
b. WWF
c. CARTA
d. Obama Foundation
6. On which social media platform, Prime Minister Modi has become the
world's most-followed global leader?
a. Twitter
b. Facebook
c. Instagram
d. YouTube

7. What is the name of Indian nun who was declared saint by Pope Francis
recently?
a. Norah Williams
b. Jenifer Gobalchar
c. Vandana Narayan
d. Mariam Thresia
8. What is the name of the person who invented ‘Phenakistoscope’ and
Google celebrated his 218th birthday with a Doodle recently?
a. John Pattison
b. Joseph Antoine Ferdinand
c. Dracker Ontario
d. Richard Brown Williamson
9. Which Indian player has won Dutch Open Badminton 2019?
a. Harshit Chaudhary
b. Arpit Gupta
c. Vivek Joshi
d. Lakshya Sen
10. Which state/UT has started postpaid mobile service after 70 days of
ban?
a. Bihar
b. Uttar Pradesh
c. Jammu and Kashmir
d. New Delhi
Answers
1. (b) Silver
Indian boxer Manju Rani participated for the first time in this championship.
She was defeated 4–1 by Ekaterina Paltseva of Russia in the final. At the
same time, India has won 1 silver medal and 3 bronze medals in this
competition.
2. (c) 6%
In earlier estimates, India’s growth rate for 2018-19 was 6.9% but in its
recent edition of growth rate projection, the World Bank has estimated 6%
of growth rate for India. As per the World Bank’s estimate, India’s growth
rate in 2021 will be 6.9%.
3. (a) 10 October
On this day, people raise their voice against the death penalty. The United
Nations Human Rights Office has called on all countries to approve and
implement a global treaty that calls for the abolition of the death penalty.
4. (b) France
The announcement came nearly one year after both companies announced a
50:50 joint venture for natural gas imports and retail sales. After this
agreement, 'Total' and 'Adani Parivar' will own 37.4-37.4% shares. The
remaining 25.2% stake will be held by common investors.
5. (a) Oxfam
As per the Oxfam’s latest report, labours get just Rs. 137-167 despite
working for more than 12 hours every day. However, the Assam government
has recently announced to raise workers’ daily wages Rs. 351 but situation
has not yet improved. Assam is the biggest tea producer state.
6. (c) Instagram
Over 30 million people are following Prime Minister Narendra Modi on
Instagram, which is the highest number of followers of any leader in the
world. Indonesia's President Joko Widodo comes second because 25.6
million people follow him and Barack Obama comes third with 24.8 million
followers.
7. (d) Mariam Thresia
Mariam Thresia, a nun from Kerala, has been given the title of saint 93 years
after her death. Sister Mary Thresia was given the title of Saint by St.
Francis of Vatican, Pope Francis.
8. (b) Joseph Antoine Ferdinand
Joseph Antoine Ferdinand Plateau is known for his invention
‘Phenakistoscope’. This invention, later, gave birth to the modern day
cinema and moving images. He invented this device in 1832.
9. (d) Lakshya Sen
Indian badminton player Lakshya Sen won the Dutch Open title 2019. He
defeated Yusuke Onodera of Japan with 15-21, 21-14, 21-15. Lakshya Sen
belongs to the Uttarakhand state. He is the student of Prakash Padukon
Badminton Academy.
10. (c) Jammu and Kashmir
Postpaid mobile services in 10 districts of Jammu and Kashmir resumed on
October 14 afternoon. All mobile and landline services were stopped from
August 5 in the valley due to security concerns in the area. Services were
blocked since Article 370 was revoked.

OCY 12
1. What is the name of India's first female graduate (Hons) for whom Google
has recently dedicated a doodle?
a. Kamini Roy
b. Avantika Deshbandhu
c. Sapna Verma
d. Payal Trivedi
2. What is the name of NASA’s recently launched a satellite to study the
ionosphere?
a. PROP
b. ICON
c. MARK
d. ROCK
3. Who has been ranked second in the Forbes List of 100 Indian billionaires?
a. Kumar Birla
b. Hinduja Brothers
c. Azim Premji
d. Gautam Adani
4. What is the name of the first person who did spacewalk in 1965 but died
recently?
a. Alexei Leonov
b. Fyodor Gorbachov
c. Joseph Huston
d. Mark Elliot
5. Who among the following has been awarded Lifetime Achievement Award
during Indian Sports Awards 2019?
a. Mary Kom
b. Bichung Bhutia
c. Milkha Singh
d. PT Usha
6. In which country more than 3,000 women watched a football match in the
stadium for the first time in nearly 40 years?
a. Pakistan
b. Iran
c. Nepal
d. Japan
7. Which state government has announced to establish a ‘Kanyashree
University’ for women empowerment?
a. Haryana
b. Madhya Pradesh
c. Rajasthan
d. West Bengal
8. Which bank has recently reduced the marginal cost-based lending rate
(MCLR) by 0.05 percent?
a. Dena Bank
b. Punjab National Bank
c. Allahabad Bank
d. Bank of Baroda
9. Which Indian woman athlete has broken her own national record in the
59th National Open Athletics Championship and won the gold medal in the
women's 100m event?
a. Duti Chand
b. Sudha Singh
c. Vidya Rajput
d. Vrinda Nayar
10. Which country has recently deployed additional 3,000 troops to Saudi
Arabia?
a. England
b. USA
c. France
d. Germany
Answers
1. (a) Kamini Roy
Google Doodle has been made on the 155th birth anniversary of Kamini Roy.
She was the first woman in Indian history to get a graduate (Hons) degree.
He was born on 12 October 1864 in the Bakerganj district of British India
2. (b) ICON
NASA has launched a satellite named ICON to study the ionosphere.
According to information released by NASA, this mission will bring out the
information about where the Earth and the ionosphere meet. The mission
was to be launched in 2017 but due to a technical flaw, it has been launched
after two years.
3. (d) Gautam Adani
Forbes has recently released a list of Indian billionaires. Gautam Adani
gained 8 points benefit compared to the previous year to finish second.
Mukesh Ambani remains in the first place for the 12th consecutive time.
4. (a) Alexei Leonov
Alexey Leonov was the first astronaut who did a spacewalk for the first time
in the world. Alexey Leonov of the Soviet Union has died in Russia at the age
of 85 years recently. On 18 March 1965, he took a spacewalk for 12 minutes
in the space.
5. (c) Milkha Singh
India’s legendary athlete Milkha Singh was recently awarded Lifetime
Achievement Award 2019. Bajrang Poonia received the Sportsman of the
Year award in the men's category while Vinesh Phogat received the
Sportswoman of the Year title.
6. (b) Iran
After the 1979 Islamic revolution in Iran, women were banned from
watching men's sports. Earlier, the international football federation FIFA had
suggested suspending Iran’s team due to male-only policy. Iran is the last
country in the world where women are not allowed to watch football
matches.
7. (d) West Bengal
Chief Minister Mamata Banerjee announced that for the empowerment of
girls, the West Bengal government has decided to open Kanyashree
University in Nadia district and Kanyashree Colleges across the state. It was
launched in 2013 and the state government has allocated a budget of seven
thousand crores for it.
8. (c) Allahabad Bank
This deduction will be applicable from 14 October 2019. Now, the bank's
one-year benchmark MCLR will come down from 8.40 percent to 8.35
percent. Similarly, for one-day to six-month loans, the MCLR will be between
7.85 to 8.20 percent.
9. (a) Duti Chand
Duti Chand has broken her own record of 11.26 seconds at the Asian
Championship with a time of 11.22 seconds in the semi-finals of the
tournament. It is also a new national record. Earlier, with a time of 11.26
seconds, this record was jointly made with Rachita Mistry.
10. (b) USA
Special US Envoy for Iran, Brian Hook recently announced that the United
States is ready to deploy additional 3,000 troops to Saudi Arabia. Troops will
be deployed to Saudi Arabia with the aim of providing more security to the
oil facilities.

OCT 11
1. Who won the 2019 Nobel Peace Prize?
a) Abiy Ahmed Ali
b) Greta Thunberg
c) Jacinda Ardern
d) Isaias Afwerki
2. Who has become the first Indian captain to score 40 hundreds?
a) MS Dhoni
b) Rohit Sharma
c) Virat Kohli
d) Ravindra Jadeja
3. Which city is hosting the historic second informal summit between PM
Modi and Chinese President Xi Jinping?
a) Mamallapuram
b) Mallapuram
c) Idukki
d) Coimbatore
4. The Union Government has launched which scheme to assure no-cost
health care facilities to pregnant women and newborns?
a) POOJA
b) ARPAN
c) SUMAN
d) SHRESHTHA
5. Government e-Marketplace (GeM) signed an MoU with which bank to offer
funds transfer services to its registered users?
a) State Bank of India
b) Punjab National Bank
c) Union Bank of India
d) Central bank of India
6. Anita Pai has been appointed as the Chief Operating Officer of which
bank?
a) HDFC
b) Yes Bank
c) Axis Bank
d) ICICI Bank
7. Which Ministry launched the ‘mHariyali’ App recently to encourage Public
engagement in planting trees?
a) Ministry of Environment, Forest and Climate Change
b) Ministry of Housing & Urban Affairs
c) Ministry of Home Affairs
d) Ministry of Corporate Affairs
8. How many Jawahar Navodaya Vidyalayas were inaugurated recently by
Union Minister Ramesh Pokhriyal ‘Nishank’?
a) 10
b) 8
c) 7
d) 5
9. When is the International Day of the Girl Child observed every year?
a) 10th October
b) 11th October
c) 12th October
d) 13th October
10. Which United Nations’ agency recently became the first to make
transactions in cryptocurrency?
a) UNESCO
b) UNICEF
c) UNEP
d) UNHCR
Answers
1. (a) Abiy Ahmed Ali
Ethiopian Prime Minister Abiy Ahmed Ali has been awarded the 2019 Nobel
Peace prize for his efforts to secure a peace deal between Ethiopia and its
neighbour Eritrea and put an end to the over 20-year-old border conflict.
2. (c) Virat Kohli
Virat Kohli created a new record on October 11, 2019 by becoming the first
Indian captain to score 40 hundreds. He achieved the feat after he scored
his 26th test century during India’s second test against South Africa in
Pune.
3. (a) Mamallapuram
The historic second informal summit between PM Modi and Xi Jinping will
take place at Mamallapuram in Tamil Nadu. Chinese President Xi Jinping
arrived in Chennai, India on October 11 for a two-day visit.
4. (c) SUMAN
The Union Government has launched Surakshit Matritva Aashwasan or
SUMAN scheme to provide quality healthcare facilities at no cost to pregnant
women, new mothers and newborns.
5. (c) Union Bank of India
The Government e-Marketplace (GeM) signed a MoU with the Union Bank of
India (UBI) to offer financial services to the registered users such as funds
transfer, advice on Performance Bank Guarantees and Earnest Money
Deposit (EMD).
6. (b) Yes Bank
Anita Pai has been appointed as the Chief Operating Officer (COO) of Yes
Bank. Jasneet Bachal was appointed as the CMO of the bank.
7. (b) Ministry of Housing & Urban Affairs
Hardeep S Puri, Minister of State for Housing & Urban Affairs recently
launched the ‘mHariyali’ mobile app to motivate people in planting trees. The
mobile app offers automatic geo-tagging of plants.
8. (d) 5
Union Human Resource Development Minister, Ramesh Pokhriyal ‘Nishank’
on October 11, 2019 inaugurated 5 Jawahar Navodaya Vidyalayas (JNVs)
and laid the foundation stone of 9 new JNVs.
9. (b) 11th October
The International Day of the Girl Child is observed every year on 11 October
to highlight the challenges faced by girls globally and promote girls'
empowerment and their human rights. The Theme of International Day of
the Girl Child 2019 was Girl Force: Unscripted and Unstoppable.
10. (b) UNICEF
The United Nations International Children's Emergency Fund (UNICEF)
became the first UN agency to carry out transactions in cryptocurrency
through its newly-launched UNICEF Cryptocurrency Fund.

OCT10
1. India has condemned which country’s military offensive in Syria?
a) United States
b) Turkey
c) Iran
d) Israel
2. Which state has lifted travel restrictions on tourists?
a) New Delhi
b) Maharashtra
c) Bihar
d) Jammu and Kashmir
3. When was World Mental Health Day 2019 observed?
a) October 9th
b) October 5th
c) October 10th
d) October 8th
4. After Union Cabinet’s approval, what is the new honorarium amount that
will be given to ASHA workers?
a) Rs 3000
b) Rs 2000
c) Rs 1000
d) Rs 1500
5. Which Programme has been launched by the Union Government to
encourage talented children to enrich their skills?
a) DHRUV
b) LAKSHAY
c) PRATIBHA
d) PRATIGYA
6. Who has been awarded with the 2018 Nobel Prize in Literature?
a) Olga Tokarczuk
b) Patrick Modiano
c) Alice Munro
d) Mo Yan
7. Who won the Nobel Prize in Literature 2019?
a) Olga Tokarczuk
b) Peter Handke
c) Kazuo Ishiguro
d) Bob Dylan
8. Which Bank has recently reduced the lending rates by ten basis points?
a) PNB
b) SBI
c) OBC
d) PMC
Answers with Explanation
1. (b) Turkey
India has hit out against Turkey for its unilateral military offensive in north-
east Syria. India called on Turkey to exercise restraint and respect Syria’s
sovereignty and territorial integrity.
2. (d) Jammu and Kashmir
Jammu and Kashmir has lifted the travel ban on tourists, after almost two
months. A security advisory asking all tourists to leave the valley with
immediate effect due to intelligence on terror threats was issued on August
2, 2019.
3. (c) October 10th
The World Mental Day was observed across the world on October 10, 2019
with the theme ‘suicide prevention’. The day’s main aim is to raise
awareness regarding mental health issues.
4. (b) Rs 2000
The Union Cabinet, chaired by PM Narendra Modi gave its approval to double
the honorarium for ASHA workers to Rs 2000/ month.
5. (a) DHRUV
Union Ministry of Human Resource Development recently launched the
Pradhan Mantri Innovative Learning Programme – DHRUV to encourage
talented children to enrich their skills and knowledge. The Programme was
launched from the ISRO Headquarters at Bengaluru. Under the programme,
the selected student will be named as Dhruv Tara.
6. (a) Olga Tokarczuk
Olga Tokarczuk has been awarded with the 2018 Nobel Prize in Literature.
The Prize was postponed in 2018 for the first time in 70 years amidst the
sexual assault scandal when one of the Academy members Katarina
Frostenson resigned due to the sexual assault charges against her husband
Jean-Claude Arnault.
7. (b) Peter Handke
Austrian author Peter Handke has been awarded with the 2019 Nobel Prize
for Literature. The Nobel Prize in Literature is considered as the most
prestigious award in the literary world.
8. (b) SBI
The State Bank of India (SBI) on October 10, 2019 reduced its lending rates
by ten basis points and revised the savings rate by 25 basis points for fixed
deposits under Rs 1 lakh.

OCT 9
1. Who among the following won the Nobel Prize in Chemistry 2019?
a) Frances H. Arnold, George P. Smith, Sir Gregory P. Winter
b) John B Goodenough, M Stanley Whittingham and Akira Yoshino
c) William Kaelin Jr, Sir Peter Ratcliffe and Gregg Semenza
d) James Peebles, Michel Mayor and Didier Queloz
2. Who among the following won the Nobel Prize in Physics 2019?
a) Frances H. Arnold, George P. Smith, Sir Gregory P. Winter
b) William Kaelin Jr, Sir Peter Ratcliffe and Gregg Semenza
c) James Peebles, Michel Mayor and Didier Queloz
d) John B Goodenough, M Stanley Whittingham and Akira Yoshino
3. When is World Post Day observed?
a) October 9th
b) October 5th
c) October 8th
d) October 4th
4. India was ranked at which position at the Global Competitiveness Index
2019?
a) 57
b) 65
c) 68
d) 63
5. Which economy topped the Global Competitiveness Index 2019?
a) United States
b) Switzerland
c) Denmark
d) Singapore
6. Union Cabinet recently approved additional installment of how much
percent Dearness Allowance to Government employees?
a) 5%
b) 7%
c) 10%
d) 12%
7. Union Cabinet recently approved inclusion of how many displaced families
in Cabinet approved Rehabilitation Package?
a) 1,500 displaced families
b) 2,700 displaced families
c) 4,100 displaced families
d) 5,300 displaced families
8. Cabinet recently relaxed the requirement of Aadhaar Card seeded data to
avail of the benefits of which scheme?
a) Pradhan Mantri Jan Arogya Yojana (PMJAY)
b) Pradhan Mantri Kisan Samman Nidhi (PM-Kisan)
c) Pradhan Mantri Shram Yogi Maan-dhan (PM- SYM)
d) Pradhan Mantri Awas Yojana (PMAY)
9. When is the World Mental Health Day observed every year?
a) 8th October
b) 9th October
c) 10th October
d) 11th October
10. Which newspaper recently announced to stop its print venture and go
digital completely?
a) Daily News & Analysis
b) The Statesman
c) The Millennium Post
d) Business Today
Answers
1. (b) John B Goodenough, M Stanley Whittingham and Akira
Yoshino
The Nobel Prize in Chemistry was jointly given to M Stanley Whittingham,
John B Goodenough and Akira Yoshino for developing the rechargeable,
lightweight, powerful and energy efficient lithium-ion batteries.
2. (c) James Peebles, Michel Mayor and Didier Queloz
The Nobel Prize in Physics 2019 was awarded jointly, one half to James
Peebles for the theoretical discoveries in physical cosmology and the other
half jointly to Michel Mayor and Didier Queloz for the discovery of an
exoplanet.
3. (a) October 9th
World Post Day is observed globally on October 9, to commemorate the
establishment anniversary of Universal Postal Union and highlight the
significant role of postal services.
4. (c) 68
India slipped 10 places to be ranked at the 68th position in the annual
Global Competitiveness Index 2019. Singapore topped the index with the
highest score, displacing the United States.
5. (d) Singapore
Singapore with a score of 84.8 took the top rank in the Global
Competitiveness Index 2019, pushing the United States to the second spot.
6. (a) 5%
Union Cabinet on October 9, 2019 approved additional installment of 5
percent Dearness Allowance (DA) to Government employees; and Dearness
Relief (DR) to pensioners with effect from July 1, 2019. This additional
installment would cost Rs 15909.35 crore per annum extra to the Central
Government.
7. (d) 5,300 displaced families
Union Cabinet on October 9, 2019 approved the inclusion of 5,300 Displaced
(DP) families of Jammu & Kashmir of 1947 in Cabinet approved
Rehabilitation Package that was approved on November 30, 2016. These
Displaced families are those who initially moved outside the Jammu &
Kashmir but later returned and settled in J&K itself.
8. (b) Pradhan Mantri Kisan Samman Nidhi (PM-Kisan)
Union Cabinet recently relaxed the requirement of Aadhaar seeded data to
receive the benefits of the Pradhan Mantri Kisan Samman Nidhi (PM-Kisan)
scheme. Under the PM-Kisan scheme, income support of Rs 6,000 per year
is being provided to landholding farmer families.
9. (c) 10th October
The World Mental Health Day is observed every year on October 10, 2019 to
spread awareness about the mental health issues faced by people around
the world. The Day was first celebrated in 1922.
10. (a) Daily News & Analysis
Daily News & Analysis (DNA), the 14-year-old newspaper under the parent
Zee Group, will see its last print on October 10, 2019 in Mumbai and
Ahmedabad. Facing the liquidity crunch, the brand will now go digital.

OCT 7
1. Who won the 2019 Nobel Prize in Medicine?
a) William G Kaelin Jr, Peter J Ratcliffe & Gregg L Semenza
b) James P. Allison & Tasuku Honjo
c) Michael Rosbash, Michael W Young & Jeffrey C Hall
d) Yoshinori Ohsumi
2. Who will perform Shastra Puja in France after receiving the first Rafale
fighter jet?
a) PM Narendra Modi
b) President Ram Nath Kovind
c) Rajnath Singh
d) Nirmala Sitharaman
3. Which organisation hosted the celebrations of the first World Cotton Day
in Geneva?
a) United Nations
b) World Trade Organisation
c) World Bank
d) International Cotton Association
4. Justice Ajai Lamba recently took the oath as Chief Justice of which High
Court?
a) Madras High Court
b) Bombay High Court
c) Guwahati High Court
d) High Court of Punjab and Haryana
5. India recently received the first tranche of details about the Indian
Citizens account in which institution?
a) Swiss Banks
b) Panama Papers
c) Singapore Banks
d) Cayman Islands Banks
6. Who has been awarded with the UNEP’s Asia Environmental Enforcement
Award 2019?
a) Ram Nath Kovind
b) Prakash Javadekar
c) Ramesh Pandey
d) Maneka Gandhi
7. When was the first World Cotton Day celebrated?
a) October 7, 2019
b) October 5, 2019
c) October 2, 2019
d) October 6, 2019
8. The Supreme Court directed which state government to assure that no
more trees will be cut in Aarey colony?
a) Delhi
b) Uttar Pradesh
c) Karnataka
d) Maharashtra
9. How much monetary assistance will be provided to the families of battle
casualties after Government’s latest hike?
a) Rs 8 lakh
b) Rs 5 lakh
c) Rs 10 lakh
d) Rs 6 lakh
10. Which country has fined 80 people and groups for alleged 1MDB
payments?
a) Singapore
b) China
c) Malaysia
d) Thailand
Answers with Explanation
1. (a) William G Kaelin Jr, Peter J Ratcliffe & Gregg L Semenza
Scientists William G Kaelin, Jr, Peter J Ratcliffe and Gregg L Semenza jointly
won the 2019 Nobel Prize in Medicine for their discoveries of "how cells
sense and adapt to oxygen availability”.
2. (c) Rajnath Singh
Defence Minister Rajnath Singh will be performing a Shastra Puja in France
on the occasion of Dussehra after receiving India’s first Rafale fighter jet.
Rajnath Singh will be receiving the first Rafale jet in Bordeaux, the French
port city on October 8, 2019.
3. (b) World Trade Organisation
World Trade Organisation hosted the first World Cotton Day on October 7,
2019 in Geneva, Switzerland. Smriti Irani, the Union Textiles Minister is
representing India at the World Cotton Day celebrations.
4. (c) Guwahati High Court
Justice Ajai Lamba took the oath as the Chief Justice of Guwahati High Court
on October 7, 2019. He was administered the oath of secrecy by Governor
Professor Jagdish Mukhi at the Raj Bhawan, Guwahati.
5. (a) Swiss Bank
India on October 7, 2019 received the first tranche of details of the accounts
opened by Indian citizens in Swiss Banks. The information was revealed
under the exchange of information framework that was negotiated between
India and Switzerland earlier in 2018.
6. (c) Ramesh Pandey
Ramesh Pandey, the Indian Forest Service officer, was selected for the
UNEP’s Asia Environmental Enforcement Award 2019. The United Nations
Environment Programme (UNEP) will be bestowing the award on Pandey on
November 13, 2019.
7. (a) October 7, 2019
The first-ever World Cotton Day was celebrated by the WTO on October 7,
2019 to highlight the challenges of cotton economies. The Day aims to
provide exposure to cotton and all its stakeholders in production,
transformation and trade.
8. (d) Maharashtra
The Supreme Court directed the Maharashtra state government to assure
that no more trees will be cut in Aarey colony. The apex court delivered the
ruling after hearing petitions against Mumbai Metro’s move of felling trees in
the colony to build a metro car shed.
9. (a) Rs 8 Lakh
Defence Minister Rajnath Singh has approved a four-fold increase in the
monetary assistance provided to the families of battle casualties. The
families will now be provided with a sum of Rs 8 lakh, unlike earlier Rs 2
lakh.
10. (c) Malaysia
Malaysia has ordered 80 people and groups to pay fines amounting to about
USD 100m for allegedly receiving funds from the 1MDB state investment
fund.

OCT 5
1. Which Indian cricketer has become the first Indian player who played 100
T20 internationals?
a. Smriti Mandhana
b. Harmanpreet Kaur
c. Virat Kohli
d. Rohit Sharma
2. Which PSU company has announced to launch an initiative to turn plastic
into bitumen?
a. Indian Oil
b. NTPC
c. SAIL
d. GAIL
3. Which day World Teachers Day is celebrated?
a. 03 October
b. 04 October
c. 05 October
d. 06 October
4. Which Indian player has become the first Indian athlete to qualify for
Tokyo Olympics 2020 in individual competition?
a. Avinash Sable
b. Mridul Pandey
c. Kapil Patra
d. Yogesh Awasthi
5. Who was recently named as MD and CEO of Tata Communications?
a. R. Srinivasan
b. Y.V. Giri
c. A.S. Lakshminarayanan
d. Pradyut Bonwal
6. Which state has launched YSR Vahana Mitra scheme recently?
a. Telangana
b. Odisha
c. Maharashtra
d. Andhra Pradesh
7. What is the name of audio guide facility app launched by Ministry of
Tourism?
a. Audio Odigos
b. Radio Guide
c. Mobile Guide
d. Guide on Drive
8. Who among the following was recently appointed as chief justice of
Punjab & Haryana High Court?
a. S. Manikumar
b. Ajay Lamba
c. Arup. K. Goswami
d. Ravishankar Jha
9. Which Union Ministry has instituted Vayoshreshtha Samman-2019?
a. Ministry of Human Resource Development
b. Ministry of Social Justice and Empowerment
c. Ministry of Culture
d. Ministry of Women and Child Development
10. How many states including Delhi, Uttar Pradesh have issued 15 point
guidelines to save Ganga from pollution?
a. 11
b. 12
c. 13
d. 14

Answers with Explanation


1. (b) Harmanpreet Kaur
Harmanpreet Kaur has become first-ever Indian cricketer who played 100 T-
20 international matches. Rohit Sharma played 95 T20 international
matches. New Zealand’s Suzi Bates is on the top of the list who have played
111 matches.
2. (a) Indian Oil
The Public Sector Undertaking (PSU) Indian Oil has announced to start a
venture that can transform plastic into bitumen. The company said that
bitumen will be used commercially for road construction. The first short
patch of bitumen road was constructed at Faridabad under this initiative.
3. (c) 05 October
World Teachers Day 2019 is being celebrated across the globe with a theme
– Young Teachers: The future of the Profession. UNESCO said that
youngsters should come forward to take this profession. It has been
observed in recent times that incidents of violence against teachers have
been increased across the globe.
4. (a) Avinash Sable
India’s Avinash Sable has become first Indian athlete who qualified for Tokyo
Olympics 2020 in an individual competition. The qualifying mark for Tokyo
Olympic was 8.25.23 minutes while Avinash finished 8.22.51 minutes.
5. (c) A.S. Lakshminarayanan
Tata Communications has named as A.S. Lakshminarayanan MD and CEO of
Tata Communications. He will communicate with the interim management
committee of Tata Communications. He has a vast experience of 35 years in
the same field.
6. (d) Andhra Pradesh
The YSR Vahana Mitra scheme will provide Rs. 10,000 allowance to self-
employed drivers of Andhra Pradesh. Chief Minister of Andhra Pradesh YS
Jagan Mohan Reddy launched this scheme. As per the norms of the scheme,
the allowance will be credited into the bank account of the beneficiary.
7. (a) Audio Odigos
The audio guide app ‘Audio Odigos’ launched for 12 states of India by the
Ministry of Tourism. Now, tourists can look at the historical insights of the
iconic monuments of India. It will provide inbuilt site map and navigation
facility to the tourists.
8. (d) Ravishankar Jha
Ravishankar Jha was recently appointed as Chief Justice of Punjab and
Haryana High Court while S. Manikumar, Ajay Lamba and Arup K. Goswami
were appointed as Chief Justices of Kerala, Guwahati and Sikkim High
Courts.
9. (b) Ministry of Social Justice and Empowerment
President Ram Nath Kovind has recently conferred Vayoshreshtha Samman-
2019 to prestigious senior citizens and institutions working for the welfare of
old people. This award has been instituted by the Ministry of Social Justice
and Empowerment.
10. (a) 11
The Clean Ganga National Mission has given these instructions to Delhi,
Uttarakhand, Uttar Pradesh, Bihar, Jharkhand, West Bengal, Himachal
Pradesh, Madhya Pradesh, Chhattisgarh, Rajasthan, Haryana under section
five of the Environmental Protection Act 1986. After this order, no idols can
be immersed in the Ganges and its tributaries on festivals or on ordinary
days.

OCT 4
1. Recently government has launched which portal to improve coal supply to
power plants?
a) Deepak
b) Prakash
c) Light
d) Ujjwala
2. Who from the following became the second fastest Indian to take 200
Test wickets?
a) Ishant Sharma
b) Md. Shami
c) Ravindra Jadeja
d) Hanumant Bihari
3. What is the household income limit for borrowers of NBFC-MFI in rural
areas?
a) 1 lakh
b) 1.5 lakh
c) 1.25 lakh
d) 2 lakh
4. Who became the new president of Rajasthan Cricket Association?
a) Vaibhav Gehlot
b) Amar Sharma
c) Akash Chopra
d) Nikil Arora
5. Which country’s PM arrived for an official visit in India on October 3,
2019?
a) Bangladesh
b) Mauritius
c) Maldives
d) Nepal
6. Which state government has decided to purchase single-use plastic waste
to make the state plastic free?
a) Himachal Pradesh
b) Bihar
c) Jharkhand
d) Rajasthan
7. Haza Al Mansuri is the first astronaut of which country who returned from
the International Space Station recently?
a) Jordan
b) Kuwait
c) Iran
d) UAE
8. What is the name of Chandrayaan-2’s payload that found charged
particles on the Moon?
a) URJA
b) CLASS
c) RUSTAM
d) CLOUD
9. Which of the following countries have announced to leave membership of
OPEC?
a) Ecuador
b) Brazil
c) Venezuela
d) Chile
10. In which place India's first floating basketball court has started?
a) Goa
b) Panji
c) Chennai
d) Mumbai
Answers with Explanation
1. (b) Prakash
Prakash-(Power Rail Koyla Availability through Supply Harmony) portal will
also help in mapping coal stock in mines as well as monitor the movement of
railway rakes and the availability of coal in power plants for stakeholders.Full
name of Prakash Portal. The main objective of the government is to improve
coordination between the ministries of power, coal and railways to ensure
coal supply to power plants.
2. (c) Ravindra Jadeja
Ravindra Jadeja has achieved this feat during the test match between India
and S.Africa. He is also the quickest amongst left arm bowlers in the World
cricket.
3. (c) 1.25 lakh
Recently, in a new policy declaration RBI announced that income limit for
rural areas to borrow from NBFC-MFI are raised to Rs.1.25 lakh from Rs. 1
lakh. Whereas, in urban and semi-urban it is Rs. 2 lakh.
4. (a) Vaibhav Gehlot
Vaibhav Gehlot is the son of Rajasthan’s CM Ashok Gahlot. He won with a
good margin of 25-6 against his nearest rival Ram Prakash Choudhary.
5. (a) Bangladesh
Bangladesh’s PM Sheikh Hasina arrived for a four-day visit to India on
October 3, 2019. The visit is aimed at strengthening India-Bangladesh ties
amidst the tensions after Assam’s NRC exercise.
6. (a) Himachal Pradesh
Himachal Pradesh government will buy back single-use plastic waste at Rs.
75 per kg. The plastic waste comprising polythene bags, chips packets,
wrappers etc. Himachal Pradesh had banned use of polythene in 2009.
7. (d) UAE
Haza Al Mansuri is the first astronaut of the United Arab Emirates, who flew
to the International Space Space Station in Russian spacecraft Soyuz and
returned recently. He spent eight days in ISS along with US and Russian
astronauts.
8. (b) CLASS
Chandrayaan-2's Orbiter Payload CLASS has claimed to found 'charged
particles' on the Moon. According to the information released by ISRO, the
solar air flow from the Sun to the Moon's orbit was recorded.
9. (a) Ecuador
Ecuador has decided to leave membership of the OPEC group. This decision
of Ecuador will be effective from January 1, 2020. Ecuador has decided to
leave OPEC due to internal financial problems.
10. (d) Mumbai
India's first floating basketball court has been started in Mumbai by the
National Basketball Association (NBA). India is organizing first NBA game in
Mumbai on October 4, 2019.

OCT 3
1. Under Swachh Bharat Mission, India built how many toilets for 60 crore
people?
a) 11 crore
b) 15 crore
c) 10 crore
d) 20 crore
2. The UK court dismissed Pakistan’s claims and ruled in favour of India in a
legal battle involving the funds of the 7th Nizam of Hyderabad. How much
fund was involved?
a) £35m
b) £12m
c) £50m
d) £45m
3. Who among the following accepted Punjab CM Amarinder Singh’s invite to
attend the inaugural event of Kartarpur corridor in Pakistan?
a) PM Narendra Modi
b) President Ram Nath Kovind
c) Former PM Manmohan Singh
d) Rajnath Singh
4. All three members of BCCI's Cricket Advisory Committee (CAC) submitted
their resignations recently. Who among the following was not one of the
members.
a) Kapil Dev
b) Shantha Rangaswamy
c) Anshuman Gaekwad
d) Ravi Shastri
5. Who among the following scored his maiden test century during India’s
first test against South Africa?
a) Cheteshwar Pujara
b) Hanuma Vihari
c) Shubman Gill
d) Mayank Agarwal
6. Which country released postage stamps on the occasion of 150th birth
anniversary of Mahatma Gandhi?
a) Israel
b) France
c) Germany
d) Italy
7. Which state government announced a complete ban on tobacco and pan-
masala products in the state?
a) Rajasthan
b) Uttar Pradesh
c) Jharkhand
d) Odisha
8. Who has been appointed as executive director for India at IMF?
a) Rajiv Gopalachari
b) Surjeet Bhalla
c) S. Nariman
d) Vivek Mohan
9. PM Narendra Modi released a commemorative coin worth this much value
on Mahatma Gandhi’s 150th birth anniversary.
a) Rs. 150
b) Rs. 200
c) Rs. 250
d) Rs. 300
10. The newly-launched Vande Bharat train will take how many hours to
reach Katra from Delhi?
a) 11
b) 10
c) 9
d) 8
Answers
1. (a) 11 crore
PM Modi declared India open defecation free on October 2, 2019. The Prime
Minister announced that in 60 months, the government has been able to
provide 11 crore toilets to over 60 crore people.
2. (a) £35m
India has won a decades-old legal dispute over funds worth £35m that
belonged to the 7th Nizam of Hyderabad at the UK Court, while Pakistan’s
claim over the money was dismissed.
3. (c) Former PM Manmohan Singh
Former PM Manmohan Singh accepted Punjab CM Captain Amarinder’s invite
to join the first Jatha to Kartarpur Sahib Gurudwara in Pakistan on
November 9. Manmohan Singh will also attend the main event at Sultanpur
Lodhi in Punjab, India.
4. (d) Ravi Shastri
All three members of the BCCI’s Cricket Advisory Committee (CAC)
submitted their resignations recently. The committee, headed by Kapil Dev,
also comprised Shantha Rangaswamy and Anshuman Gaekwad. The
members resigned from their post after they were served with a notice for
conflict of interest by the BCCI Ethics officer, Retired Justice DK Jain.
5. (d) Mayank Agarwal
Indian opener, Mayank Agarwal, scored his maiden test century on October
3, 2019 after a record-first wicket partnership with Rohit Sharma in India’s
first test against South Africa at ACA-VDCA Stadium, Visakhapatnam.
6. (b) France
On the occasion of the 150th birth anniversary of Mahatma Gandhi, a
postage stamp bearing the photograph of Gandhiji has been issued in
France. French postal service company La Poste has released this stamp.
Similar stamps were issued on Gandhi Jayanti in many countries of the world
- Uzbekistan, Turkey, Palestine and other countries.
7. (a) Rajasthan
Rajasthan has imposed a complete ban on all types of tobacco products
including storage, advertising and sale of pan-masala products. Earlier, two
states Maharashtra and Bihar also banned tobacco and pan-masala products.
8. (b) Surjeet Bhalla
Surjeet Bhalla is a famous economist and former member of economic
advisory council of Prime Minister of India. Earlier, RBI’s former deputy
Governor Subir Gokarn was working as India’s executive director at IMF but
he died on July 30 this year.
9. (a) Rs. 150
Prime Minister Narendra Modi paid tribute to the Father of the Nation,
Mahatma Gandhi at the Sabarmati Ashram in Ahmedabad to mark his 150th
birth anniversary and released a commemorative coin worth Rs 150 on the
occasion.
10. (d) 8 hours
Home Minister Amit Shah flagged off India’s fastest train, Vande Bharat
Express on October 3, 2019. This train will reach Vaishno Devi (Katra) from
Delhi in 8 hours while other trains usually take 12-13 hours. The train has
total 16 coaches with 14 chair car coaches.

OCT 1
1. Who has become the first girl student to win President of India Prize?

a) Subimal Ghosh
b) Lipi Thukral
c) Kavitha Gopal
d) Divya Agrawal
2. When will single-use plastic be banned in India?
a) October 1st
b) October 2nd
c) October 5th
d) October 4th
3. When will be the next hearing of J&K petitions?
a) November 14th
b) November 1st
c) October 31st
d) October 12th
4. “All Hindu, Sikh, Christian, Jain and Buddhist refugees will get Indian
citizenship.” Whose statement is this?
a) Narendra Modi
b) Amit Shah
c) Rajnath Singh
d) Sonia Gandhi
5. Who was awarded the GN Ramachandran Gold Medal by President Ram
Nath Kovind?
a) Prof. Amitabha Chattopadhyay
b) Dr Amol Prakash
c) Dr Bodhisatwa Hazra
d) Dr Prabhat Ranjan Prem
6. How many countries including India have signed an agreement to stop the
spread of online fake news?
a) 25
b) 20
c) 22
d) 28
7. Which state topped the 'School Education Quality Index' released by NITI
Aayog?
a) Uttar Pradesh
b) Bihar
c) Kerala
d) Punjab
8. When is International Coffee Day celebrated?
a) October 1st
b) October 2nd
c) October 3rd
d) October 4th
9. What is the strike range of land-attack version of BrahMos supersonic
cruise missile?
a) 240km
b) 290km
c) 310km
d) 360km
10. Which birth anniversary of Father of the Nation Mahatma Gandhi is being
celebrated on October 02, 2019?
a) 150
b) 160
c) 170
d) 180
Answers
1. (c) Kavitha Gopal
Kavitha Gopal has made history by becoming the first girl student in IIT
Madras to win the President of India Prize. Previously, only male students
had ever won the prize.
2. (b) October 2nd
Single-use plastic will be banned in India from October 2, 2019, on the
150th birth anniversary of Mahatma Gandhi.
3. (a) November 14th
The Supreme Court has listed the hearing on J&K petitions to November 14,
2019. The top court has given the Union Government 28 days to reply to all
the petitions.
4. (b) Amit Shah
Union Home Minister Amit Shah assured recently that all Hindu, Sikh,
Christian, Jain and Buddhist refugees will get Indian citizenship. The Minister
said that for the same, the Citizenship (Amendment) Bill will be passed in
the Parliament.
5. (a) Prof. Amitabha Chattopadhyay
The President presented the GN Ramachandran Gold Medal to Prof.
Amitabha Chattopadhyay of CSIR-CCMB.
6. (b) 20
Twenty countries have committed to promote the credible news only after
taking action against fake news. Fake news has become a challenge for the
world. It is largely misused during elections. In Malaysia, fake news can lead
to 6 years in jail.
7. (c) Kerala
Kerala topped the 'School Education Quality Index', which was recently
launched by NITI Aayog. Rajasthan was at second position and Karnataka
was at third position on the list. NITI Aayog prepared this index on the basis
of 2016-17 data.
8. (a) October 1
The International Coffee Day is celebrated on October 1. It is an occasion
that is used to promote and celebrate coffee and its importance.
International Coffee Organisation (ICO) has marked the day as an
International Day in 2014. It is believed that Coffee is the second most
traded commodity after crude oil.
9. (b) 290km
DRDO has recently test-fired a land-attack version of BrahMos supersonic
cruise missile. It has a strike range of 290km. It can be fired from land and
sea-based platforms. Newly, test fired BrahMos missile has been fitted with
indigenous equipments.
10. (a) 150
The nation is celebrating the 150th birth anniversary of Father of the Nation
Mahatma Gandhi on 02 October 2019. On the 150th birth anniversary of
Mahatma Gandhi, many programs will be held across the country and the
world.

SEP30
1. Which fast fashion retailer has filed for bankruptcy?
a) Forever 21
b) H&M
c) Zara
d) Topshop
2. Who has been named ‘India’s most powerful women in media’?
a) Barkha Dutt
b) Sagarika Ghose
c) Shweta Singh
d) Kallie Puri
3. Which Indian wrestler has become world no. 1 in 86kg category?
a) Bajranj Punia
b) Deepak Punia
c) Ravi Dahiya
d) Rahul Aware
4. Who has taken over as the new IAF Chief?
a) Air Marshal RKS Bhadauria
b) Air Marshal RNS Thakur
c) Air Marshal SN Gupta
d) Air Marshal Ratul Shah
5. Which Indian classical musician has been honoured with a planet named
after him?
a) Bhimsen Joshi
b) Lata Mangeshkar
c) Kumar Gandharv
d) Pandit Jasraj
6. Pakistan has invited which Indian leader for Kartarpur Corridor
inauguration?
a) PM Modi
b) Rajnath Singh
c) Sonia Gandhi
d) Manmohan Singh
7. Which American sprinter recently won the gold medal in the World
Athletics Championship by completing 100 meters in 9.76 seconds?
a) Christian Colman
b) Justin Gatlin
c) Andre Di Grass
d) Usain Bolt
8. What is the name of the murderer of former Punjab Chief Minister Beant
Singh, whose death sentence has been converted to life imprisonment?
a) Ajit Singh
b) Rajwant Singh
c) Balwant Singh
d) Rajendra Singh
Answers
1. (a) Forever 21
Forever 21, the US fast-fashion giant, has filed for Chapter 11 bankruptcy
protection and said that it will close its stores in Asia and Europe.
2. (d) Kallie Puri
Kallie Puri was honoured with “India’s Most Powerful Women in Media” award
at the British Parliament. Kallie Puri is currently serving as the Vice-
Chairperson of the India Today Group.
3. (b) Deepak Punia
Deepak Punia has been ranked world number one in 86 kg category in the
latest rankings issued by the International Wrestling Federation. Punia is
also one of the four Indian wrestlers to have secured their place at the 2020
Tokyo Olympic Games.
4. (a) Air Marshal RKS Bhadauria
Air Marshal RKS Bhadauria was appointed as the next chief of IAF by the
Union Government to succeed Air Chief Marshal BS Dhanoa after his
retirement.
5. (d) Pandit Jasraj
The International Astronomical Union (IAU) has named 'Minor Planet' 2006
VP 32 (Number 300128) after Pandit Jasraj. The planet was discovered on
11 November 2006. He is the first Indian artist to receive this honor.
6. (d) Manmohan Singh
Pakistan’s foreign minister Shah Mahmood Qureshi has invited former Indian
PM Manmohan Singh for the inauguration of the Kartarpur corridor on
November 9.
7. (a) Christian Colman
The 23-year-old Christian Coleman took 9.76 seconds, which is the fastest
time of the year and the sixth all-time fastest overall. The world record of
100 meters race is Usain Bolt’s 9.58 seconds. Coleman has won a gold
medal at the World Championships for the first time in his career. Justin
Gatlin finished the race in 9.89 seconds.
8. (c) Balwant Singh
The Ministry of Home Affairs has decided to convert the death sentence of
Balwant Singh Rajoana, convicted in the murder case of former Punjab Chief
Minister Beant Singh, to life imprisonment. Beant Singh was assassinated on
31 August 1995. He was famous for ending terrorism in Punjab.

23 -29 SEP
1. Which film has been nominated for Oscar 2019 as India’s official entry?
a) Gully Boy
b) Raazi
c) Padmavat
d) Andhadhun
2. Who among the following was conferred “Global Goalkeeper" award?
a) Donald Trump
b) Emmanuel Macron
c) Angela Merkel
d) Narendra Modi
3. Who has been selected for the Dadasaheb Phalke award?
a) Amitabh Bachchan
b) Rajinikanth
c) Asha Bhosle
d) Lata Mangeshwar
4. National unity award has been constituted on whose name?
a) Mahatma Gandhi
b) Narendra Modi
c) Sardar Vallabhbhai Patel
d) B R Ambedkar
5. Which city hosted the Howdy Modi event wherein PM Narendra Modi and
US President Donald Trump shared the stage together?
a) Houston
b) Boston
c) Las Vegas
d) New York
6. What was India’s rank in the IMD World Digital Competitiveness Ranking
2019?
a) 45th
b) 44th
c) 48th
d) 47th
7. Which among the following travel giants collapsed recently due to
bankruptcy?
a) Emirates
b) Virgin Atlantic
c) British Airways
d) Thomas Cook
8. Who is the current president of UN General Assembly?
a) Tijjani Muhammad-Bande
b) María Fernanda Espinosa
c) Antonio Guterres
d) Kyriakos Mitsotakis
9. Which country is going to give online tourist visa for the first time?
a) Qatar
b) Jordan
c) Saudi Arabia
d) South Africa
10. Kristalina Georgieva has been selected as the new Chief of which
organization?
a) EU
b) IMF
c) ADB
d) ECB
11. Which Indian athlete was recently awarded with IAAF Veteran Pin Prize?
a) Sania Mirza
b) Vinesh Phogat
c) Mary Kom
d) PT Usha
12. Which Indian was awarded the 'Changemaker Award' by Bill & Milinda
Gates Foundation for bringing change in society?
a) Lalita Malik
b) Suman Awasthi
c) Payal Jangir
d) Deepti Sharma
Answers
1. (a) Gully Boy
'Gully Boy' has been nominated by India for the Oscar Awards to be held in
February 2020. The film is directed by Zoya Akhtar while Ranveer Singh and
Alia Bhatt are the main characters in this film. Gully Boy depicts the story of
a man who becomes a famous rapper coping with his adverse
circumstances.
2. (d) Narendra Modi
Prime Minister Narendra Modi was conferred the "Global Goalkeeper" award
by the Bill and Melinda Gates Foundation on September 24, 2019 for his
leadership in India’s massive Swachh Bharat Abhiyan.
3. (a) Amitabh Bachchan
Amitabh Bachchan has been selected for the highest Indian film honour-
Dadasaheb Phalke award for the year 2019.
4. (c) Sardar Vallabhbhai Patel
The Union Government has instituted Sardar Vallabhbhai Patel award, the
highest civilian award in the field of contribution to the unity and integrity of
India.
5. (a) Houston
PM Narendra Modi and the US President Donald Trump addressed the
'Howdy, Modi’ event in Houston on September 23rd, 2019. Trump is the first
US President to share the stage with Indian Prime Minister to address the
Indian-American Community.
6. (b) 44th
India has risen by four places to be ranked 44th in the IMD World Digital
Competitiveness Ranking 2019 (WDCR). In 2018, India was ranked at the
48th position and its position this year has improved overall in all factors
including future readiness in exploring more digital technologies, knowledge
and technology.
7. (d) Thomas Cook
Thomas Cook, 178-year-old British travel giant has declared bankruptcy,
leaving 150000 British citizens stranded abroad, triggering the largest-ever
peacetime emergency rescue operation in the British history.
8. (a) Tijjani Muhammad-Bande
The current President of UN General Assembly is Tijjani Muhammad-Bande
of Nigeria. Tijjani Muhammad-Bande succeeds María Fernanda Espinosa of
Ecuador, whose term ends in September 2019.
9. (c) Saudi Arabia
The government of Saudi Arabia has decided to offer tourist visas for the
first time. Tourism Visa is one of the main aims of Crown Prince Mohammed
bin Salman’s Vision 2030 reform programme. Crown Prince Mohammed bin
Salman wants to prepare the biggest Arab economy post-oil era.
10. (b)IMF
Bulgaria’s Kristalina Georgieva has been selected as the new chief of
International Monetary Fund (IMF). Kristalina Georgieva has become the
second-ever woman to lead the IMF.
11. (d) PT Usha
India’s former athlete and famous as ‘Payyoli Express’, PT Usha has been
recently awarded with IAAF Veteran Pin Prize. PT Usha shared this
information on her Tweeter handle. PT Usha has also received Arjuna Award
in 1983 and Padma Sri in 1985.
12. (c) Payal Jangir
Payal Jangid of Rajasthan has been awarded the 'Changemaker Award' by
the Bill and Milinda Gates Foundation for bringing change in society. Payal
Jangid was given this honor because of the efforts made by her to stop child
marriages in her area.

1. Which organization has selected Kristalina Georgieva as the new head?


a. EU
b. IMF
c. ADB
d. ECB
2. What is the name of Rajasthan’s 16 year old girl who was awarded the
'Changemaker Award' by Bill & Milinda Gates Foundation for bringing change
in society?
a. Lalita Malik
b. Suman Awasthi
c. Payal Jangid
d. Deepti Sharma
3. Which state government has recently launched emergency number for
police, fire and ambulance emergency services?
a. Delhi
b. Uttar Pradesh
c. Haryana
d. Rajasthan
4. Which among the following vessels was recently commissioned by
Defence Minister Rajnath Singh?
a. Varaha
b. Vamana
c. Matsya
d. Kurma
5. Amitabh Bachchan was selected for which award recently?
a. National Unity Award
b. Swachh Bharat Award
c. National Tourism Award
d. Dada Saheb Phalke Award
6. What is the name of India’s first corporate-run train that is scheduled to
run between Lucknow and Delhi?
a. Tejas Express
b. Vande Bharat Express
c. Majesty Express
d. Humsafar Express
7. Who has given this statement in UN climate summit 2019, ““How dare
you? You have stolen my dreams, my childhood…..”
a. Sonita Alizadeh
b. Bana Alabed
c. Greta Thunberg
d. Malala Yousafzai
8. Which Indian state/UT has issued highest numbers of Golden Cards
under Ayushman Bharat scheme?
a. Delhi
b. Jammu & Kashmir
c. Uttar Pradesh
d. Tamil Nadu
9. Which Indian player won silver medal in the World Boxing Championship
2019?
a. Vijay Sardana
b. Anuj Chugh
c. Sarvesh Dayal
d. Amit Panghal
10. Name of the world’s biggest travel company which was recently
collapsed?
a. Thomas Cook
b. Emirates
c. Virgin Atlantic
d. British Airways
11. On which date International Day of Sign Languages observed across the
world every year?
a. 21st September
b. 22nd September
c. 23rd September
d. 24th September
12. What is the name of scientist who recently won National Geoscience
Young Scientist Award-2018?
a. Dr. Sohini Ganguly
b. Dr. Yamuna Singh
c. Dr. Debjani Sarkar
d. Mr. Pradipta Sur
13. Who among the following was recently appointed as the new National
Security Adviser of the United States?
a. Jay Powell
b. Robert O’Brien
c. Terrence K. Williams
d. Jim Jordan
14. In which city Google Research India AI lab will be established?
a. New Delhi
b. Hyderabad
c. Bengaluru
d. Chennai
15. Name of the Indian documentary which has been nominated for Oscars.
a. Roar of the Lion
b. Lady Tiger
c. Wild Karnataka
d. Moti Bagh
16. Which Indian player who became the first Indian wrestler to qualify for
the Tokyo Olympics 2020?
a. Yogeshwar Dutt
b. Vinesh Phogat
c. Sushil Kumar
d. Sakshi Malik
17. Which Indigenous missile was launched recently by DRDO from Su-30
MKI fighter jet?
a. NAG
b. Astra
c. Exocet
d. Dhanush
18. Which Indian state has launched helpline number 1100 under
Mukhyamantri Seva Sankalp?
a. Rajasthan
b. Haryana
c. Punjab
d. Himachal Pradesh
19. Which State/UT will start Odd-Even scheme from November 4 to 15?
a. Haryana
b. Uttar Pradesh
c. Delhi
d. Gujarat
20. Which city PM Narendra Modi inaugurated India’s second Multi-Modal
Terminal (MMT)
a. Ranchi
b. Joshimath
c. Cuttack
d. Sahibganj
Answers
1. (b) IMF
Kristalina Georgieva has become the first IMF chief from emerging economy.
In 2010, she was named ‘European of the Year’ and ‘Commissioner of the
Year’ by European Voice.
2. (c) Payal Jangid
Payal Jangid of Alwar district of Rajasthan, received the Changemaker Award
by the Bill & Melinda Gates Foundation in New York for raising the voice
against child marriage and child labour.
3. (a) Delhi
Delhi Govt launched single emergency number ‘112’ for Police, Fire,
Ambulance and other services. Government announced that even if someone
dials 100 or 101 or 102, the call will connect to 112.
4. (a) Varaha
Defence Minister Rajnath Singh formally launched Coast Guard coastal patrol
vessel 'Varah' at Chennai Port Trust. After unveiling a commemorative
plaque to commemorate the launch ceremony, Singh said that this state-of-
the-art ship will further enhance the strength of the Indian Coast Guard.
5. (d) Dadasaheb Phalke Award
The Dadasaheb Phalke Award was announced to Amitabh Bachchan. This
award will be given to him for his invaluable contribution to the Indian
cinema.
6. (a) Tejas Express
IRCTC has launched the first corporate train Tejas Express between New
Delhi and Lucknow and Vande Bharat Express between New Delhi and
Varanasi.
7. (c) Greta Thunberg
Greta Thunberg’s address to the UN’s Climate Action Summit and said that
no international agency has done anything to save climate. She said that
you are stealing future from the children.
8. (b) Jammu & Kashmir
J&K is the first Indian state that issued most numbers of Golden Cards under
Ayushman Bharat Scheme. 55 Hospitals including 126 public and 29 private
hospitals are empanelled under this scheme in the state.
9. (d) Amit Panghal
Born in Rohtak district of Haryana, Amit became the first Indian athlete to
won silver medal in this biggest boxing event. In the final, Amit Panghal
suffered a 5–0 defeat at the hands of Shakhobidin Joirov of Uzbekistan.
10. (a) Thomas Cook
Thomaskook, UK-based company that provides travel services, finally went
bankrupt with a debt burden. Around six lakh tourists with its bookings got
stuck around the world.
11. (c) 23rd September
International Day of Sign Languages observed everyear on 23rd September.
A UN Report said that about 72 million deaf people are present worldwide.
12. (a) Dr. Sohini Ganguly
Dr. Sohini Ganguly from Goa University was awarded the Young Scientist
Award-2018. She has done significant work in the fields of volcanology,
geochemistry and petrology.
13. (b) Robert O’Brien
US President Donald Trump has named Robert O’Brien as new national
securit adviser. He has a long career in foreign policy and International
relations.
14. (c) Bengaluru
It was announced Google for India event that Google Research India will
establish an AI lab in Bengaluru. Google Research India will be led by Manish
Gupta, a renowned computer scientist.
15. (d) Moti Bagh
An Indian documentary Moti Bagh has been nominated for Oscars. It is
about struggles of a farmer. Director of the film is Nirmal Chander
Dandriyal.
16. (b) Vinesh Phogat
Vinesh Phogat secured her position for 2020 Tokyo Olympics. Ravi Kumar
also won bronze in the 57kg category while Bajrang Punia claimed the
bronze medal in the 65kg category.
17. (b) Astra
As per the official statement released by Indian Air Force (IAF) the state-of-
the-art missile was launched from Sukhoi-30 MKI as a part of user trials by
the Indian Air Force.
18. (d) Himachal Pradesh
A toll-free helpline number 1100 was recently launched by Himachal Pradesh
government. It has been launched with an aim to provide services to those
people who are living in remote areas of the state.
19. (c) Delhi
The Chief Minister of Delhi Arvind Kejriwal announced that Odd-Even scheme
will be applied between 4 to 15 November. The objective of this scheme is to
curb the pollution from Delhi.
20. (d) Sahibganj
India’s second Multi-Modal Cargo Terminal has been inaugurated at
Sahibganj by PM Narendra Modi. It will be helpful in transportation of
domestic coal from local mines to other thermal power plants.

16-22 SEP
1. PM Narendra Modi addressed the Howdy Modi event along with US
President Donald Trump in which city?
a) New York
b) Houston
c) Las Vegas
d) Boston
2. What is the new income tax rate prescribed for the new domestic
manufacturing companies?
a) 22.5
b) 22
c) 15
d) 20
3. When is the International Day of Peace observed every year globally?
a) 21st September
b) 22nd September
c) 23rd September
d) 24th September
4. Who won the Best Actress Award at IIFA Awards 2019?
a) Sara Ali Khan
b) Katrina Kaif
c) Alia Bhatt
d) Deepika Padukone
5. Which State launched the ‘Tatpar’ mobile app?
a) Delhi
b) Rajasthan
c) Chandigarh
d) Uttar Pradesh
6. Which of the following Indian documentaries has been nominated for the
Oscars?
a) Roar of the Lion
b) Wild Karnataka
c) Moti Bagh
d) Lady Tiger
7. EPFO would be crediting how much interest for the year 2018-19 in 6
crore accounts?
a) 8.65%
b) 8.57%
c) 8.60%
d) 8.63%
8. Who issued the following statement-“PoK is a part of India”?
a) Rajnath Singh
b) Amit Shah
c) S Jaishankar
d) Narendra Modi
9. Which country announced to press for dropping Sudan off the US blacklist
of state sponsors of terrorism?
a) Germany
b) France
c) United Kingdom
d) India
10. Who was appointed as the first Lokayukta of Andhra Pradesh?
a) Justice A Goswami
b) Justice Ram Rao
c) Justice P Lakshman Reddy
d) Justice S D Manirattnam
Answers
1. (b) Houston
PM Narendra Modi addressed the 'Howdy, Modi! Shared Dreams, Bright
Futures’ event in Houston along with the US President Donald Trump. Trump
is the first US President to attend an Indian-American Community event.
2. (c) 15
To attract more investment in manufacturing sector, Finance Minister
Nirmala Sitharaman announced a new provision in the Income Tax Act, 1961
that will allow any new domestic company making fresh investment in
manufacturing to pay income tax of 15 percent.
3. (a) 21st September
The International Day of Peace is observed globally every year on 21st
September with an aim to promote peace globally. The 2019 theme of the
International Day of Peace is "Climate Action for Peace".
4. (c) Alia Bhatt
Alia Bhatt has won Best Actress Award in IIFA Awards 2019 for film Raazi.
This film was based on a story of an Indian female spy. Vicky Kaushal was
male supporting actor in this movie. It was directed by Meghna Gulzar.
5. (a) Delhi
Delhi Police recently launched ‘Tatpar’ mobile application offering over 50
services. The app offers services launched by Delhi Police such as e-FIR, lost
and found, Himmat Plus etc.
6. (c) Moti Bagh
Moti Bagh documentary has been nominated for the Oscars. It is based on
the life of an Uttarakhand farmer Vidyadutt and has been directed by Nirmal
Chander.
7. (a) 8.65%
Employees' Provident Fund Organisation would credit 8.65 per cent of
interest to over 6 crore subscribers for 2018-19. The announcement
regarding this was made by Labour Minister Santosh Gangwar.
8. (c) S Jaishankar
Union External Affairs Minister, S Jaishankar recently stated that the PoK is
and has always been a part of India and also said that India expects to have
physical jurisdiction over the area one day.
9. (b) France
France announced to press for dropping Sudan off the US blacklist of state
sponsors of terrorism. The announcement was made by the French Foreign
Minister Jean-Yves Le Drian.
10. (c) Justice P Lakshman Reddy
Justice P Lakshman Reddy was sworn in as the first Lokayukta of the Andhra
Pradesh for the next five years. He is the first Lokayukta to be assuming
charge since the state bifurcated in 2014.

9-14SEP
1. Which of the following personalities has been bestowed with the 2019
Ramon Magsaysay Award?
a) Ravish Kumar
b) Kiran Bedi
c) Arvind Kejriwal
d) Arnab Goswami
2. Which among the following Union Territories will have a common high
court?
a) Ladakh and Chandigarh
b) Jammu & Kashmir and Daman and Diu
c) Jammu & Kashmir and Ladakh
d) Jammu & Kashmir and Himachal Pradesh
3. Ram Jethmalani, one of India’s most renowned and respected criminal
lawyers, passed away at the age of 95 on September 8, 2019. Jethmalani
had not fought in which among the following cases?
a) Ajmal Kasab death penalty case
b) Indira Gandhi Assassination case
c) Rajiv Gandhi assassination case
d) Jessica Lall case
4. PM has inaugurated South Asia’s first cross-border oil pipeline. The
pipeline will connect India with which nation?
a) Nepal
b) Malaysia
c) Bhutan
d) Bangladesh
5. Which Committee has been constituted by the Union Government to
oversee the bifurcation of Jammu and Kashmir (J&K) into Union Territories?
a) Sanjay Mitra Committee
b) Rajnath Singh Committee
c) Amit Shah Committee
d) Ajay Kumar Committee
6. Which nation’s president announced recently that he has cancelled his
secret meeting with the Taliban leaders and Afghanistan President?
a) Russia
b) United States
c) Pakistan
d) France
7. How many times has Spain's star tennis player Rafael Nadal won the US
Open title?
a) 17
b) 18
c) 19
d) 20
8. Water has been discovered on which potentially habitable super-Earth?
a) K2-18d
b) K2-18c
c) K2-18a
d) K2-18b
9. Which of the following countries became the 79th member of the
International Solar Alliance?
a) Cuba
b) St. Vincent and Grenadines
c) St. Lucia
d) Dominican Republic
10. Which country has won the ‘Best Video’ award for the Asia & Pacific
Region in the UNWTO Tourism Video Competition 2019?
a) Malaysia
b) Sri Lanka
c) Thailand
d) India
Answers
1. (a) Ravish Kumar
Indian journalist Ravish Kumar has been honoured with the prestigious 2019
Ramon Magsaysay Award. The four other winners of the Award are Ko Swe
Win from Myanmar, Raymundo Pujante Cayabyab from the Philippines,
Angkhana Neelapaijit from Thailand, and Kim Jong-Ki from South Korea.
2. (c) Jammu & Kashmir and Ladakh
The two Union Territories- Jammu and Kashmir and Ladakh will have a
common high court, Jammu and Kashmir High Court. Among the old laws,
164 state laws would be repealed and 166 state laws will continue to be
applicable and 108 Central Laws would be made applicable for the two Union
Territories.
3. (a) Ajmal Kasab death penalty case
Ram Jethmalani, senior Supreme Court lawyer passed away on September
8, 2019. He was 95. Jethmalani, a criminal lawyer par excellence, fought
several high-profile cases such as Indira Gandhi and Rajib Gandhi’s
assassination case, the Jessica Lall case, Jayalalitha disproportionate assets
case and Arun Jaitely defamation case against Arvind Kejriwal among
others.
4. (a) Nepal
Prime Minister Narendra Modi jointly inaugurated South Asia’s first cross-
border pipeline to enhance the energy security of Nepal. The India-Nepal
cross-border petroleum pipeline is South Asia’s first cross-border pipeline.
The project will ensure a reduction in fuel prices by substantially cutting
down on transit costs.
5. (a) Sanjay Mitra Committee
The Union government recently constituted a 3-member committee chaired
by former defence secretary Sanjay Mitra to oversee the bifurcation of
Jammu and Kashmir (J&K) into two Union Territories. The other Members of
the Committee are Arun Goyal, retired IAS officer and Giriraj Prasad Gupta,
retired Indian Civil Accounts Service officer. The Committee will look into
distribution of assets and liabilities of J&K between two UTs.
6. (b) United States
US President Donald Trump revealed on September 8, 2019 that he has
called off his planned secret meeting with top Taliban leaders after they
admitted to an attack in Kabul that killed a US soldier and 11 others.
7. (c ) 19
Rafael Nadal won his 19th Grand Slam title by winning the US Open final,
defeating Daniil Medvedev 7-5, 6-3, 5-7, 4-6, 6-4 to seize his fourth crown
in New York. Spain’s star tennis player Rafael Nadal (33) is one behind of
Roger Federer’s 20 Grand Slam triumphs.
8. (d) K2-18b
Water has been detected by scientists in the atmosphere of a potentially
habitable super-Earth. The exoplanet, named K2-18b, is twice the size and
eight times the mass of Earth. K2-18b is the first planet outside the solar
system in the habitable zone of its star known to have water in its
atmosphere.
9. (b) St. Vincent and Grenadines
Saint Vincent and Grenadines became the 79th country to join the
International Solar Alliance (ISA). It is an initiative by India aimed at
combating the threats of climate change and global warming. The
International Solar Alliance is a group of 121 solar resource-rich countries
with headquarters in Gurugram, India.
10. (d) India
India’s ‘Yogi of the Racetrack’ has won ‘Best Video’ award for the Asia &
Pacific Region in the UNWTO Tourism Video Competition 2019. Yogi of the
Racetrack is a short promotional film on Yoga, which was produced by the
Union Ministry of Tourism.

2-8 SEP
1. Robert Mugabe passed away at the age of 95 on September 6, 2019. He
served as the President of which nation for almost four decades?
a) Ghana
b) Libya
c) Zimbabwe
d) South Africa
2. Which place won the Best Swachh Iconic Place award in Swachh Bharat
Mission Awards?
a) Vaishno Devi
b) Golden Temple
c) Shirdi
d) Lotus Temple
3. Which among the following Ministries won the Swachh Bharat award in
Swachhta Action Plan category?
a) Ministry of Women and Child
b) Ministry of Urban Development
c) Ministry of Defence
d) Ministry of Railways
4. Which city is hosting the 28th edition of India-Thailand Coordinated Patrol
exercise?
a) Vishakhapatnam
b) Bangkok
c) Phuket
d) Brahmapur
5. When is the International Literacy Day observed every year?
a) 6th September
b) 7th September
c) 8th September
d) 9th September
6. Which country recently established a space command SpaceCom
dedicated to space war?
a) China
b) Russia
c) USA
d) India
7. Which of the following countries is hosting the fifth meeting of the Eastern
Economic Forum?
a) India
b) Bhutan
c) Indonesia
d) Russia
8. Which political leader will be bestowed with an award from Bill and
Melinda Gates Foundation for Swachh Bharat Abhiyaan?
a) Narendra Modi
b) Smriti Irani
c) Dr Harsh Vardhan
d) Rajnath Singh
9. Who assumed the charge of Chairmanship of Association of World Election
Bodies recently?
a) Sunil Arora
b) Sushil Chandra
c) Umesh Sinha
d) Amit Tyagi
10. Exercise Yudh Abhyas 2019 began recently between India and which
country?
a) Singapore
b) Russia
c) USA
d) Indonesia
Answers
1. (c) Zimbabwe
Robert Mugabe, former Zimbabwe President passed away at the age of 95
on September 6, 2019. Robert Mugabe had ruled Zimbabwe for almost four
decades before he was ousted from power in a military coup two years ago.
2. (a) Vaishno Devi
Vaishno Devi Shrine Board in Jammu and Kashmir received the Best Swachh
Iconic Place award. The Shrine was selected on the basis of the overall
improvement in sanitation.
3. (d) Ministry of Railways
Ministry of Railways won the Swachh Bharat award in the category of
Swachhta Action Plan. The award was conferred by President Ram Nath
Kovind in a ceremony in New Delhi on September 6, 2019.
4. (b) Bangkok
The 28th edition of India-Thailand Coordinated Patrol is being conducted
between the Indian Navy (IN) and the Royal Thai Navy (RTN) during 5 – 15
September 2019 in Bangkok, Thailand. For this purpose, IN Dornier arrived
at Bangkok along with aircrew.
5. (c) 8th September
International Literacy Day is celebrated across the world on September 8
every year to highlight improvements in world literacy rates. The Day aims
to mobilise public opinion in favour of struggle against illiteracy.
6. (c) USA
US President Trump authorized the creation of the United States Space
Command, citing the need for a centralized unit to protect American
interests in what he called “the next war-fighting domain.” The Space Force
will organize, train and equip warriors to support Spacecom’s mission.
7. (d) Russia
Prime Minister Narendra Modi will leave for Russia today to participate in
three-day Eastern Economic Forum at Vladivostok. It is fifth edition of
Eastern Economic Forum (EEF). EEF was established by the order of the
President of the Russian Federation in the year 2015.
8. (a) Narendra Modi
Prime Minister Narendra Modi will be honoured with the award from Bill and
Melinda Gates Foundation for Swachh Bharat Abhiyaan.PM Modi will be
conferred with the award during his visit to the United States later in
September 2019.
9. (a) Sunil Arora
Chief Election Commissioner (CEC) Sunil Arora assumed the Chairmanship of
‘Association of World Election Bodies’ (AWEB) on September 3, 2019, Arora
will serve the post during 2019-21. His appointment came after India took
over the Chair of AWEB from Romania.
10. (c) USA
Exercise Yudh Abhyas 2019, a joint military training exercise, will begin on
September 5, 2019 between India and the United States in Washington,
USA. The exercise will conclude on September 18, 2019.

26AUG-1SEP
1. Which three banks will be merged into a single entity to form the 2nd
largest bank of India?
a) PNB, OBC & United Bank of India
b) Indian Bank, OBC, Syndicate Bank
c) Allahabad Bank, Canara Bank & PNB
d) PNB, Corporation Bank & Bank of India
2. Which nation has launched its own Space Command?
a) Russia
b) China
c) United States
d) Israel
3. Who among the following cricketers has come out of his retirement and
made himself available for all formats?
a) Shoaib Malik
b) Yuvraj Singh
c) Ambati Rayudu
d) Hashim Amla
4. Which telecom company topped the telecom market with the highest
revenue share in the April-June quarter?
a) Airtel
b) Jio
c) BSNL
d) Vodafone Idea
5. The Indian Railways has decided to offer how much discount in its AC
Chair car trains like Shatabdi, Tejas?
a) 30 percent
b) 25 percent
c) 15 percent
d) 20 percent
6. Who has become the first IPS officer to be honoured with the prestigious
Tenzing Norgay National Adventure Award 2018?
a) Manikandan K
b) Prabhat Raju Koli
c) Wangchuk Sherpa
d) Aparna Kumar
7. When is the National Sports Day observed every year in India?
a) 27th August
b) 28th August
c) 29th August
d) 30th August
8. Andhra Pradesh government recently announced that the Polavaram
Project work will be resumed from November 2019. The project runs on
which river?
a)Krishna
b)Godavari
c)Sabari
d)Penna
9. Which State Government recently announced that it will waive water
arrears of people who live in colonies categorised as E, F, G and H?
a)Haryana
b)Rajasthan
c)Delhi
d)Uttar Pradesh
10. Which country recently requested the United Nations to call back all
peacekeepers from the country by June 2020?
a)Algeria
b)Sudan
c)Turkey
d)Myanmar

Answers
1. (a) PNB, OBC & United Bank of India
Finance Minister Nirmala Sitharaman on August 30, 2019 announced the
merger of several banks under the Bank Consolidation plan. Punjab National
Bank, Oriental Bank of Commerce, United Bank of India will be merged as
one single entity. Punjab National Bank will be the anchor bank.
2. (c) United States
US President Donald Trump launched the US Space Command on August 29,
2019. Trump announced the establishment of the space command as the
newest US military combatant command in a ceremony at the Rose Garden,
White House.
3. (c) Ambati Rayudu
Ambati Rayudu, who had announced his retirement on July 3, made a u-turn
and said that he would like to come out of retirement on August 30, 2019.
Ambati Rayudu wrote a letter to Hyderabad Cricket Association (HCA), in
which he said, "I would like to bring to your notice that I would like to come
out of retirement and play cricket in all formats.”
4. (b) Jio
Reliance Jio has topped the telecom market with a 31.7 percent market
share in the second quarter (April-June) of 2019. Bharti Airtel has regained
its second spot, leaving behind Vodafone Idea, the revenue of which plunged
substantially.
5. (b) 25 percent
Indian Railways is set to offer up to 25 percent discount on trains such as
Shatabdi Express, Tejas and Gatimaan Express, as per a senior official. The
move is a result of the stiff competition faced by the Indian railways from
low-cost airlines.
6. (d) Aparna Kumar
IPS Officer Aparna Kumar will be honoured with the Tenzing Norgay National
Adventure Award 2018 in the ‘Land adventure’ category. Aparna Kumar is a
2002 batch IPS officer from the Uttar Pradesh Cadre. She is the first IPS
officer to be honoured with the prestigious Tenzing Norgay Award.
7. (C) 29th August
National Sports Day is observed every year on 29 August to mark the birth
anniversary of Major Dhyan Chand, the legendary hockey player who was
born on August 29, 1905. Dhyan Chand was one of the greatest field hockey
players of all time.
8. (b) Godavari
Andhra Pradesh government announced that the construction works relating
to the Polavaram project will taken up again from November 2019, after the
rainy season. Under the Polavaram project, a dam is being constructed
across the Godavari River in West & East Godavari Districts of Andhra
Pradesh.
9. (c) Delhi
Delhi Chief Minister Arvind Kejriwal announced to waive water arrears of
consumers who live in E, F, G and H colonies and have functional domestic
water meters. In Delhi, colonies are categorized from A to H.
10. (b) Sudan
Sudan requested the United Nations Security Council to lift its suspension of
troop withdrawals and ensure all peacekeepers leave Darfur by June 2020.
Omer Mohamed Siddig, Sudan’s U.N. Ambassador, informed the council that
it’s time to shift from peacekeeping to peace-building in Darfur.

19-25 AUG
1. Who among the following has been nominated for the Rajiv Gandhi Khel
Ratna in the National Sports Awards 2019?
a) Ravindra Jadeja
b) R. Ashwin
c) Deepa Malik
d) Dipa Karmakar
2. A raging fire has engulfed this rainforest, which contributes to about 20
percent of the earth’s oxygen.
a) Amazon
b) Nile
c) Congo
d) New Guinea
3. Who among the following was arrested by CBI in relation to the INX
Media corruption case?
a) Yashwant Sinha
b) P. Chidambaram
c) Arun Jaitley
d) Pranab Mukharjee
4. The Direct Tax Code (DTC) task force has recommended a major cut in
corporate tax for both domestic and foreign companies. What is the newly
recommended corporate tax rate?
a) 25 percent
b) 30 percent
c) 15 percent
d) 20 percent
5. Which glacier in Iceland has lost its identity due to climate change?
a) Okjokull
b) Sesami
c) Milami
d) Sumatra
6. Which country is the largest emitter of SO2, according to Greenpeace
data?
a) Russia
b) India
c) China
d) Saudi Arabia
7. Which country bestowed its highest civilian award 'Order of Zaid’ upon
Indian Prime Minister Narendra Modi?
a) Bhutan
b) Thailand
c) UAE
d) Japan
8. Which among the following Indian structures featured in Time’s 2019 list
of 100 greatest places in world?
a) The Light Box Restroom
b) Nalanda Ruins
c) Galden Namgey Lhatse
d) Statue of Unity
9. US President Donald Trump recently showed interest in buying which
nation?
a) Iceland
b) Ireland
c) Cuba
d) Greenland
10. Which court has agreed to examine validity of new triple talaq law?
a) Delhi HC
b) Madras HC
c) Supreme Court
d) Allahabad HC
11. What will be the retirement age of the Central Armed Police Force
personnel as per the order issued by the Ministry of Home Affairs?
a) 58
b) 59
c) 60
d) 61
12. PM Modi recently inaugurated Mangdechhu hydroelectric power plant in
which country?
a) Bangladesh
b) Bhutan
c) Sri Lanka
d) Malaysia
Answers
1. (c) Deepa Malik
A day after wrestler Bajrang Punia's name was recommended for the Rajiv
Gandhi Khel Ratna, the 12-member selection committee added 2016 Rio
Paralympics silver medallist Deepa Malik to this prestigious list. Malik is the
second Indian Paralympian after Devendra Jhajharia (2017), and the first
woman in the category, to be nominated for this coveted award.
2. (a) Amazon
Amazon rainforest, the world’s largest rain forest, which provides almost 20
percent of the earth’s oxygen, has been burning for over 16 days resulting in
a major loss of trees and biodiversity.
3. (b) P. Chidambaram
Former finance minister and Congress leader P Chidambaram was arrested
by the Central Bureau of Investigation (CBI). P Chidambaram is accused of
facilitating foreign investment into a television company, INX Media, in 2007,
when he was the Finance Minister.
4. (a) 25 percent
The Direct Tax Code (DTC) task force has recommended a major cut in the
corporate tax for both domestic and foreign companies. The panel has
proposed reducing the tax from earlier 30 percent for large companies and
40 percent for foreign companies to an even rate of 25 percent for both
domestic and foreign companies.
5. (a) Okjokull
Iceland unveiled a plaque to its Okjokull ice sheet, the first of the country’s
hundreds of glaciers to melt away due to climate change. Scientists see the
shrinking of glaciers as one of many warning signs that the earth’s climate is
lurching toward dangerous tipping points.
6. (b) India
India is the largest emitter of anthropogenic sulphur dioxide in the world, as
per the data released by environmental NGO Greenpeace on August 19,
2019. Anthropogenic sulphur dioxide is produced from burning of coal and it
is known to largely contribute to air pollution.
7. (c) UAE
Prime Minister Narendra Modi was conferred with the Order of Zayed, UAE’s
highest civilian award by UAE Crown Prince, Mohamed bin Zayed Al Nahyan
on August 24, 2019. The gulf nation decided to honour the PM with its
highest civilian award to recognise his "pivotal role" in strengthening ties
between the two nations.
8. (d) Statue of Unity
India’s 'Statue of Unity' and Soho House in Mumbai have been featured in
the Time’s list of 100 greatest places in the world. The Time magazine in its
annual list of 2019, compiled a list of 100 greatest places in the world that
are new and noteworthy destinations to experience.
9. (d) Greenland
In a surprising move, US President Donald Trump expressed desire to buy
Greenland. The Prime Minister of Denmark Mette Frederiksen, who found the
notion to be absurd, declined the offer. Trump, in fact, cancelled his
scheduled visit to Denmark in September after the refusal.
10. (c) Supreme Court
The Supreme Court has agreed to hear a plea by a Muslim body challenging
the new triple talaq law. The Jamiat Ulama-i-Hind moved the apex court on
August 23, 2019 questioning the validity of the new triple talaq law.
11. (c) 60
As per the order issued by Union Home Ministry, all personnel of the forces,
the Border Security Force, Sashastra Seema Bal, Central Reserve Police
Force and Indo-Tibetan Border Police will retire at the age of 60, instead of
57 years.
12. (b) Bhutan
Mangdechhu hydroelectric power plant in Bhutan was recently inaugurated
by Prime Minister of India Narendra Modi. It is a one of the major projects
under Bhutan's initiative to generate 10,000 MW hydropower by 2020.

12-18AUG
1. Prime Minister Narendra Modi announced the creation of the post of Chief
in which sector in his Independence Day speech 2019?
a) Finance
b) Defence
c) Railways
d) Infrastructure
2. Which police department was dragged to court for referring to
Independence Day as Republic Day?
a) UP Police
b) Delhi Police
c) MP Police
d) Karnataka Police
3. Donald Trump accused which two countries of taking advantage of the
WTO’s ‘developing economies’ tag?
a) China and Japan
b) India and Pakistan
c) Pakistan and China
d) India and China
4. Which batsman has become first cricketer to score 20,000 international
runs in a decade?
a) Virat Kohli
b) Rohit Sharma
c) Babar Azam
d) Francois du Plessis
5. What is the name of scheme PM Modi recently announced for water
conservation and household pipeline supply?
a) Pradhanmantri Jal Abhiyan
b) Jal Shakti Mission
c) Jal Jeevan Mission
d) Jal hai toh kal hai
6. Which court refused to pass any order on J&K saying that it will wait for
normalcy to return to the state?
a) Delhi HC
b) Madras HC
c) Supreme Court
d) Allahabad HC
7. Who has become the first Indian to win world title in motorsports?
a) Aishwarya Pissay
b) Mehul Sahney
c) Mrinal Pandey
d) Rita Vieira
8. Who won the National Film Award 2019 for Best Actress?
a) Alia Bhatt
b) Radhika Apte
c) Tabu
d) Keerthy Suresh
9. Which film won the national film award 2019 for “Best Film”?
a) Uri
b) Padman
c) Hellaro
d) Andhadhun
10. What is the name of the father of ISRO, on whose 100th birthday,
Google has dedicated a doodle?
a) Homi Bhabha
b) Ramesha Chandra
c) Vikram Sarabhai
d) Anil Vijayan
Answers

1. (b) Defence
Prime Minister Narendra Modi announced the appointment of Chief of
Defence Staff (CDS) in his sixth consecutive Independence Day speech. The
Chief of Defence Staff will help integrate all three services – the Indian
Army, the Air Force and the Indian Navy.
2. (b) Delhi Police
Delhi Police reportedly referred to August 15 as India’s Republic Day instead
of Independence Day. This blunder got printed in an advisory. Following the
incident, a petition was filed against Delhi Police in the Delhi High Court on
August 13.
3. (d) India and China
US President Donald Trump stated recently that India and China are no
longer “developing nations” and that both were taking advantage of the
developing nations tag given by the World Trade Organization.
4. (a) Virat Kohli
Virat Kohli has become the first batsman to score 20,000 international runs
in a decade during the match against West Indies. He has scored 20,502
runs across three formats out of which he has scored 20,018 runs in the
ongoing decade.
5. (c) Jal Jeevan Mission
Prime Minister Narendra Modi announced during his Independence Day
speech that the government will launch a Jal Jeevan Mission to bring piped
water supply to every house. He also urged the people to come forward and
contribute in conserving water resources.
6. (c) Supreme Court
The Supreme Court has refused an urgent hearing of a PIL on the security
clampdown in Kashmir Valley. The petition was filed by activist Tehseen
Poonawalla challenging the Centre's decision to impose restrictions in Jammu
and Kashmir, following revoking of Article 370.
7. (a) Aishwarya Pissay
Aishwarya Pissay won the FIM World Cup in women’s category in Hungary,
becoming the first-ever Indian to win a world title in motorsports. Aishwarya
Pissay also finished second in the FIM Junior category.
8. (d) Keerthy Suresh
Keerthy Suresh won the Best Actress award at the 66th National Film
Awards for her performance in Telugu film Mahanati.
9. (c) Hellaro
Gujarati Movie Hellaro won the ‘Best Film’ award at the 66th national film
awards 2019.
10. (c) Vikram Sarabhai
Google has paid its tribute to the founder of ISRO Vikram Sarabhai by a
Google Doodle. Google is celebrating the 100th birth anniversary of award-
winning Indian physicist, industrialist, and innovator Vikram Sarabhai. Dr.
Vikram Sarabhai is also known as the father of Indian space programme.

5-11 AUG
1. Which Article was scrapped by the Parliament recently?
a) Article 370
b) Article 356
c) Article 377
d) Article 246
2. Which of the following Articles got revoked alongside Article 370?
a) Article 174
b) Article 244
c) Article 153
d) Article 35A
3. Name of the former External Affairs Minister and first women chief
minister of Delhi who passed away recently.
a. Sonia Gandhi
b. Rekha Mahajan
c. Sushma Swaraj
d. Sheila Dikshit
4. Which of the following would be the new Union Territory without
Legislature?
a) Puducherry
b) Chandigarh
c) Ladakh
d) Jammu
5. Who won the Best Director award at the 66th National Film Awards?
a) Sriram Raghavan
b) Aditya Dhar
c) Amit Sharma
d) Sanjay Leela Bhansali
6. Pathamadai Silk Mat has recently granted a GI tag. It belongs to which
state?
a. Kerala
b. Tamil Nadu
c. Telangana
d. Odisha
7. How many individuals will be scanned under the Human Genome Mapping
Project of Government of 7. India?
a) 12,000
b) 16,000
c) 20,000
d) 22,000
8. Who will be awarded with Vir Chakra in 2019?
a) Wing Commander Abhinandan Vardhman
b) Captain Saurabh Verma
c) Wing Commander JS Verma
d) Captain Suresh Tiwari
9. Which organization has recently launched Jeevan Amar Plan?
a. SBI
b. ICICI
c. RBI
d. LIC
10. Who among the following was conferred the Bharat Ratna honour?
a) Ram Nath Kovind
b) Pranab Mukherjee
c) Arun Jaitely
d) Meira Kumar
Answers
1. (a) Article 370
Article 370 was scrapped by the Parliament on August 5, 2019, after Union
Home Minister tabled the historic amendment in the Parliament. All the
clauses of Article 370 were revoked, except Clause 1 which states that J&K is
a part of India.
2. (d) Article 35A
With the revocation of Article 370, Article 35A gets revoked as well. Article
35A defines "permanent residents" of Jammu & Kashmir and grants special
rights to them.
3. (c) Sushma Swaraj
Sushma Swaraj, former external affairs minister and Delhi’s first female
chief minister, passed away on August 6, 2019. She was 67. Sushma Swaraj
was taken to the AIIMS soon after she suffered from a heart attack.
4. (c) Ladakh
The Jammu and Kashmir Reorganization Bill, 2019 was presented by Union
Home Minister Amit Shah in Rajya Sabha on August 5, 2019. The Bill calls
for forming two Union Territories - Jammu and Kashmir with Legislature and
Ladakh without Legislature.
5. (b) Aditya Dhar
Uri Director Aditya Dhar won the Best Director award and Ayushmann
Khurrana and Vicky Kaushal shared the Best Actor national film award.
6. (b) Tamil Nadu
The traditional art of weaving pattamadai silk mat belongs to the Tirunelveli
district of Tamil Nadu. Pathamadai mats have been granted Geographical
Indication (GI) status by the trademarks and GI authorities.
7. (c) 20,000
India is planning to launch its first Human Genome Mapping Project. The
project involves scanning of 20,000 Indian genomes (in the next five years)
to develop diagnostic tests and effective therapies for treating diseases such
as cancer.
8. (a) Wing Commander Abhinadan Vardhman
Abhinandan Varthaman, Indian Air Force pilot, is likely to be awarded a Vir
Chakra for his gallantry. Five Mirage-2000 fighter jet pilots who dropped
bombs on the terror facility of the Jaish-e-Mohammed will also get the Vayu
Sena Medal for gallantry.
9. (d) LIC
The Life Insurance Corporation of India (LIC) has launched a new and
affordable term insurance plan named the ‘LIC Jeevan Amar Plan’. This plan
provides greater flexibility to the policyholders as it allows them to choose
two death benefit options of their own choice. These two options are - level
sum assured and increasing sum assured.
10. (b) Pranab Mukherjee
President Ram Nath Kovind conferred the Bharat Ratna to former President
Pranab Mukherjee and posthumously to Assamese singer Bhupen Hazarika
and Bharatiya Jana Sangh leader Nanaji Deshmukh.

29JULY-4AUG
1. The Supreme Court has order transfer of all 5 Unnao cases to which city’s
court?
a) Delhi
b) Chandigarh
c) Lucknow
d) Pune
2. Who among the following was expelled by BJP for being the main accused
in the Unnao case?
a) Sanjay Singh
b) Kuldeep Singh Sengar
c) BC Patil
d) Mahesh Kumutali
3. RN Ravi was recently sworn in as the Governor of which state?
a) Meghalaya
b) Manipur
c) Mizoram
d) Nagaland
4. Which among the following states does not fall under the one nation one
ration card pilot project?
a) Karnataka
b) Maharashtra
c) Telangana
d) Andhra Pradesh
5. Delhi Government recently announced to provide free electricity to
households consuming up electricity up to what power?
a) 100 units
b) 150 units
c) 200 units
d) 250 units
6. Which among the following nations announced one-month free visa on
arrival to foreign nationals from nearly 50 nations?
a) Sri Lanka
b) Bangladesh
c) Myanmar
d) Maldives
7. Union Minister Dharmendra Pradhan launched the Atal Community
Innovation Centre in which city?
a) Bangaluru
b) Karnal
c) New Delhi
d) Ranchi
8. When is the World Day Against Trafficking in Persons observed globally?
a) 28th July
b) 29th July
c) 30th July
d) 31st July
9. The Shawala Teja Singh temple is now open for Hindus after 72 Years.
Where is the temple located?
a) China
b) Pakistan
c) Bangladesh
d) Myanmar
10. Which nation has offered to mediate between Bangladesh and Myanmar
for early repatriation of Rohingyas?
a) Russia
b) China
c) India
d) Japan
Answers
1. (a) Delhi
The Supreme Court has directed the transfer of all five cases registered in
connection to the Unnao rape incident from Uttar Pradesh court to Delhi
court.
2. (b) Kuldeep Singh Sengar
The ruling BJP government expelled Unnao rape accused MLA Kuldeep
Sengar on August 1, 2019. Kuldeep Sengar, a BJP MLA in Uttar Pradesh, was
accused of raping a minor girl from Unnao.
3. (d) Nagaland
RN Ravi was sworn in as the new Governor of Nagaland on August 1, 2019
at a function at Raj Bhawan in Nagaland’s capital city Kohima.
4. (a) Karnataka
One nation one ration card scheme was launched on Thursday on a pilot
basis in four states- Gujarat, Maharashtra, Telangana and Andhra Pradesh.
The scheme will provide a major boost to the national food security
government.
5. (c) 200 units
Delhi Government announced to provide free electricity to all Delhi
households consuming 200 units of power or less every month. Delhi Chief
Minister Arvind Kejriwal stated that households will get free lifeline electricity
up to 200 units.
6. (a) Sri Lanka
Sri Lanka announced on Wednesday that it will give a one-month free visa
on arrival to visitors from nearly 50 countries. The move is a part of its
latest efforts to revive its tourism industry, which received a major hit after
the Easter Sunday terror attacks that killed 263 people.
7. (c) New Delhi
Union Minister of Petroleum and Natural Gas, Dharmendra Pradhan launched
the Atal Community Innovation Centre in New Delhi with an aim to
encourage innovation at the community level through solution-driven design
thinking.
8. (c) 30th July
The World Day against Trafficking in Persons is observed across the world on
July 30 with an aim to highlight that the human trafficking is a crime that
exploits women, children and men across the world.
9. (b) Pakistan
Shawala Teja Singh Temple has been re-opened for Hindu devotees for the
1st time in 72 years since partition. The temple is located in Sialkot city of
Pakistan and was sealed for last 72 years.
10. (d) Japan
Japan has offered to mediate between Bangladesh and Myanmar for early
repatriation of the Rohingyas. This was decided after a meeting between
Bangladesh Foreign Minister Dr AK Abdul Momen and his Japanese
counterpart Taro Kono in Dhaka.
22-28JULY
1. When was the 20th anniversary of Kargil Vijay Diwas observed?

a) 25th July

b) 26th July

c) 27th July

d) 28th July

2. Who has been appointed as UK’s first Indian-origin Home Secretary?

a) Priti Patel

b) Rishi Sunak

c) Alok Sharma

d) Nisha Kulhari

3. Who has been chosen as the next Prime Minister of United Kingdom?

a) Adward Luke

b) Jeremy Hunt

c) Boris Johnson

d) Karl Parkinson

4. Chandrayaan-2 took off from which of the following places?

a) Sriharikota

b) Porbandar

c) Chandipur

d) Pokhran
5. MS Dhoni will be deployed with the Territorial Army Battalion in which
state?

a) Himachal Pradesh

b) Arunachal Pardesh

c) Sikkim

d) Jammu and Kashmir

6. The Supreme Court has extended the deadline for Final NRC publication
till which date?

a) August 31st

b) July 31st

c) August 1st

d) August 15th

7. Who among the following won her fifth gold medal in the last 19 days?

a) Dipa Karmakar

b) Dutee Chand

c) Hima Das

d) Vinesh Phogat

8. India’s campaign in the World Test Championship will begin with which
nation’s tour?

a) Sri Lanka

b) West Indies

c) England

d) Bangladesh
9. Biswa Bhusan Harichandan has taken charge as the Governor of which
among the following states?

a) Maharashtra

b) Gujarat

c) Andhra Pradesh

d) Himachal Pradesh

10. India has rejected which among the following leader’s claim that PM
Modi asked him to be the mediator in Kashmir issue?

a) Kim Jong Un

b) Donald Trump

c) Xi Jinping

d) Vladimir Putin

Answer
1. (b) 26th July
20th Anniversary of Kargil Vijay Diwas was celebrated on July 26, 2019 to
commemorate the Operation Vijay of 1999, when India regained the control
of Kargil from Pakistan.

2. (a) Priti Patel


Priti Patel has been appointed as UK’s first Indian-origin home secretary.
Patel was appointed by Britain's new Prime Minister Boris Johnson. She
succeeds Pakistani-origin Sajid Javid, who has been appointed as the first
ethnic minority Chancellor of the Exchequer.

3. (c) Boris Johnson


Boris Johnson blustering Brexit campaigner was chosen as the U.K.’s next
prime minister recently, with a resounding mandate from the Conservative
Party but conflicting demands from a politically divided country.

4. (a) Sriharikota
Chandrayaan-2 was launched on 22 July 2019 at 2.43 pm IST from Satish
Dhawan Space Center, Sriharikota. Chandrayaan-2 consists of an orbiter
that will observe the surface of the Moon and become a source of
communication between Earth and the Moon mission.
5. (d) Jammu and Kashmir
MS Dhoni has begun his training with Indian Army's Territorial Army
Battalion. Dhoni will be deployed in Kashmir Valley for 15 days. He will be on
patrol and guard duties.

6. (a) August 31st


The Supreme Court on July 23, 2019 extended the deadline for publication
of the final National Register of Citizens (NRC) in Assam from July 31 to
August 31, 2019. The court also rejected the pleas seeking permission for 20
per cent sample re-verification.

7. (c) Hima Das


Hima Das, the Indian sprinter is on a gold run as she won her fifth gold
medal in the last 19 days on July 20 in the Czech Republic. Hima Das
finished at the top with a season-best time of 52.09 seconds in 400m at the
Nove Mesto nad Metuji Grand Prix.

8. (b) West Indies


The inaugural ICC World Test Championship will begin with the first Ashes
test between Australia and England at Birmingham on August 1, 2019. India
will begin its World Test Championship campaign with its two-match Test
series against the West Indies in August 2019.

9. (c) Andhra Pradesh


Biswa Bhusan Harichandan has taken charge as the new Governor of Andhra
Pradesh in Rajbhavan in Vijayawada, Andhra Pradesh. His oath was
administered by the AP High Court Acting Chief Justice Justice C. Praveen
Kumar.

10. (b) Donald Trump


India has sharply rejected US President Donald Trump’s claim stating that
PM Modi asked for US mediation in Kashmir issue. India lodged a protest
regarding the controversial claim with the US State Department.

8-14JULY
1. Which public sector bank recently announced to do away with
charges on NEFT and RTGS transactions through internet and mobile
banking from July 1?
a) PNB

b) SBI
c) OBC

d) Bank of India

2. Which nation has become the first to qualify into the World Cup
2019 finals?
a) England

b) Australia

c) India

d) New Zealand

3. Which country was recently declared measles free by World


Health Organization?
a) Sri Lanka

b) Nepal

c) Bangladesh

d) Pakistan

4. Which Indian sprinter won gold in 100m race during 30th Summer
University Games held in Italy?
a) Hima Das

b) Dutee Chand

c) Renuka Awasthi

d) Mani Ratna

5. Ten Congress MLAs joined BJP in which among the following


states?
a) Maharashtra

b) Karnataka

c) Goa

d) Madhya Pradesh
6. The RBI recently finalised a ‘Three Year Roadmap’ to improve
supervision and regulation. What is the name of roadmap?
a) Utkarsh 2022

b) Utsaah 2022

c) Vishwas 2022

d) Vikalp 2022

7. Who has been recently appointed as head of the National Cricket


Academy?
a) MS Dhoni

b) Sachin Tendulkar

c) Rahul Dravid

d) Harbhajan Singh

8. According to the United Nations, which country has enriched


uranium more than limit set in an international treaty of 2015?
a) Iran

b) Iraq

c) Jordan

d) Pakistan

9. Which city of India has recently been included in the list of World
Heritage Sites by UNESCO?
a) Lucknow

b) Ranchi

c) Jaipur

d) Ludhiana

10. India’s Child mortality reduced to what percent as per the


Multidimensional Poverty Index 2019?
a) 4 percent
b) 3.5 percent

c) 2.2 percent

d) 2.0 percent

ANSWERS
1. (b) SBI
State Bank of India recently announced to do away with charges on NEFT
and RTGS transactions through internet and mobile banking from July 1. The
move came after the RBI decided announced the same decision to boost
digital payments.

2. (d) New Zealand


New Zealand on July 10, 2019 became the first World Cup finalist after it
beat India by 18 runs in a narrow match at the Old Trafford cricket ground in
Manchester.

3. (a) Sri Lanka


The World Health Organization (WHO) announced that Sri Lanka had
successfully eliminated measles. WHO praised Sri Lanka's achievement in a
statement at a time when measles cases were increasing globally.

4. (b) Dutee Chand


India’s sprinter Dutee Chand creates history by winning first ever gold medal
in 100m race at World University Games in Naples. She has become first
Indian sprinter to win 100m Gold in this race. Dutee Chand clocked 11.32
seconds to win this gold.

5. (c) Goa
The Congress has split in Goa with ten Congress MPs from the total 15
joining the BJP in the presence of party working president J P Nadda in New
Delhi. The Congress MLAs formally split and merged with BJP on July 10,
2019.

6. (a) Utkarsh 2022


The Reserve Bank of India (RBI) finalised ‘Three Year Roadmap’ named as
Utkarsh 2022 to improve supervision and regulation. The plan is in line with
Global standards to strengthen regulatory and supervisory mechanism.

7. (c) Rahul Dravid


BCCI announced that former Indian captain Rahul Dravid was appointed as
Head of Cricket at the National Cricket Academy (NCA). Rahul Dravid was
supposed to take over as NCA head from July 1 but his job at the India
Cements caused the delay.

8. (a) Iran
The United Nations' atomic energy agency has confirmed Iran has surpassed
the uranium enrichment limits spelled out in the 2015 nuclear deal. The
International Atomic Energy Agency said its inspectors verified that Iran has
surpassed the 3.67% enrichment limit set in the accord, aimed at restraining
Tehran's ability to develop nuclear weapons in return for sanctions relief.

9. (c) Jaipur
'Pink City' Jaipur made an official entry in the list of the UNESCO World
Heritage sites and leaders across the political spectrum welcomed the signal
honour to the historical city. The UNESCO shared the announcement on
Twitter. The decision was taken at the 43rd session of the Unesco World
Heritage Committee.

10. (c) 2.2 percent


UN Development Programme released the 2019 Multidimensional Poverty
Index, which states that India’s Child mortality dropped from 4.5 percent to
2.2 percent. India successfully lifted over 250 million people out of poverty
during 2006 – 2016.

1-7JULY
1. Which batsman became the first Indian to cross 1000-run mark in 2019?
a) Virat Kohli
b) Rohit Sharma
c) KL Rahul
d) Rishabh Pant

2. India recently signed a deal to buy ‘Strum Ataka’ anti-tank missile from
which country?
a) Russia
b) USA
c) Japan
d) Israel

3. What is India’s position in Henley Passport Index 2019?


a) 76th
b) 82nd
c) 78th
d) 86th

4. Who has been recently re-appointed as RBI’s Deputy Director?


a) NS Viswanathan
b) RP Verma
c) SP Chaudhary
d) KK Shankar

5. Union Jal Shakti Ministry recently launched which initiative for water
conservation and water security in 1592 stressed blocks in 256 districts?
a) Jal Shakti Abhiyan
b) Jal Shakti Morcha
c) Jal Shakti Yojana
d) Jal Shakti Samman

6. Which country banned single-use plastic shopping bags with effect from 1
July 2019?
a) China
b) Japan
c) Australia
d) New Zealand

7. Which state government has decided to prepare the National Citizen


Register on the basis of Assam Government?
a) Karnataka
b) Tamil Nadu
c) Nagaland
d) Manipur

8. Which organisation launched the STRIDE scheme to boost research


culture in institutions?
a) Ministry of Human Resource Development
b) University Grants Commission
c) Central Board of Secondary Education
d) Ministry of Skill Development

9. The Economic Survey 2019 has projected how much growth in GDP in
2019-20?
a) 8 percent
b) 7 percent
c) 9 percent
d) 11 percent

10. The world- renowned Jagannath Rath Yatra has begun in which Indian
state?
a) Uttar Pradesh
b) Odisha
c) Jharkhand
d) Chhattisgarh

Answers

1. (b) Rohit Sharma


Rohit Sharma becomes the first Indian batsman to cross 1000-run mark in
ODIs in 2019 after hitting a century in India vs Bangladesh cricket match of
ICC World Cup 2019.

2. (a) Russia
India signed Rs 200 crore deal to buy ‘Strum Ataka’ anti-tank missile from
Russia for making its fleet of Mi-35 attack choppers. Recently, the Indian Air
Force has acquired the Spice-2000 stand-off weapon system.

3. (d) 86th
India is ranked at the 86th position in the Henley Passport Index 2019 with a
mobility score of 58. India shares the position with Mauritiana and Sao Tome
and Principe. In 2018, India was ranked at the 79th position.

4. (a)NS Viswanathan
The Appointment Commitments of the Cabinet has approved the re-
appointment of NS Viswanathan as Deputy Governor of RBI for a period of
one year with effect from 4 July. Earlier on June 24, RBI Deputy Governor
Viral Acharya resigned six months before the scheduled end of his term to
return to New York University.

5. (a) Jal Shakti Abhiyan


Union Jal Shakti Minister Shri Gajendra Singh Shekhawat recently launched
Jal Shakti Abhiyan for water conservation and water security in 1592
stressed blocks in 256 districts.

6. (d) New Zealand


New Zealand banned single-use plastic shopping bags with effect from 1 July
2019. New Zealand retailers will no longer be able to sell or distribute any
single-use plastic bags. Over 80 countries have already introduced such
similar bans.

7. (c) Nagaland
Nagaland government has decided to start a variant of the National Register
of Citizens (NRC), and the updating process is ongoing in Assam to detect
illegal immigrants. With an objective to prevent the issuance of fake
indigenous inhabitant certificates, the Nagaland government has decided to
set up a Register of Indigenous Inhabitants Certificates of Nagaland (RIIN)
soon, said officials.

8. (b) University Grants Commission


University Grants Commission (UGC) launched the STRIDE scheme to
enhance the research culture and innovation in colleges and universities.
STRIDE stands for “Scheme for Trans-disciplinary Research for India’s
Developing Economy.”

9. (b) 7 percent
The Economic Survey has projected 7 percent growth in GDP in 2019-20.
The Economic survey was tabled by Finance Minister Nirmala Sitharaman in
the Parliament on July 4, 2019.

10. (b) Odisha


The world-famous Jagannath Rath Yatra began at Puri in Odisha on July 4,
2019. Lakhs of people are gathering in Puri to witness Lord Jagannath, Lord
Balabhadra and Devi Subhadra riding three giant wooden chariots as they
travel to Gundicha temple.
Union Budget 2019 Quiz: Important
Questions Based on Budget 2019-20
1. What is the name of new water conservation scheme announced
by Finance Minister Nirmala Sitharaman during budget 2019 speech?
a) Jal Jeevan Mission

b) Jal Jyoti Mission

c) Jal Sanrakshan Mission

d) Samaan Jal Adhikar Mission

2. Name of education body announced by Nirmala Sitharaman to


promote research in India.
a) National Research Foundation

b) National Scholars Foundation

c) Research Institute of India

d) All India Research Institute

3. The Union Government has proposed to make which two identity


cards interchangeable?
a) PAN and Aadhaar

b) Aadhaar and Voter ID

c) Metro card and Aadhaar

d) Voter ID and PAN card

4. How much amount is allocated in the Union Budget 2019 to boost


credit in Public Sector Banks?
a) Rs 20,000 crore

b) Rs 50,000 crore

c) Rs 70,000 crore

d) Rs 90,000 crore
5. The Union Budget 2019 targets what amount through the
disinvestment of CPSEs including Air India?
a) Rs 90,000 crore

b) Rs 1,05,000 crore

c) Rs 1,50,000 crore

d) Rs 1,95,000 crore

6. The Union Government has proposed modernization of which


among the following in Budget 2019 to enable ease of living?
a) Airport

b) Bus Stands

c) Metro Stations

d) Railway stations

7. An encyclopedia is being developed on which among the following


freedom fighters?
a) Vallabhbhai Patel

b) Subhas Chandra Bose

c) Chandra Shekhar Azad

d) Mahatma Gandhi

8. Name the yojana that has been proposed by the Union


Government for the fisheries sector?
a) Pradhan Mantri Karamyogi Maandhan Yojana

b) Pradhan Mantri Matsya Sampada Yojana

c) Pradhan Mantri Matsya Shram Yojana

d) Pradhan Mantri Bandargah Maandhan Yojana

9. The Budget 2019 reduces tax rate for companies having annual
turnover up to Rs 400 crore. How much tax these companies need to
pay now?
a) 10 percent

b) 15 percent

c) 20 percent

d) 25 percent

10. For which sector a new TV channel would be launched as


announced in Budget 2019?
a) Banking

b) Start-ups

c) Education

d) Parenting

Answer
1. (a) Jal Jeevan Mission
Nirmala Sitharaman announced Jal Jeevan Mission under Jal Shakti
Mantralaya to achieve Har Ghar Jal aim to all rural households by 2024. She
also informed that 1592 critical and over exploited Blocks spread across 256
District being identified for the Jal Shakti Abhiyan.

2. (a) National Research Foundation


Finance Minister Nirmala Sitharaman proposed National Research Foundation
(NRF) to fund, coordinate and promote research in the country. It will also
help to assimilate independent research grants given by various Ministries.

3. (a) PAN and Aadhaar


Union Finance Minister Nirmala Sitharaman while presenting the Union
Budget 2019 announced that PAN card and Aadhaar will be made
interchangeable.

4. (c) Rs 70,000 crore


Union Budget 2019 proposes the provision of Rs. 70,000 crore to Public
Sector Banks to boost credit. The banks have been tasked to leverage
technology, offer online personal loans and doorstep banking, and enable
customers of one PSB to access services across all PSBs.

5. (b) Rs 1,05,000 crore


Budget 2019-20 targets Rs 1, 05,000 crore through the disinvestment
receipts of the FY 2019-20. Government will re-initiate the process of
strategic disinvestment of Air India, and other CPSEs for strategic
participation by the private sector.

6. (d) Railway stations


The Union Government has proposed modernization of Railway stations to
enable ease of living. The modernization process will be undertaken later
this year.

7. (d) Mahatma Gandhi


Finance Minister Nirmala Sitharaman has announced that the development
of Gandhipedia. The encyclopedia is being developed by national council for
science museum to sensitize the youth on Gandhian philosophy.

8. (b) Pradhan Mantri Matsya Sampada Yojana


The Union Government has proposed Pradhan Mantri Matsya Sampada
Yojana to address critical infrastructure gap in fisheries sector. A robust
fisheries management framework will be set up by department of fisheries.

9. (d) 25 percent
The budget 2019 reduces Tax rate to 25% for companies with annual
turnover up to Rs. 400 crore. The move will benefit over 99 percent Indian
Companies.

10. (b) Start-ups


Government will start a new TV channel under DD bouquet of channels for
start-ups. This channel will help start-ups to strengthen and grow their
initiatives in a better way. Channel’s name and programmers will be
announced soon.

24-30JUNE
1. Name of RBI Deputy Governor who resigned six months before his term
ends?
a) Pankaj Sinha
b) Abhijit Prasad
c) Suresh Prajapati
d) Viral Acharya

2. What is the venue of G-20 Summit 2019?


a) Paris
b) New Delhi
c) Moscow
d) Osaka
3. Who was appointed as Director of Research and Analysis Wing (RAW)?
a) Samant Kumar Goel
b) Anil Kumar Dhasmana
c) Sushil Chaturvedi
d) T Chandrashekaran

4. Padma Vibhushan E. Sreedharan recently resigned as the Principal


Advisor to which corportation?
a) DMRC
b) LMRC
c) KMRC
d) MMRC

5. Who has been recently appointed IB director?


a) Vipin Kumar
b) Arvind Kumar
c) Sanjiv Seth
d) Ashwani Singh

6. The IOC has stripped the Olympic status of which among the following
International sports associations?
a) FIH
b) ISSF
c) IWF
d) AIBA

7.Which country has agreed to revoke its citizenship given to Mehul Choksi?
a) Antigua
b) Australia
c) Peru
d) Brazil

8. Which Indian bowler has recently become the second Indian bowler who
has made hat-tricks in World Cup history?
a) Mohammed Shami
b) Bhuvneshvar Kumar
c) Kuldeep Yadav
d) Jasprit Bumrah

9. When was World Refugee Day observed?


a) June 21st
b) June 22nd
c) June 20th
d) June 23rd

10. Which industry icon is set to retire as Executive Chairman of his


company in July?
a) Azim Premji
b) Ratan Tata
c) Lakshmi Mittal
d) Shiv Nadar
Answers

1.(d) Viral Acharya


RBI Deputy Governor Viral V Acharya has resigned from his post, six months
before the scheduled end of his term. Acharya joined the central bank in
January 2017 and was RBI’s youngest deputy governor, post economic
liberalisation. Acharya will be returning to New York University as CV Starr
Professor of Economics.

2. (d) Osaka
Prime Minister Narendra Modi met Japanese Prime Minister Shinzo Abe
ahead of the two-day G20 Summit slated to take place in Osaka. He will be
meeting several world leaders, including US President Donald Trump. He will
also hold a trilateral meeting on the sidelines of the summit with Russian
and Chinese Presidents Vladimir Putin and Xi Jinping, respectively.

3. (a) Samant Kumar Goel


Senior IPS officer Samant Kumar Goel was appointed as the Director of
Research and Analysis Wing (RAW), external intelligence of India. His
appointment was approved by Appointments Committee of Cabinet (ACC) for
a fixed tenure of two years.

4. (b) LMRC
Padma Vibhushan E. Sreedharan recently resigned as the Principal Advisor to
Lucknow Metro Rail Corporation (LMRC) citing health issues. Sreedharan is
popularly known as the Metro Man. He holds years of experience in handling
mega Metro projects in the country.

5. (b) Arvind Kumar


The government has appointed senior IPS officer Arvind Kumar as the
director of the Intelligence Bureau. He is considered as an expert hand on
matters related to Kashmir.

6. (d) AIBA
The International Olympic Committee (IOC) has stripped Olympic status
from the troubled world boxing body AIBA. The AIBA president, Gafur
Rakhimov is under the United States federal sanctions for suspected links to
eastern European organized crime. As a result, the IOC will now organize
qualifying and final tournaments for the 2020 Tokyo Games.

7. (a) Antigua
In fresh trouble for fugitive diamantaire Mehul Choksi, the Antiguan
government decided to revoke his citizenship. Once his citizenship is
revoked, Choksi is likely to be extradited to India where he is wanted by the
Central Bureau of Investigation (CBI) and the Enforcement Directorate (ED).

8. (a) Mohammed Shami


Mohammed Shami has become the second-ever Indian bowler to get a hat-
trick in a world cup match with his hat-trick in the last over of India’s world
cup match against Afghanistan. The first-ever Indian bowler to get a hat-
trick was Chetan Sharma in 1987 Cricket World Cup.

9. (c) June 20th


The World Refugee Day was observed globally on June 20, 2019. The day’s
theme this year was ‘#StepWithRefugees -Take a Step on World Refugee
Day’. The day aimed to challenge everyone to join together and take a step
with refugees.

10. (a) Azim Premji


IT Industry icon Azim Premji will retire as Executive Chairman of Wipro with
effect from July 30, 2019. He led the company for 53 years. He will continue
to serve on the board as Non-Executive Director and founder Chairman. His
son Rishad Premji, Chief Strategy Officer and a board member, will take
over as the new Executive Chairman of the company.

GK Quiz and Answer on Indian Polity


Parliamentary System in India
1. “Parliamentary Government” is also known as....
(A) Cabinet Government

(B) Responsible Government

(C) Westminster forms of government

(D) All of the above

Ans. D
Explanation: The Parliamentary Government is also known as cabinet
government or Responsible Government responsible and Westminster forms
of government and is prevalent in Britain, Japan Canada, India and other
countries.
2. Which of the following characteristics is not related to
Parliamentary Government?
(A) Resolution of lower house

(B) Collective liability

(C) Leadership of the Prime Minister

(D) Single Executive

Ans. D
Explanation: features of parliamentary system are: Dual Executive,
majority party rule, collective responsibility, political homogeneity, double
membership, leadership of Prime Minister, dissolution of lower and house
fusion of power.
3. Which of the following is not the merit of the Presidential System?
(A) Permanent Government

(B) Confirmation in policies

(C) Limited Representation

(D) Government by experts

Ans. C
Explanation: Features of presidential system are: single executive, non-
responsibility, political homogeneity may not exist, domination of president,
no dissolution power of lower house and separation of powers.
4. Which of the following is a demerit of the Parliamentary System?
(A) Compressed representation

(B) Uncertainty of policies

(C) Temporary Government

(D) All of the above

Ans. D
Explanation: All of the above are demerits of the Parliamentary System.
5. Which of the following statements is not correct?
(A) According to the 42nd and the 44th amendment, it is compulsory for
president to comply with the advice of the Council of Ministers.
(B) Presidential System is based on double executive.

(C) According to the Article 74 the Council of Ministers works under the
leadership of the Prime Minister.

(D) Article 74 and 75 provides parliamentary system in the center.

Ans. B
Explanation: Presidential System is based on single executive.
6. Which of the following characteristics is not related to the
Presidential System?
(A) Governance of the Prime Minister

(B) Single Executive

(C) Single Membership

(D) Dissolution of lower house is baned

Ans. A
Explanation: Presidential System is the Governance of the President.
7. Which of the following characteristics is not related to the federal
government?
(A) Written Constitution

(B) Flexible Constitution

(C) Supremacy of the Constitution

(D) Independent Judiciary

Ans. B
Explanation: Flexible Constitution provision is not related to the federal
government.
8. In the Federal Government................
(A) All the powers and functions are vested in the Central Government and
Regional Government.

(B) All powers are divided into Central Government and Regional
Government

(C) Both A and B

(D) None of the above


Ans. B
Explanation: Both A and B
9. The Federal System in India is based on the model of which
country?
(A) Canada

(B) UK

(C) America

(D) Japan

Ans. A
Explanation: The Federal System in India is adopted from the Canada.
10. Which of the following statements is wrong?
(A) Lok Sabha represents the people of India.

(B) The Rajya Sabha represents the states.

(C) There are only 98 topics in the center list at this time.

(D) Rajya Sabha protects the state with unnecessary interference from the
Center

Ans. C
Explanation: The Union List or List-I is a list of 101 items given in Seventh
Schedule in the Constitution of India on which Parliament has exclusive
power to legislate

Quiz on Indian Polity: Chief Minister of the


State
1. Which of the following statement is false about the Chief Minister?
(A). There is no special provision for appointment and election of Chief
Minister in the constitution
(B). Article 154 states that the Governor will appoint the Chief Minister
(C). The Governor is free to appoint any person as Chief Minister
(D). There is no such system in the Constitution that the Chief Minister
should prove a majority before his appointment
Ans. B
2. What is the minimum age to be appointed as the Chief Minister of
a state?
(A). 25 years
(B). 30 years
(C). 35 years
(D). 18 years
Ans. A
3. Which of the following post is hold by the Chief Minister?
(A). Chairman of the State Planning Board
(B). Member of National Development Council
(C). Chief Spokesperson of the State Government
(D). All of the above
Ans. D
4. Which of the following is not matched correctly?
(A). Article 167: Duties of the Chief Minister
(B). Article 163: sworn in of the Chief Minister
(C). Article 164: Provisions related to State Ministers
(D). Article 166: Operations by the State Government
Ans. B
5. Who determines the salary and allowances of the Chief Minister?
(A). Governor
(B). State Legislature
(C). President
(D). Parliament
Ans. B
6. Which of the following statements is false?
(A). Chief Minister's tenure is not fixed (with some conditions)
(B). If the Chief Minister resigns from his post then the entire Council of
Ministers has to resign
(C). The Governor is the President of the Inter-state Council
(D). The collective responsibility of the Council of Ministers is to the State
Legislature
Ans. C
7. Which of the following power is not enjoyed by the Chief Minister?
(A). The Governor appoints ministers only to those people who are
recommended by the Chief Minister
(B). Chief Minister shifts all the ministers' departments
(C). Chief Minister can ask the governor to disassociate the Legislative
assembly
(D). Chief Minister appoints judges of the state's high court
Ans. D
8. The State Council is responsible to whom?
(A). To the Governor
(B). To the Legislative Assembly
(C). To the Legislative Council
(D). To the State Legislature
Ans.B
9. Who provide oath to other ministers of the state other than the
Chief Minister?
(A). Chief Minister
(B). Speaker of the assembly
(C). Chief Justice of the High Court
(D). Governor
Ans. D
10. The Advocate General of the State is responsible to whom?
(A). Governor
(B). Chief Minister
(C). Speaker of the Assembly
(D). None of these
Ans. A

GK Questions and Answers on the


proceedings of the State Legislature
1. What is the minimum age for becoming a member of the State
Legislative Council?
(A) 18 years

(B) 21 years

(C) 25 years

(D) 30 years

Ans. D
2. Which of the following statements is not correct?
(A) The person becoming the member of the Legislative Assembly should
also be a voter in the constituency of the respective state

(B) It is not necessary that a person who is elected to the Legislative Council
should also be eligible to be choosen for the Legislative Assembly.

(C) Should not hold office of profit

(D) Keeps all the qualifications as per the People's Representation Act, 1951

Ans. B
3. How soon should the elections to be held in the state after the end
of the Emergency?
(A) 1 month
(B) 3 months

(C) 6 months

(D) 12 months

Ans. C
4. Who can take the decision to cancel the Assembly membership of
any person under the 10th schedule?
(A) Governor

(B) Chief Minister

(C) Assembly Speaker

(D) Chief Justice of the High Court

Ans. C
5. Member of the Legislative Council submits his resignation to ....
(A) Chairman

(B) President

(C) Chief Minister

(D) Governor

Ans. A
6. Who does not participate in the election of the Legislative Council?
(A) Graduates

(B) Members of the municipality

(C) Teachers of secondary schools

(D) District Panchayat members

Ans. D
7. Speaker of Legislative Assembly surrenders his resignation
to.......?
(A) Chief Minister

(B) Governor
(C) Chief Justice of the High Court

(D) Deputy Speaker

Ans. D
8. Which of the following statements is not correct?
(A) A minister can also participate in the proceedings of the House, he is not
a member of that house

(B) A minister who is not a member of the House, can participate in the
proceedings of both houses

(C) The quorum is the 10th share of the total members in a single house

(D) The Advocate General can vote in the House

Ans. D
9. Which of the following statements is not correct?
(A) Finance Bill cannot be submitted in the Legislative Council

(B) The Finance Bill is presented by the Chief Minister of the state

(C) The Legislative Council has no right to change the Finance Bill

(D) Finance Bill is presented in both houses

Ans. B
10. Who decides the salary and allowance of the speaker of the
Legislative Assembly?
(A) Legislature

(B) Governor

(C) Chief Minister

(D) None of the following

Ans. A

GK Questions and Answers on Gases and


their Uses
1. Name the gas which is found in abundance in earth in both
combine and free state with other elements?
A. Oxygen
B. Nitrogen
C. Hydrogen
D. Sulphur
Ans. A
2. Name an inert diatomic gas which is neither combustible nor helps
in combustion?
A. Carbon Dioxide
B. Hydrogen
C. Nitrogen
D. None of the above
Ans. C
3. Name a gas which is used in refrigeration and in fire extinguisher?
A. Nitrogen
B. Hydrogen
C. Carbon Dioxide
D. Methane
Ans. C
4. Name a gas which is used in the manufacture of vanaspati ghee,
alcohol and ammonia?
A. Hydrogen
B. Oxygen
C. Propane
D. Butane
Ans. A
5. Name a gas used as rocket propellant and in welding also?
A. Ether
B. Neon
C. Hydrogen
D. Cyanogen
Ans. D
What is SMOG and how it is harmful for us?
6. Name a tear gas which is used in riots?
A. Ethylene
B. Krypton
C. CS gas
D. Nitrous Oxide
Ans. C
7. Name a gas used in high speed photography?
A. Nitrous Oxide
B. Krypton
C. Xenon
D. Radon
Ans. B
8. Name a gas which is separated from the air by passing an electric
current through water?
A. Carbon Dioxide
B. Oxygen
C. Butane
D. Nitrogen
Ans. B
9. Name a gas which is used in the fermentation of sugar?
A. Ammonia
B. Butane
C. Propane
D. Carbon Dioxide
Ans. D
10. Name a gas which is used in making chloroform?
A. Methane
B. Propane
C. Butane
D. Acetylene
Ans. A
From the above questions it is clear that what are gases and for what
purpose they are used
GK Questions and Answers on the Student
Movements in India
1. Who was the founder of Student’s scientific and historic society?
A. Dadabhai Naoroji

B. Motilal Nehru

C. Jawaharlal Nehru

D. Subash Chandra Bose

Ans: A
Explanation: The student’s scientific and historic society was founded by
Dadabhai Naoroji in 1848 as a discussion forum. A is the correct option.
2. Where was the first student strike organised during British rule?
A. NIIT, Calicut

B. King Edward Medical College, Lahore

C. Women Sanskrit College, Pattambi


D. Patna University, Bihar

Ans: B
Explanation: The student strike at King Edward Medical College Lahore was
the first student strike regarding academic discrimination between the
English people and Indians. Hence, B is the correct option.
3. Which of the following national movement during British India
gave a revolutionary outlook such as boycott colleges as well as
British Goods, Student clubs?
A. Non-Cooperation Movement

B. Quit India Movement

C. Swadeshi Movement

D. Rowlatt Satyagraha

Ans: C
Explanation: Swadeshi Movement organised the student movement in India
and gave a revolutionary outlook such as boycott colleges as well as British
Goods, Student clubs. C is the correct option.
4. When did 'All India College Students Conference' was organised?
A. 1909 AD

B. 1910 AD

C. 1911 AD

D. 1912 AD

Ans: D
Explanation: In 1912, All India College students Conference of Ahmedabad
nailed the student commitment to work for the freedom of the India. D is the
correct option.
GK Questions and Answers on the Environmental Movements in
India
5. What was slogan given in All India College Students Conference?
A. Charka Swaraj first, and education after

B. Swaraj is our birth right

C. Education for all

D. School chalo, padhai karo


Ans: A
Explanation: In 1912, All India College students Conference of Ahmedabad
nailed the student commitment to work for the freedom of the India and
gives moto of ‘Charka Swaraj first, and education after”. A is the correct
option.
6. When was Hindu Students Federation founded?
A. 1933 AD

B. 1934 AD

C. 1936 AD

D. 1937 AD

Ans: C
Explanation: The Hindu Students Federation (HSF) was started in 1936
with the ideology of RSS. This wing of student directly appeals to the
sentiments of Hindu youth from its inception. C is the correct option.
7. In which year All India Muslim Students Federation (AIMSF) was
founded by the Muslim League?
A. 1935 AD

B. 1936 AD

C. 1937 AD

D. 1938 AD

Ans: C
Explanation: All India Muslim Students Federation (AIMSF) founded by the
Muslim League in 1937 to address the grievances of Muslim students and
wanted a support for separate state for Muslims from the Muslim student
wing.
8. Which of the following student organisation founded after
splitting with Congress in 1948 AD?
A. Young Socialist League

B. Akhila Bharathiya Vidyarthi Parishad (ABVP)

C. National Students Union of India (NSUI)

D. India Student Congress Federation (AISCF)

Ans: A
Explanation: The Young Socialist League was founded after splitting with
Congress in 1948. A is the correct option.
GK Questions and Answers on the Cooperative Movements in India
9. Which of the following is not correctly matched?
A. Naxalite Movement: started by the Communist Party of India in 1959

B. Dashauli Gram Swaraj Sangh (DGSS): Chandi Prasad Bhatt

C. Student Federation of India (SFI): Founded by the CPI (M) in 1970

D. Chhatra Sangharsh Samiti: Lalu Prasad and Sushi Modi

Ans: A
Explanation: Naxalite Movement was started by Charu Majumdar, Kanu
Sanyal, and Jangal Santhal in 1967.Dashauli Gram Swaraj Sangh (DGSS)
was started by Chandi Prasad.Student Federation of India (SFI) was founded
by the CPI (M) in 1970. During the anti-emergency movement, the ABVP
and the Socialists first time join hands and formed the Chhatra Sangharsh
Samiti. Lau Prasad was elected as president and Sushi Modi was elected as
general secretary. Hence, A is the correct option.
10. Which of the following student movement was the creative
resistance of the professional college students against globalisation?
A. Women student's movements

B. Cyber activist student movements

C. Anti-Mandal Student Agitation

D. Chipko Movement

Ans: B
Explanation: Cyber activist student movement was started by the
engineering students of NIIT Calicut against the monopoly of the Micro Soft
corporations in the software industry. It is also known as ‘free software
movement’. Hence, B is the correct option.
GK Questions and Answers on Nobel Prize
2019
Nobel Prize is awarded in 6 fields namely Chemistry, Physics, Medicine,
literature, Economics and Peace prize.

1. In which year first Nobel Prize was awarded?


A.1893
B.1895
C.1898
D.1901
Ans. D
Explanation: The first Nobel Prize was awarded in 1901. The first Nobel
prize winners were given 150,782 SEK.
2. Who won Nobel Prize in Physics in 2019?
A.JamesPeebles
B.MichelMayor
C.DidierQueloz
D.Alltheabove
Ans. D
Explanation: James Peebles is given for the theoretical discoveries in
physical cosmology, and Michel Mayor and Didier Queloz for discovering an
exoplanet orbiting a solar-type star.
3. Peter Handke received the Nobel Prize of 2019 in which field?
A. PhysiologyorMedicine
B. Literature
C. Peaceprize
D. EconomicScience
Ans.B
Explanation: Peter Handke wins Nobel Prize 2019 in Literature for an
influential work with linguistic ingenuity which has explored the periphery
and the specificity of human experience.
4. Abiy Ahmed Ali won Nobel Peace Prize 2019 for...
A. His efforts to achieve peace and international cooperation.
B. For his decisive initiative to resolve the border conflict with neighbouring
Eritrea.
C. BothAandB
D. NeitherAnorB
Ans.C
Explanation: Abiy Ahmed Ali wins Nobel Peace 2019 for his efforts to
achieve peace and international cooperation and in particular for his decisive
initiative to resolve the border conflict with neighbouring Eritrea.
5. Name the youngest Nobel Laureates who got the Peace Prize in
2014?
A. MalalaYousafzai
B. ArthurAshkin
C. JamesP.Allison
D. YoshinoriOhsumi
Ans.A
Explanation: Malala Yousafzai is the youngest Nobel Laureates who got the
Peace Prize in 2014 at the age of just 17 years.
6. Name the Nobel laureate who has been awarded the Nobel prize in
Physics twice?
A. Bob Dylan
B. Angus Deaton
C. John Bardeen
D. Hiroshi Amano
Ans.C
Explanation: John Bardeen is the only Nobel Laureate who has been
awarded the Nobel Prize in Physics twice, in 1956 and 1972.
7. Name the only woman who has been honoured with Nobel Prize
twice in different areas?
A. Gerty Cori
B. Marie Curie
C. Elinor Ostrom
D. Barbara McClintock
Ans.B
Explanation: Marie Curie is the only one woman who has been honoured
twice, with the 1903 Nobel Prize in Physics and the 1911 Nobel Prize in
Chemistry.
8. Who received the Nobel Prize 2019 in Chemistry?
A. John.B.Goodenough
B. M.Stanley.Whittingham
C. AkiraYoshino
D. Alltheabove
Ans.D
Explanation: Nobel Prize in Chemistry 2019 winners are John B.
Goodenough, M. Stanley Whittingham and Akira Yoshino "for the
development of lithium-ion batteries”.
9. The Sveriges Riksbank Prize in Economic Sciences in Memory of
Alfred Nobel 2019 is given to:
A. Abhijit Banerjee
B. Esther Duflo
C. Michael Kremer
D. All the above
Ans. D
Explanation: Abhijit Banerjee, Esther Duflo and Michael Kremer won The
Sveriges Riksbank Prize in Economic Sciences in Memory of Alfred Nobel
2019 for their experimental approach to alleviating global poverty.
10. Who won the first Nobel Peace Prize in 1901?
A. Henry Duant
B. Frederic Passy
C. Both A and B
D. Neither A nor B
Ans.C
Explanation: The first Nobel Peace Prize was awarded in 1901 to Henry
Dunant (Switzerland) and Frederic Passy (France).
GK Questions and Answers about Dr. APJ
Abdul Kalam
1. What is the full name of Dr. Abdul Kalam?
(a) Avul Jakir Jalaluddin Kalam
(b) Avul Pakir Jainulabdeen Abdul Kalam
(c) Abdul Sakir Jainulabdeen Kalam
(d) None of the above
Answer: b
Explanation: The full name of APJ Abdul Kalam was Avul Pakir Jainulabdeen
Abdul Kalam.
2. When was Abdul Kalam born?
(a) 15 October 1931
(b) 2 September 1929
(c) 15 August 1923
(d) 29 February 1936
Answer: a
Explanation: Abdul Kalam was born in Dhanushkodi, Rameswaram, Tamil
Nadu, India on 15 October 1931. He was born in a Muslim Family.
3. Which island is named after Abdul Kalam?
(a) Wheeler Island, Odisha
(b) Landfall Island
(c) Bhavani Island
(d) Sriharikota
Answer: a
Explanation: The Wheeler Island is located in Odisha. Currently this island
is known as the Dr. Abdul Kalam Island.This is an island off the coast of
Odisha, approximately 150 kilometres east of the state capital
Bhubaneshwar.
4. Which of the following book is not written by the Abdul Kalam?
(a) Failure to Success: Legendry Lives
(b) You Are Born to Blossam
(c) Ignited Minds
(d) A House for Mr Biswas
Answer:d
Explanation: A House for Mr Biswas is written by the V. S. Naipaul in 1961.
Rest of the books given in the options were written by the Dr. Abdul Kalam.
5. Which of the following statement is NOT correct about the Dr.
Abdul Kalam.
(a) Dr. Abdul Kalam received the Bharat Ratna in 2007
(b) Dr. Abdul Kalam died on 17 July 2015 (aged 83) in Assam, India
(c) India 2020: A Vision for the New Millennium was written in 1998.
(d) Kalam had worked at the Defence Research and Development
Organisation
Answer: b
Explanation: Dr. Abdul Kalam died on 27 July 2015 (aged 83) in Shillong,
Meghalaya, India while delivering a lecturer at the Indian Institute of
Management Shillong. Kalam saab collapsed and died from cardiac arrest.
6. Which of the following award is not given to DR. Kalam?
(a) Padma Bhushan
(b) Padma Vibhushan
(c) Shanti wraroop Bhatnagar
(d) Bharat Ratna
Answer:c
Explanation: Shanti wraroop Bhatnagar award is not given to DR. Kalam
Saab. While other awards were given to Abdul Kalam. Kalam received Bharat
Ratna (1997), Padma Vibhushan (1990) and Padma Bhushan (1981).
7. Dr. Abdul Kalam was the ......President of India.
(a) 9th
(b) 10th
(c) 11th
(d) 12th
Answer: c
Explanation: Dr. Abdul Kalam was the 11th (in person) President of India.
He remained in office from 25 July 2002 to 25 July 2007.
8. To whom Abdul Kalam defeated in the Presidential election in
2002?
(a) K. R. Narayanan
(b) Lakshmi Sahgal
(c) Krishan Kant
(d) Bhairon Singh Shekhawat
Answer: b
Explanation: Abdul Kalam defeated Lakshmi Sahgal in the Presidential
election held in 2002. He had won the 2002 Presidential election with an
electoral vote of 922,884 while Lakshmi Sahgal got just 107,366 votes.
GK Question and Answer on Parliamentary
Committees in India
1. Which of the following statement is wrong with regard to
Parliamentary Committee?
(a) Parliamentary Committee is appointed/elected by the speaker/chairman

(b) Works under the guidance of speaker/chairman


(c) Has a secretariat provided by the president of India

(d) Submits its reports to house or speaker/chairman

Answer c
Explanation: Secretariat provided by the Lok Sabha/Rajya Sabha
2. Which of the following statement is not correct?
(a) Mainly Parliamentary Committees are of two types

(b) Ad hoc committees are of permanent nature

(c) Parliamentary Committees are mentioned in the Indian constitution

(d) Indian Constitution does not mentioned any time period for the
Parliamentary Committees

Answer b
Explanation: Ad hoc committees are of temporary nature.
3. Which of the following committee is NOT made for day to day
business of the house?
(a) Business Advisory Committee

(b) Committee of private members bill

(c) Rules committee

(d) Ad hoc committee

Answer d
Explanation: Ad hoc committee
4. How many members are in Public Account Committee?
(a) 22

(b) 15

(c) 7

(d) 10

Answer a
Explanation: 22
5. Which of the following statement is not correct about Public
Account Committee?
(a) Public Account Committee is made as per the provision of Government of
India Act, 1919

(b) This Committee consists of members from both the house of the
Parliament

(c) Term of the members of the Committee is 5 year

(d) A minister cannot be member of the Committee

Answer c
Explanation: Term of the members of the Public Account Committee is 1
year
6. Which of the following committee regulates the programme and
time table of the house of Parliament?
(a) Committee on public understanding

(b) Public Account Committee

(c) Estimates Committee

(d) Business advisory committee

Answer d
Explanation: Business advisory committee
7. Which of the following is not matched correctly?
(a) Committee on labour : Labour and employment committee

(b) Committee on rural development: Panchayti Raj Ministry

(c) Committee on transport: Civil Aviation Ministry

(d) Committee on energy: New and Renewable Energy Ministry

Answer b
Explanation: Committee on rural development: Rural Development
Ministry
8. Which of the following committee enforces code of conduct of
members of Parliament?
(a) Ethics committee

(b) House committee

(c) Committee of privileges


(d) Ad-hoc committee

Answer. a
Explanation: Ethics committee enforces code of conduct of members of
Parliament. It examines the cases of misconduct and recommends
appropriate action.
9. Which of the following is matched correctly?
(a) House Committee: deals with residential accommodation of members
and other amenities like food, medical etc.

(b) Library Committee: it is related to calculate all the expenses of


parliament during business meetings.

(c) Rules committee: it frames rules for regulating salary allowances and
pension of the members of Parliament

(d) Public Account Committee: To examine the reports and accounts of


public undertakings

Answer. a
Explanation: House Committee: deals with residential accommodation of
members and other amenities like food, medical etc.
10. The function of the …………….committee is to examine the
estimates included in the budget and suggests economies in public
expenditure.
(a) Public Accounts Committee

(b) Rules Committee

(c) Estimate Committee

(d) Ethics Committee

Answer. c
Explanation: Estimate Committee

GK Questions and Answers on the Planets


with the Largest Number of Natural
Satellite
1. Which of the following planet has largest number natural
satellite?
A. Jupiter
B. Saturn

C. Uranus

D. Neptune

Ans: A
Explanation: Jupiter is the fifth planet from the Sun and named after the
Roman god Jupiter. This planet has 67 natural satellites. Hence, A is the
correct option.
2. Which of the following planet has second largest number of
natural satellite and it is named after the Roman god of agriculture?
A. Jupiter

B. Saturn

C. Uranus

D. Neptune

Ans: B
Explanation: Saturn is the sixth planet from the Sun and the second-
largest in the Solar System, after Jupiter. It is named after the Roman god
of agriculture. It has 62 natural satellites and second highest number of
natural satellites after Jupiter. Hence, B is the correct option.
3. Which of the following planet situated in the Kuiper belt, a ring of
bodies beyond Neptune and has fifth largest number of natural
satellite?
A. Neptune

B. Uranus

C. Pluto

D. Mars

Ans: C
Explanation: Pluto is a dwarf planet in the Kuiper belt, a ring of bodies
beyond Neptune. It has 5 natural satellites. Hence, C is the correct option.
4. Which of the following planet have two natural satellite Phobos
and Deimos?
A. Neptune
B. Uranus
C. Pluto
D. Mars
Ans: D
Explanation: Mars has two natural satellites, Phobos and Deimos. The
discovery of the two Mars satellites dates back to the late 1800s, owing
thanks to astronomer Asaph Hall, who observed the two moons through a
telescope at the United States Naval Observatory in Washington, D.C.
Hence, D is the correct option.
5.Which of the following is/are correct?
A. Uranus-27 (Number of natural satellite)

B. Neptune- 14 (Number of natural satellite)

C. Haumea- 2 (Number of natural satellite)

D. All the above are correct

Ans. D
Explanation: Uranus has 27, Neptune 14 and Haumea has 2 number of
natural satellites.
GK Questions and Answers on the Natural Satellite of the Solar
System
6. Which of the following planet caused Pluto to lose its status as a
planet?
A. Eris

B. Neptune

C. Makemake

D. Uranus

Ans: C
Explanation: Makemake is a dwarf planet and discovered by a team led by
Michael E. Brown on March 31, 2005. It is one of the bodies that caused
Pluto to lose its status as a planet. Hence, C is the correct option.
7. Which of the following dwarf planet discovered by a team led by
Michael E. Brown?
A. Eris

B. Pluto

C. Haumea
D. Makemake

Ans: D
Explanation: Makemake is a dwarf planet and discovered by a team led by
Michael E. Brown on March 31, 2005. Hence, D is the correct option.
8. Which of the following is the third planet from the sun and only
planet known to have an atmosphere containing free oxygen, oceans
of water on its surface?
A. Earth

B. Mars

C. Venus

D. Jupiter

Ans: A
Explanation: Earth is the third planet from the sun and only planet known
to have an atmosphere containing free oxygen, oceans of water on its
surface and, of course, life. Hence, A is the correct option.
9. Which of the following planet is the fourth-largest planet by
diameter, the third-most-massive planet, the densest giant planet of
the Solar System and has 14 natural satellites?
A. Neptune

B. Uranus

C. Jupiter

D. Mars

Ans: A
Explanation: Neptune is the eighth and farthest known planet of the Solar
System. It is the fourth-largest planet by diameter, the third-most-massive
planet, and the densest giant planet of the Solar System. It has 14 natural
satellites. Hence, A is the correct option.
GK Quiz on Women Empowerment
Schemes in India
Indian Government has started several schemes to empower women in
various spheres. The principles of gender equality are also enshrined in the
Constitution of India. The Indian government has recognised various women
issues. Some of the government schemes are as follows which will increase
your general knowledge and also help in the preparation of competitive
examinations.

1. Name the online platform launched by the Ministry of Women and


Child Development to support women entrepreneurs?
A. Digital India
B. Mahila E-haat
C. One Stop Centre Scheme
D. None of the above
Ans.B
Explanation: Mahila E-haat scheme was launched by the Ministry of Women
and Child Development on 7 March, 2016 to support women entrepreneurs.
It is basically a direct online marketing platform.
2. The Central Social Welfare Board was established in....
A. 1950
B. 1951
C. 1952
D. 1953
Ans.D
Explanation: The Central Social Welfare Board was established in 1953 by a
Resolution of Government of India.
3. Women Helpline Scheme was launched in...
A. 2012
B. 2015
C. 2017
D. 2018
Ans.B
Explanation: Women Helpline Scheme was launched on 1 April, 2015.
4. Which of the following scheme provide education to girls and their
welfare?
A. One Stop Centre Scheme
B. UJJAWALA
C. SWADHAR Scheme
D. Beti Bachao Beti Padhao
Ans. D
Explanation: Beti Bachao Beti Padhao scheme provide education to girls
and their welfare.
5. Which scheme helps in the prevention of trafficking and provide
support for rescue, rehabilitation, reintegration and repatriation of
women?
A. Ujjawala scheme
B. STEP scheme
C. Priyadasrshini Scheme
D. DWCRA
Ans. A
Explanation: Ujjawala scheme is a Comprehensive Scheme for Prevention
of trafficking and Rescue, Rehabilitation and Re-integration of Victims of
Trafficking and Commercial Sexual Exploitation.
6. Support to Training and Employment Programme for Women scheme
is....
A. SWADHAR
B. STEP
C. Nari Shakti Puraskar
D. RMK
Ans. B
Explanation: STEP Scheme is Support to training and Employment
Programme for Women which provide skills that give employability to
women.
7. Priyadarshini scheme provides:
A. Women Empowerment
B. Livelihood in Mid-Gangetic plains
C. Empower to address women legal, political and health problems
D. All the above are correct.
Ans. D
Explanation: Priyadasrshini is a Women Empowerment and Livelihood
programme in Mid-Gangetic Plains.
8. Indira Gandhi Matritva Sahyog Yojana was launched in...
A. 2008
B. 2010
C. 2012
D. 2015
Ans. B
Explanation: Indira Gandhi Matritva Sahyog Yojana was launched in 2010.
It is a Conditional Maternity Benefit (CMB) Scheme of the Government of
India.
9. National Mission for Empowerment of Women was launched on:
A. 15 August, 2010
B. 15 August, 2011
C. 15 October, 2010
D. 15 October, 2011
Ans. B
Explanation: National Mission for Empowerment of Women was launched
on 15 August, 2011 to empower women holistically.
10. Which scheme promotes community participation through
involvement of Students Volunteers for empowerment of rural
women?
A. NAND-GHAR YOJANA
B. eSamvad Portal
C. Pradhan Mantri Mahila Shakti Kendra scheme
D. She-Box Portal
Ans. C
Explanation: Pradhan Mantri Mahila Shakti Kendra scheme promote
community participation through involvement of Students Volunteers for
empowerment of rural women.
These are the Indian Government Schemes that work for the empowerment
of the women in different spheres.
GK Quiz on life of Mahatma Gandhi: Set 1
1. Where was Gandhi ji born?
(a) Porbandar

(b) Rajkot

(c) Ahmedabad

(d) Delhi

Answer: a
Explanation: Mohandas Karamchand Gandhi was born in Porbandar,
Gujarat, India on 2 October 1869.
2. What was the age of Gandhi ji at the time of his marriage?
(a) 12 yr

(b) 13 yr

(c) 16 yr

(d) 20 yr

Answer: b
Explanation: The 13-year-old Mohandas Gandhi got married to 14-year-old
Kasturbai Makhanji Kapadia (her first name was usually shortened to
"Kasturba", and affectionately to "Ba") in May 1883. It was a arrange
marriage.
3. About how old was Gandhi when he reached London to become a
barrister?
(a) 20 years

(b) 19 years

(c) 21 years
(d) 16 years

Answer: b
Explanation: At the age of 19 Gandhi ji left from Bombay to London.
Gandhi attended University College, London which is a constituent college of
University of London.
4. Who was the political guru of Mahatma Gandhi ji?
(a) Ravindra Nath Tagore

(b) Swami Vivekananda

(c) Gopal Krishna Gokhale

(d) None of the above

Answer: c
Explanation: Mahatma Gandhi used to seek the opinion of the Gopal
Krishna Gokhale through letters from South Africa. Gokhale was the one who
persuaded Gandhi to came back to India, invest time in understanding India
and work for Indian independence struggle movement.
5. From which station in South Africa Gandhi was thrown out of the
train?
(a) Natal

(b) Johannesburg

(c) Pietermaritzburg

(d) Durban

Answer: c
Explanation: In April 1893, Gandhi aged 23, set sail for South Africa to be
the lawyer for Abdullah's cousin. He was thrown off a train at
Pietermaritzburg after refusing to leave the first-class.
6. At which place was Gandhiji arrested for the first time by the
British Government for sedition?
(a) Bombay

(b) Pune

(c) Calcutta

(d) Ahmedabad
Answer: d
Explanation: Mahatma Gandhi was arrested at Sabarmati on 10 March
1922 and sentenced to six-year jail for sedition. However, he eventually
served only two years of that term.
7. On which day of March 1930, famous Dandi March launched by the
Gandhi ji?
(a) Tenth

(b) Thirteenth

(c) Twelfth

(d) Eleventh

Answer: c
Explanation: Dandi March is also known as the Salt March, Salt Satyagraha
and the Dandi Satyagraha was started on 12 Mar 1930 and concluded on 6
Apr 1930. Dandi March was an act of nonviolent civil disobedience against
the British.
8. When was the Gandhi - Irwin Pact signed?
(a) March 1, 1932

(b) March 5, 1931

(c) March 10, 1935

(d) March 7, 1937

Answer: b
Explanation: The 'Gandhi-Irwin Pact' was a political agreement between the
Lord Irwin and Mahatma Gandhi on 5 March 1931 before the second Round
Table Conference in London.
9. Who given the slogan of 'Do or die'?
(a) Subhash Chandra Bose

(b) Bipin Chnadra Pal

(c) Sarojini Naidu

(d) None of these

Answer: d
Explanation: Mahatma Gandhi had given the slogan of 'Do or die' during
the Quit India movement of 1942.
10. Who killed Gandhi Ji?
(a) Ruskin Bond

(b) Nathuram Godse

(c) Lord Mountbatten

(d) Satya Bhan Gokhale

Answer: b
Explanation: Nathuram Vinayak Godse assasinated the Gandhi in New Delhi
on 30 January 1948.Godse was an advocate of Hindu nationalism, from
Pune, Maharashtra.
1. Which of the following statement is NOT true about the Gandhi ji?
(a).He was married to Kasturba Makhanji at the age of 13 years.
(b). Gandhi ji returned to India at the age of around 46 years.
(c). Gopal Krishna Gokhle is assumed to be the political guru of Gandhi ji.
(d). He founded the Sabarmati Ashram in Ahmedabad (Gujarat) in 1918.
Answer: d
Explanation: Gandhi ji returned to India at the age of around 46 years then
roamed around whole India and finally founded the Sabarmati Ashram in
Ahmedabad (Gujarat) in 1915 to preach his teachings.
2. Which of the following statement is/are CORRECT about the
Champaran Satyagraha?
i. It was started on the 1 April 1918
ii. The Champaran Satyagraha was the first Satyagraha movement inspired
by Gandhi
iii. It was farmer’s movement
(a). Only i & iii

(b). Only ii & iii

(c). Only i

(d). Only I & ii

Answer: b
Explanation: Champaran Satyagraha was launched on 19 April 1917.This
was Gandhi’s first Satyagraha movement and a farmer’s revolt in Bihar
against the indigo crop production.
3. Who of the following person is not associated with the Champaran
Satyagraha?
(a). Brajkishore Prasad
(b). Rajendra Prasad
(c). Anugrah Narayan Sinha
(d). Govind Ballabh Pant
Answer: d
Explanation: Champaran Satyagraha was organised by the Gandhi ji,
Rajendra Prasad (first President of India), Brajkishore Prasad, Anugrah
Narayan Sinha, Mazhar-ul-Haq and J. B. Kripalani.
4. Which of the following statement is not CORRECT about 'The Story
of My Experiments with Truth'?
i. It was originally written in the English language

ii. It covers life of Gandhi ji since early childhood to 1921.

iii. This book was originally published in two volumes, first in 1927 and
second in 1929.

(a). i & ii

(b). Only i

(c). ii & iii

(d). i & iii

Answer: c
Explanation: ‘The Story of My Experiments with Truth’ was originally
published in Gujarati language. This book was translated in English by
Mahadev Desai in 1940. This book was originally published in two volumes,
the first in 1927 and second in 1929.
5. Which of the following was the second Satyagrah of Gandhi ji in
India?
(a). Kheda Satyagraha
(b). Ahmedabad mill strike
(c). Champaran Satyagraha
(d). None of the above
Answer: b
Explanation: Ahmedabad Mill Strike, 1918 was second movements led by
Gandhi ji in India. The Mill Owners wanted to withdraw the bonus while the
workers demanded a 50% wage hike against 20% offered by the Mill
Owners.
6. What was 'Teen Kathiya' practice during the British Raj in India?
(a). A type of rent to be paid by the farmers of Bihar
(b). Mandatory farming of Indigo
(c). A type of Pilgrim tax to be paid by the farmers to the Britishers
(d). None of the above
Answer: b
Explanation: 'Teen Kathiya' was a practice of Mandatory farming of Indigo
for the Farmers of Bihar. The British forced farmers in Bihar to grow indigo
on three Kathas land for every twenty Kathas (3/20) they owned.
7. What was the name of mother of Mahatma Gandhi?
(a). Leelawati
(b). Putlibai
(c). Sharda Bai
(d). Kusuma Devi
Answer: b
Explanation: Father of Mahatma Gandhi was Karamchand Uttamchand
Gandhi while mother's name was Putlibai Gandhi.
8. Who said 'Half-naked fakir' to Gandhi ji?
(a). Lord Mountbatten
(b). Clement Attlee
(c). Ramsay MacDonald
(d). Winston Churchill
Answer: d
Explanation: Winston Churchill referred to Mohandas Gandhi as the 'half-
naked fakir' while Gandhi regarded the expression as a compliment.
9. What is the correct chronological order of movements led by
Gandhi ji?
(a). Kheda Peasant Satyagraha, Khilafat and non-Co-operation Movement,
Individual Civil Disobedience started
(b). Non-Co-operation Movement, Champaran Satyagraha, Kheda Peasant
Satyagraha
(c). Khilafat Movement, Kheda Peasant Satyagraha, Champaran
Satyagraha
(d). Champaran Satyagraha, Khilafat Movement, Kheda Kheda Peasant
Satyagraha
Answer: a
Explanation: Champaran Satyagraha (1917), Kheda Peasant Satyagraha
(1918), Khilafat and non-Co-operation Movement (1920-21), Individual Civil
Disobedience (1933).
10. Which of the following is not matched correctly?
(a). Gandhi-Irwin Pact:1931
(b). Gandhi ji presided over the Belgaum session of Congress: 1926
(c). Chauri Chaura incident: 1922
(d). Gandhi started Individual Civil Disobedience:1933
Answer: b
Explanation: The Indian National Congress (INC) held its 39th session at
Belgaum (Karnataka) on the 26th & 27th Dec.1924. This was the only
session of INC which was headed by the Gandhi ji ever.
1. Who called the 'father of the nation' to Gandhi ji?
(a) Ravindra Nath Tagore
(b) Subhash Chandra Bose
(c) Gopal Krishna Gokhale
(d) None of these
Answer. b
Explanation: Mahatma Gandhi was known as ‘the father of the nation’ but
there is no legal backup for this title from the Government of India. Subhas
Chandra Bose claimed Mahatma Gandhi as Rashtriyapita on a radio program
in Singapore in 1944.
2. When did Gandhiji take the vow of Brahmacharya or celibacy of
life?
(a) 1911

(b) 1906

(c) 1900

(d) 1905

3. Who was the Private Secretary of Mahatma Gandhi?


(a) Mahadev Desai

(b) Pyarelalji

(c) Kishorilal Mashruwalla

(d) Sushila Nayyar

4. Who in South Africa gave Gandhiji 'Unto This Last' to read which
proved to be one of the most decisive books of his life?
(a) John Holmes Haynes

(b) H S Polak

(c) Hermann Kallenbach

(d) Louis Fischer

5. To put the ideas of 'Unto This Last' into practice, Gandhiji founded
the Phoenix Settlement near Durban which came into being in the
middle of the year ________________.
(a) 1903

(b) 1904
(c) 1905

(d) 1906

6. Who described Gandhi's march to Dandi in the following words?


"Like the historic march of Ramchandra to Lanka, the march of
Gandhi will be memorable"
(a) Motilal Nehru

(b) Sarojini Naidu

(c) Jawaharlal Nehru

(d) Vallabhai Patel

7. The historic August session of the All-India Congress Committee,


at which the Quit India Resolution was passed, was held at Gowali
Park in __________________.
(a) Bombay

(b) Calcutta

(c) Ahmedabad

(d) Amritsar

8. Gandhiji accorded very high priority to communal harmony in his


programme of actions. At which place did he undertake his last fast
for it on January 13, 1948?
(a) Nasik

(b) Bombay

(c) Calcutta

(d) Delhi

9. Which of the following did Gandhiji describes as his two lungs?


(a) Ahimsa and peace

(b) Truth and Peace

(c) Ahimsa and truth


(d) Brahamcharya and Aparigriha

10. The differences with Gandhiji led Subhas Chandra Bose to resign
the Presidentship of the India National Congress in 1939. Leaving
the Congress he formed a new party called __________________.
(a) Indian National Party

(b) Forward Bloc

(c) Freedom Party

(d) Freedom Bloc

Question No. Answer


1 b
2 b
3 a
4 b
5 b
6 a
7 a
8 d
9 c
10 d
1. Who is the author of 'Unto This Last'?
A. John Ruskin

B. Ruskin Bond

C. Hermann Kallenbach

D. Louis Fischer

Ans: A

Explanation: 'Unto This Last' is an essay and book on economy by John


Ruskin. It was between August and December 1860 in the monthly journal
Cornhill Magazine in four articles. Gandhi ji was influenced by this book.
2. Which of the following, according to Gandhiji, is an essential
principle of Satyagraha?
A. Infinite capacity for suffering

B. Non violence

C. Truth

D. All the three

Ans: D

Explanation: 'Satyagraha' is the most important weapon of Gandhi ji. It


emerged as a weapon of conflict resolution. Gandhi ji applied satyagraha in
the non-violent struggle against exploitation, injustice and dictatorship.
3. Gandhiji's "The Story of My Experiments with Truth" was
originally written in Gujarati. Who translated it into English?
A. Maganlal Gandhi

B. Mahadev Desai

C. Pyarelalji

D. Sushila Nayyar

Ans: B

Explanation: Mahadev Desai was an activist of Indian freedom movement.


He is best known for his job of Mahatma Gandhi's personal secretary. He
died on 15 August 1942.
4. Which one of the following books is the work of Gandhiji?
A. Light of India

B. Hind Swaraj

C. My Experiments with Truth

D. Both B & C

Ans: D

Explanation: The Story of My Experiments with Truth is an autobiography


of Gandhi ji. This book covers life of Gandhi ji from early childhood through
to 1921. It was published in his journal Navjivan from 1925 to 1929.
5. When Ganadhi ji won Nobel peace Prize?
A. 1937

B. 1947

C. 1939

D. Never

Ans: D

Explanation: Gandhi Ji never won Nobel peace Prize although Gandhi was
nominated in 1937, 1938, 1939, 1947 and, finally, a few days before he was
murdered in January 1948.
6. Who established the Natal Indian Congress (NIC)?
A. Vallabhbhai Patel

B. Sarojini Naidu

C. Jawaharlal Nehru

D. None of the above

Ans: D

Explanation: On 22 May, 1894 Gandhi established the Natal Indian


Congress (NIC) and worked hard to improve rights of Indians in South
Africa.
7. When Gandhi ji returned to India from South Africa?
A. 1918

B. 1910

C. 1915

D. 1905

Ans: C

Explanation: Gandhi ji returned to India in 1915 permanently and joined


the Indian National Congress with Gopal Krishna Gokhale as his mentor.
8. Book ‘The Satyahrah’ was originally written in ....................
A. English
B. Hindi

C. Gujarati

D. Bengali

Ans: C
9. Who was the political Guru of Mahatma Gandhi ji?
A. Gopal Krishna Gokhale

B. Dayanand Saraswati

C. Ravindra Nath Tagore

D. None of the above

Ans: A
Explanation: Mahatma Gandhi used to seek the opinion of the Gopal
Krishna Gokhale through letters from South Africa. Gokhale was the one who
persuaded Gandhi to came back to India, invest time in understanding India
and work for Indian independence struggle movement.
10. What was the name of mother of Mahatma Gandhi?
A. Leelawati

B. Putlibai

C. Sharda Bai

D. Kusuma Devi

Answer: B
Explanation: Father of Mahatma Gandhi was Karamchand Uttamchand
Gandhi while mother's name was Putlibai Gandhi.

GK Questions and Answers: Dadasaheb


Phalke Award
1.Which of the following statement is NOT correct about the
Dadasaheb Phalke?
(a) The first recipient of the Dadasaheb Phalke Award was Kanan Devi.
(b) Dadasaheb Phalke Award was given for the first time in 1969.
(c) The Phalke Award is India's highest award in cinema.
(d) Phalke Award is presented annually.
Answer. a
Explanation: The first recipient of the Dadasaheb Phalke Award was Devika
Rani.
2. Who of the following never received the Dadasaheb Phalke Award
?
(a)Mrinal Sen
(b)Rajesh Khanna
(c) Shyam Benegal
(d) Dilip Kumar
Answer. b
Explanation: Rajesh Khanna did not received Dadasaheb Phalke Award
while all other people given in the options have been awarded from this
award.
3. Who is considered as the father of Indian Cinema?
(a) Prithviraj Kapoor
(b) Birendranath Sircar
(c) Ashok Kumar
(d) None of these
Answer.d
Explanation: Dadasaheb Phalke is considered as the father of Indian
Cinema. He had made India's first full-length feature Raja Harishchandra
(1913).
4. When was Dadasaheb Phalke born?
(a) 1870
(b) 1891
(c) 1879
(d) 1905
Answer a
Explanation: The full name of the Dadasaheb Phalke. He was born on 30
April, 1870 in a Marathi-speaking Brahmin family.
5.In which state Dadasaheb Phalke was born?
(a)West Bengal
(b)Bihar
(c) Maharashtra
(d) None of these
Answer. c
Explanation: Dhundiraj Govind Phalke was born in Nashik District,
Maharashtra in 1870.
6. Which of the following film is not made by the Dhundiraj Govind
Phalke?
(a) Satyavan Savitri
(b) Mohini Bhasmasur
(c) Raja Harishchandra
(d) Aalamaara
Answer d
Explanation: Aalamaara movie was directed by the Ardeshir Irani. It was
the first Indian sound film.
7. When Dadasaheb Phalke died?
(a) 1962
(b) 1973
(c) 1965
(d) 1944
Answer d
Explanation: The full name of the Dadasaheb Phalke. He was born on 30
April, 1870 while he died on 16 February, 1944.
8. How many people got the Dadasaheb Phalke award so far?
(a)66
(b)50
(c) 65
(d) 45
Answer b
Explanation: There are 50 people have been awarded with this award.
Amitabh Bachchan is the latest recipient of this award.
9.What is the correct chronological order of receiving the Dadasaheb
Phalke award?
(a) Devika Rani, Prithviraj Kapoor, Naushad, Raj Kapoor
(b) Prithviraj Kapoor, Naushad, Raj Kapoor, Satyajit Ray
(c) Kanan Devi, Prithviraj Kapoor, Devika Rani, Naushad
(d) Bhupen Hazarika, Lata Mangeshkar, Sohrab Modi,Sivaji Ganesan
Answer a
Explanation: Devika Rani was the first recipient of this award. Later on it is
given to Prithviraj Kapoor, Naushad and Raj Kapoor respectively.
10. Who got the Dadasaheb Phalke award for 2018?
(a) Vinod Khanna
(b) Amitabh Bachchan
(c) Manoj Kumar
(d) None of these
Answer. b
Explanation: Amitabh Bachchan is selected for the 66th Dadasaheb Phalke
Award 2018. The Dadasaheb Phalke Awards is India's highest award in
cinema.
Questions & Answers Cell its Structure
and Functions
1. Who discovered cell in 1665?
a) Robert Hook

b) Robert Crook
c) David Thomson

d) Marie Francois

Ans. a)
2. Name an Organelle which serves as a primary packaging area for
molecules that will be distributed throughout the cell?
a) Mitochondria

b) Plastids

c) Golgi apparatus

d) Vacuole

Ans. c)
3. Name the outer most boundary of the cell?
a) Plasma membrane

b) Cytoplasm

c) Nuclear membrane

d) None of the above

Ans. a)
4. Name the process in which the ingestion of material by the cells is
done through the plasma membrane?
a) Egestion

b) Diffusion

c) Osmosis

d) Endocytosis

Ans. d)
5. Which among the following sentence is not correct about the
organelles?
a) They are found in all Eukaryotic cells.

b) They are found in multicellular organisms.


c) They coordinate to produce the cell.

d) They are small sized and mostly internal.

Ans. b)
what is the function of Earwax in the Human Body?
6. Name the process in which the passage of water goes from a
region of higher concentration to a region of lower concentration
through a semi permeable membrane?
a) Diffusion

b) Osmosis

c) Both a) and b)

d) Neither a) nor b)

Ans. b)
7. Name an organism which contains single chromosome and cell
division occurs through fission or budding?
a) Eukaryotes

b) Prokaryotes

c) Bacteria

d) Primitive organism

Ans. b)
8. Name the process in which the membrane of a vesicle can fuse
with the plasma membrane and extrude its contents to the
surrounding medium?
a) Exocytosis

b) Endocytosis

c) Osmosis

d) Diffusion

Ans. a)
9. The jelly like substance present inside the cell is known as:
a) Cytoplasm
b) Ectoplasm

c) Nucleoplasm

d) None of the above

Ans. a)
10. Blue green Algae are:
a) Prokaryotes

b) Eukaryotes

c) Both a) and b)

d) Neither a) nor b)

Ans. a)
Questions and Answers: World Bank
Group
1. Why World Bank was established?
(a) To promote the International Trade

(b) To reconstruct the economies damaged during the Second World War

(c) To improve the adverse Balance of Payment situation of the non member
countries

(d) None of the above

Ans. b
Explanation: The prime motive behind the establishment of the World Bank
was to provide long run capital to member countries for economic
reconstruction and development of the ruined economies.
2. Which of the following statement is correct?
(a) Every member country of the IMF automatically becomes the member of
the World Bank

(b) The World Bank has 45 founder members

(c) India is not the founding member of the World Bank

(d) IMF is the part of World Bank group


Ans. a
Explanation: Every member country of the IMF automatically becomes the
member of the World Bank. In the same way any member country which
quits the IMF is automatically expelled from the membership of the World
Bank.
3. Which of the following is not matched correctly?
(a) IBRD (estd.): 1945

(b) IFC (estd.): 1948

(c) IDA (estd.): 1960

(d) MIGA (estd.): 1988

Ans. b
Explanation: International Finance Corporation (IFC) was established in
July 1956, while rests of the organisation are matched correctly.
4. Currently how many members are in the IBRD?
(a) 193

(b) 198

(c) 189

(d) 206

Ans. c
Explanation: To become a member of the Bank, under the IBRD Articles of
Agreement, a country must first join the International Monetary Fund (IMF).
Currently there are 189 members are in the IBRD. Each institution of the
World Bank Group has different numbers of members.
5. Which of the following statements is not correct?
(a) Both the IMF & IBRD have headquarters in Washington

(b) ICSID is the constituent organisation of the World Bank Group

(c) IBRD is known as World Bank also

(d) India's vote share in the International Monetary Fund is 10%

Answer d
Explanation: India's share in the International Monetary Fund's vote share
is 2.64% of total votes while as on April 2018, India’s quota in the IMF is
2.76%.
6. Who is the current President of the World Bank Group?
(a) Robert Zoellick

(b) David Malpass

(c) Christine Lagarde

(d) Jim Yong Kim

Ans. b
Explanation: The current President of the World Bank Group is David
Malpass. He is the 13th President of World Bank Group. He was selected by
the Board of Executive Directors of the World Bank Group on April 2019.
7. Which of the following statements are correct?
(a) World Bank was established 73 years ago.

(b) Headquarter of World Bank is in Washington D.C.

(c) Adam Smith is the founding father of the World Bank

(d) Only (a) and (b)

Ans. d
Explanation: The World Bank was created at the 1944 Bretton Woods
Conference along with the International Monetary Fund (IMF).John Maynard
Keynes and Harry Dexter White are considered as the" founding fathers" of
the World Bank.
8. Who of the following never appointed as the President of the
World Bank Group?
(a) Paul Krugman

(b) Robert Zoellick

(c) George Woods

(d) Eugene Meyer

Ans. a
Explanation: Paul Robin Krugman is an American economist; who is
currently Professor of Economics at the Graduate Center of the City
University of New York. He has never been appointed as the President of the
World Bank Group.
9. Which of the following is not the function of the World Bank?
(a) To provide long term loan to the member countries
(b) To provide loan to private investors belonging to member countries on
its own guarantee

(c) To ensure exchange rate stability

(d) To provides loan mainly for productive activities

Ans. c
Explanation: The World Bank is playing the main role in providing loans for
the development works to the member countries. It provides long term loans
for 5 to 25 years. Assurance of exchange rate stability is the function of IMF.
10. Which of the following institutions is not part of the World Bank
community?
(a) IBRD

(b) WTO

(c) IDA

(d) IFC

Answer b
Explanation: WTO or World Trade Organisation is not part of the World
Bank community. World Bank group is comprised of the five institutions i.e.
International Bank for Reconstruction and Development (IBRD), the
International Development Association (IDA), the International Finance
Corporation (IFC), the Multilateral Investment Guarantee Agency (MIGA)
and the International Centre for Settlement of Investment Disputes (ICSID).
Questions and Answers on the Indian
Mural Painting
1. The word mural derived from which language?
A. Latin

B. English

C. Arabic

D. Sanskrit

Ans: A
Explanation: The word ‘mural’ derived from the Latin word ‘murus’ which
means wall. Hence, A is the correct option.
2. Which of the following is the correct definition of Mural Painting?
A. Any piece of artwork painted or applied directly on canvas

B. Any piece of artwork painted or applied directly on a wall, ceiling or other


permanent surface.

C. any piece of artwork painted or applied directly on a cloth.

D. Any piece of artwork painted or applied directly on human body.

Ans: B
Explanation: Mural Painting can be defined as any piece of artwork painted
or applied directly onto a wall, ceiling or other larger permanent surfaces,
flat, concave or convex, to be precise. Hence, B is the correct option.
3. Which of the following is/are the example of cave paintings?
A. Ajanta Caves

B. Armamalai Cave

C. Badami Cave Temples

D. All of the above

Ans: D
Explanation: Indian cave paintings and rock cut structures that reflect the
ingenuity and skill of their masters. Ajanta Caves, Armamalai Cave and
Badami Cave Temples are the best example of Mural Paintings or cave
paintings. Hence, D is the correct option.
GK Questions and Answers on the Drama written during Gupta
period
4. Which of the following cave paintings is documented as the
Chitrasutra of the Vishnudharmottara Purana?
A. Ajanta Caves

B. Armamalai Cave

C. Badami Cave Temples

D. All of the above

Ans: A
Explanation: The painting of Ajanta caves is documented as the Chitrasutra
of the Vishnudharmottara Purana. This was a verbal tradition, which would
have come over many centuries, passed on through guilds of painters.
Hence, A is the correct option.
5. Which of the following statement is/are correct about the
characteristics of Mural Painting?
A. It depict the activities of a particular civilization‘s people.

B. It is a combination of wide variety of artistic style

C. Both I and II

D. Neither I nor II

Ans: C
Explanation: Mural Painting is an oldest human art form, as cave paintings
at numerous ancient human settlements suggest, and can be found all over
globe. It depict the activities of a particular civilization‘s people,
encapsulating a moment in time, and range from scenes of hunting,
gathering, and family life, to religious and funerary scenes. It is a
combination of wide variety of artistic style, Realism with a dramatic sense
of scale and amazing depth. Hence, C is the correct option.
6. Which of the following ancient treatise discuss at length all
aspects of painting, including murals?
A. Vishnudharmottara

B. Silpashastra

C. Manasollasa

D. All of the above

Ans: D
Explanation: India has a rich tradition of mural wealth. The treatises such
as Vishnudharmottara, Silpashastra, Manasollasa, Shilparatna, Narada-
shilpa-shastra and Kashyapa-shilpa, discuss at length all aspects of painting,
including murals. Hence, D is the correct option.
GK Questions and Answers on the Ancient Astronomers and their
contributions
7. Mural Painting is -------- dimensional form of artwork.
A. two

B. three

C. four
D. five

Ans: B
Explanation: Mural Painting is three-dimensional form of artwork. Hence, B
is the correct option.
GK Questions and Answers on the Ancient Indian School of Art
8. Which of the following cave painting is focus on the life of
Buddhas and Jatakas?
A. Kerala Murals

B. Bagh Murals

C. Ajanta Murals

D. None of the above

Ans: C
Explanation: Ajanta Murals (paintings carved on solid substances like rocks
or caves), mainly focus on the life of Buddhas and Jatakas, with a great deal
of beauty and tenderness carved by artists. Hence, C is the correct option.
9. Which of the following temple is a huge rock cut monolithic
structure built in the South Indian style?
A. Kailasanatha Temple at Ellora

B. Bagh Murals

C. Both A and B

D. none of the above

Ans: A
Explanation: The Kailasanatha Temple at Ellora dates back to the period of
the 8th century Rashtrakoota ruler Krishna. It is a huge rock cut monolithic
temple built in the South Indian style. The temple is known for its
architectural grandeur as well as its sculptural splendour. Hence, A is the
correct option.
10. Which of the following is not types of Indian Mural Paintings?
A. Tempera Painting

B. Oil Painting

C. Fresco painting

D. All of the above


Ans: D
Explanation: There are mainly four types of Mural Paintings: Tempera
Painting, Oil Painting, Fresco painting and Encaustic Painting. Hence, D is the
correct option.
Questions and Answers: Central Vigilance
Commission
1.When was Central Vigilance Commission was set up?
(a)1954
(b)1995
(c)1964
(c)1985
Answer. c
Explanation:
Central Vigilance Commission (CVC) was created in 1964 to address
governmental corruption, monitoring all vigilance under the central
government, and advising various authorities.
2.Who is the current Central Vigilance Commissioner of India?
(a) Sharad Kumar
(b) K. V. Chowdary
(c) Pradeep Kumar
(d) Rajeev Maharshi
Answer. a
Explanation: Sharad Kumar is the current Central Vigilance Commissioner
of India.
3.Which of the following statement is NOT correct about the Central
Vigilance Commission of India?
(a)The Central Vigilance Commissioner is to be appointed by the President of
India.
(b)Tenure of Central Vigilance Commissioner is 5 years
(c) Central Vigilance Commissioner can be removed or suspended from the
office by the President on the ground of misbehavior but only after the
Supreme Court has held an inquiry into his case and recommended action
against him.
(d) The Central Vigilance Commission is mainly an advisory body and has no
adjudicatory functions
Answer. b
Explanation: The Central Vigilance Commissioner (CVC) is to be appointed
by the President of India. He holds the office for 4 years or till the age of 65
years which ever is earlier.
4.Who of the following is not the part of committee to select the
CVC?
(a)Home Minister
(b)Leader of opposition in the Lok Sabha
(c) Prime Minister of India
(d)Leader of opposition in the Rajya Sabha
Answer.d
Explanation: The Central Vigilance Commissioner is to be appointed by the
President of India on the recommendations of the committee comprises of
the PM. Home Minister and Leader of Opposition in the Lok Sabha.
5. Which of the following is not the function of the CVC?
(a) The Commission is mainly an advisory body and has no adjudicatory
functions.
(b) To exercise superintendence over the functions of Delhi special police
establishment
(c)To tender advice to the central government and all its authorities on such
matters referred to it by them.
(d)All of the above
Answer. d
Explanation: All are the functions of the CVC.
6.Where is the head quarter of CVC?
(a) Mumbai
(b) Kolkata
(c)New Delhi
(d) Bangaluru
Answer.c
Explanation: The headquarter of CVC is in New Delhi.
7.Who was the first Central Vigilance Commissioner of India?
(a) Sharad Kumar
(b) Nittoor Srinivasa Rau
(c) T. U. Vijayasekharan
(d) B. K. Acharya
Answer. b
Explanation: Nittoor Srinivasa Rau, was selected as the first Chief
Vigilance Commissioner of India in 1964.
8.Which committee recommend to establish Chief Vigilance
Commission in India?
(a)Santhanam Committee
(b)Goipariya Committee
(c)Raj Mananar Committee
(d) None of the above
Answer.a
Explanation: The Commission was set up by the Government of India
Resolution on 11 February 1964. CVC was established on the
recommendations of Shri K. Santhanam Committee.
Central Vigilance Commission (CVC) is an apex Indian governmental body
created in 1964 to address governmental corruption, monitoring all vigilance
under the central government, and advising various authorities in central
government organization in planning, executing, reviewing their vigilance
work. It has the status of an autonomous body.

It was set up by the Government of India in February, 1964 on the


recommendations of the Committee on Prevention of Corruption, headed by
Shri K. Santhanam Committee, to advise and guide Central Government
agencies in the field of vigilance. Nittoor Srinivasa Rau, was selected as the
first Chief Vigilance Commissioner of India.

Service conditions and appointment of Vigilance Commissioner


The Central Vigilance Commissioner is to be appointed by the President of
India. He holds the office for 4 years. He can be removed or suspended from
the office by the President on the ground of misbehavior but only after the
Supreme Court has held an inquiry into his case and recommended action
against him.

Functions
The Commission is mainly an advisory body and has no adjudicatory
functions. It mainly considers the complaints relating to corruption,
misconduct, lack of integrity or some other kinds of malpractice or
misdemeanor on the part of the public servants.

It cannot extend sanction for criminal prosecution for offences committed by


public servants. It has no machinery to investigate or inquire into complaints
of corruption except to a limited extent.

The commission is not authorized to investigate the complaint itself, it has to


refer them to the Central Bureau of Investigation or Ministry or Department
for investigation. However, the Chief Technical Examiner’s Organization
attached to it, conducts technical examination of public works including
checking of bills of contractors, contracts and muster rolls. The Commission
advices as to the action to be taken in following cases:

(i) Reports of investigation by the Central Bureau of Investigation which


involves departmental action or prosecution in the matter either referred to
it by the commission or otherwise.

(ii) Reports of investigation by the Ministry or department involving the case


of disciplinary action in the matters either referred by the Commission or
otherwise.
(iii) Cases received direct from public sector undertakings and statutory
corporations; etc.

The commission is required to submit an annual report to the ministry of


Home Affairs, stating the cases in which its recommendation were accepted
and acted upon by the competent authorities.

The long-standing demand for the formation of an act to deal with the
provisions relating to constitution, jurisdiction, power and function of the
commission was ultimately meted by passing an act. The said Act was
named as central Vigilance Commission Act, 2003.

Functions and Powers of Central Vigilance Commission—


(1) The functions and powers of the commission shall be to—

(a) Exercise superintendence over the functioning of the Delhi Special Police
Establishment insofar as it relates to the investigation of offences alleged to
have been committed under the Prevention of Corruption Act, 1988

(b) Give directions to the Delhi Special Police Establishment for the purpose
of discharging the responsibility entrusted to it under sub-section (1) of
section 4 of the Delhi Special Police Establishment Act, 1964 (25 of 1946).

Questions and Answers on State


Information Commission
1.The post of State Information Commissioner is created due to the
existence of..
(a) Right to Information Act
(b) Right to Education Act
(c) Central Information Commissioner
(d) None of the above
Answer: a
Explanation: The Right to Information Act, 2005 provides way for the
creation of State Information Commission at the State level.
2. The State Information Commissioner holds the office for the
period of..
(a) 3 years
(b) 5years
(c) 6 years
(d) Not fixed
Answer:b
Explanation: The State Chief Information Commissioner and a State
Information Commissioner hold office for a term of 5 years or until they
attain the age of 65 years.
3. Which of the following sentence is not correct about the State
Information Commissioner (SIC)?
(a) SIC is appointed by the Governor of the state
(b) SIC can be reappointed
(c) SIC submits annual report to the state government
(d) State Information Commission is consists of a Chief State Information
Commissioner (SCIC) and not more than 10 State Information
Commissioners (SIC).
Answer: b
Explanation: State Information Commissioners can not be reappointed on
the post. Their tenure is fixed for 5 or until they attain the age of 65 years.
4. Who appoints the chief and members of the State Information
Commission?
(a) The Governor of the state alone
(b) The President of India
(c) A committee headed by the Chief Minister of the state
(d) None of the above
Answer: c
Explanation: The chief and members of the State Information Commission
are appointed by the Governor on the recommendation of the committee
consisting of the Chief Minister as Chairperson, the Leader of the Opposition
in the Legislative Assembly and a state Cabinet Ministers.
5. Who can remove the State Information Commissioner?
(a) The Chief Minister of the state
(b) the Chief State Information Commissioner
(c) President of India
(d) The Governor of the state
Answer: d
Explanation: He can be removed by the governor in certain circumstances
like incapacity, office of profit and insolvency.
6. Under which of the following circumstance the state chief and any
other State Information Commissioner can be removed by the
governor?
(a) If he is adjudged insolvent
(b) If he is engage in any other paid job outside his office
(c) if he is unfit to continue in the eyes of governor
(d) If he does not believe in the god
Answer: d
Explanation: Believe in the god is the matter of personal choice. So this is
not the appropriate reason for removal of the state information
commissioner and its chief.
7. Which of the following is not the power and function of the State
Information Commissioner?
(a) The commission can inquire into any matter on reasonable grounds
(b) During inquiry the commission has power of a civil court.
(c) During inquiry of a complaint the commission can not examine any
information which is in the custody of public authority.
(d) Commission can recommend the disciplinary action against the errant
official
Answer: c
Explanation: During inquiry of a complaint the commission can examine
any information which is in the custody of public authority.
8. How many State Information Commissioners are in different
states?
(a) 10
(b) 8
(c) The number varies from state to state
(d) 5
Answer: c
Explanation: The number varies from state to state. But state information
commissioners can not not be more than 10 State Information
Commissioners (SIC).
9.Salary allowance and other services conditions of the State chief
Information Commissioners are similar to ……
(a)Election commissioner
(b)Chief Secretary of the state government
(c) Chief Information Commissioner
(d) Chief Justice of Supreme Court
Answer: a
Explanation: Salary allowance and other services conditions of the State
chief Information Commissioners are similar to Election commissioner of
India.
10.Who is the current Chief Information Commissioner of India
currently?
(a) Shri Bimal Julka
(b) Shri Sudhir Bhargava
(c) Mrs Vanaja N Sarna
(d) Shri Divya Prakash Sinha
Answer: b
Explanation: Shri Sudhir Bhargava is the current Chief Information
Commissioner of India. He had been working as one of the information
commissioner since June 2015.
Quiz on Indian Polity on Article 370 and
Attached Office and Subordinate Office
. To which of the following states, Article 370 if the Indian
constitution is related:
(a) Arunachal Pradesh

(b) Meghalaya

(c) Himachal Pradesh

(d) Jammu & Kashmir

2. Article 370 is drafted in the part………of the Indian Constitution.


(a) XXI

(b) XIX

(c) XII
(d) IXX

3. Article 370 which gives special status to Jammu & Kashmir exist
in the Indian constitution because of the agreement between............
(a) Jawahar Lal Nehru and Farukh Abdullah

(b) Jawahar Lal Nehru and Maharaja Hari Singh

(c) Ballabh Bhai Patel and Maharaja Hari Singh

(d) Muhammad Ali Jinnah and J.L. Nehru

4. Which statement is NOT correct about the Jammu& Kashmir


(a) Jammu & Kashmir has its own Constitution.

(b) No decision regarding the disposition of the state of the Jammu&


Kashmir can be made by the government of India without the consent of the
state government.

(c) Residuary power in respect of Jammu & Kashmir rest with the state
government and not the union government.

(d) All of the above are incorrect.


5. Which Indian state has its own constitution?
(a) Sikkim

(b) Arunachal Pradesh

(c) Meghalaya

(d) None of these

6. Under which ministry, Directorate General of Foreign Trade


(DGFT) organisation comes:
(a) Ministry of Finance

(b) Ministry of Commerce and Industry

(c) Ministry of Foreign Trade

(d) Home Ministry

7. When the Directorate General of Anti-Dumping & Allied Duties


was was constituted?
(a) 1998

(b) 1988

(c) 1963

(d) 1985

8. Which one of the following is a subordinate office?


(a) Offices of Development Commissioner of Special Economic Zones (SEZs)

(b) Directorate General of Anti-Dumping & Allied Duties (DGAD)

(c) Directorate General of Supplies and Disposal (DGS&D)

(d) Directorate General of Foreign Trade (DGFT)

9. Where is the head quarter of the Directorate General of Foreign


Trade (DGFT)?
(a) Mumbai

(b) Bangaluru
(c) New Delhi

(d) kolkata

10. Where is the head office of pay and accounts office (supply)?
(a) Kolkata

(b) New Delhi

(c) Dehradun

(d) Mumbai

Question Answer
1 d
2 a
3 b
4 d
5 d
6 b
7 a
8 a
9 c
10 b

Question and Answer: Supreme Court of


India
1. From which source India got concept of Single order of court?
(A) Government of India Act, 1935

(B) Government of India Act, 1919

(C) Pitts India Act, 1773

(D) None of the following

Answer A
Explanation: Government of India Act, 1935
2. Which of the following statement is not true about India's
Supreme Court?
(A) Article 124 to 147 and Part V of the Indian Constitution informs about
the composition and powers of the Supreme Court?

(B) The Supreme Court was inaugurated on January 28, 1950

(C) At present there are 35 judges in the Supreme Court

(D) Judges of Supreme Court are appointed by the President of India

Answer C
Explanation: At present there are 31 judges in the Supreme Court since
2009.
3. Which qualification is wrong for being a judge in the Supreme
Court?
(A) It is compulsory to be a citizen of India.

(B) He should be a respected jurist in the eyes of Parliament

(C) Must be a judge in the High Court for at least 5 years

(D) He should be a lawyer in the High Court for at least 10 years

Answer B
Explanation: In the eyes of the President, he should be a respected jurist.
4. Which statement regarding the tenure of judges of Supreme Court
is not correct?
(A) Judge of the Supreme Court can remain in office till the age of 65 years.

(B) Judge of the Supreme Court, gives his resignation letter to the Chief
Justice

(C) On the recommendation of Parliament he can be removed by the


President.

(D) Supreme Court judge can be removed only in the condition of


misconduct.

Answer B
Explanation: A judge of the Supreme Court gives his resignation letter to
the President.
5. Who can remove the Judge of the Supreme Court?
(A) Chief Justice of the Supreme Court
(B) Only President

(C) Only Parliament

(D) Both Parliament and President

Answer D
Explanation: The proposal for the removal of the judge should be passed
with a special majority in both the Houses of Parliament, while the decision
to remove from the post is taken by the President.
6. What is the current salary of Chief Justice of the Supreme Court?
(A) 1.10 lakhs Rupees

(B) 1 lakh Rupees

(C) 90,000 Rupees

(D) 1.25 lakhs Rupees

Answer B
Explanation: From the year 2009, the Chief Justice of the Supreme Court
gets salary of Rs. 1 lakh.
7. The President can declare a judge an executive chief justice of the
Supreme Court of India when ...
(A) The post of Chief Justice is vacant

(B) Chief Justice is temporarily absent

(C) Chief Justice is unable to discharge his obligations

(D) All of the above

Answer D
Explanation: All the above
8. Who among the following has the right to establish the bench of
Supreme Court elsewhere in the country?
(A) The Chief Justice of the Supreme Court

(B) The President of India

(C) The Parliament


(D) Chief Justice of the Supreme Court has the right to allow to establish the
bench of the Supreme Court elsewhere in the country with prior approval of
the President.

Answer D
Explanation: The Chief Justice of the Supreme Court has the right to seek
the permission of the President before setting up the Supreme Court bench
elsewhere in the country.
9. Which of the following Article talks about the establishment of the
Supreme Court?
(A) Article 176

(B) Article 153

(C) Article 124

(D) Article 324

Answer C
Explanation: Article 124
10. Which of the following is not matched correctly?
(A) Article 145: salary of judges

(B) Article 143: Power of President to consult with Supreme Court

(C) Article 141: orders of Supreme Court is applicable to all courts of India

(D) Article 139: Power of Supreme Court to issue writ petition

Answer A
Explanation: Article 145 is related to rules of Supreme Court not the salary
of judges.
GK Quiz on Modern Indian History: Revolt
of 1857
The Revolt of 1857 was an important event in the Indian history. It is also
known as Indian Mutiny, Sepoy Mutiny or First War of Independence. Indian
troops or Sepoys who were in the service of the British East India Company
started the revolt from Meerut and later it spread to several other places
Do you know in India it is also known as the First war of Independence. To
know about the sepoy mutiny it is necessary to understand the root cause of
the movement. Let us study more about the Revolt of 1857 in the form of
questions and answers.
1. What was the role of Tatia Tope in 1857 mutiny?
A He was commander-in-chief of the army of Nana Saheb

B. He organized Bhils of Panchamahal region against the British


C. Both A and B

D. Neither A nor B

Ans. C
2. Who was the governor-general during the Revolt of 1857?
A. Lord Canning

B. Lord Irwin

C. Lord Lytton

D. Lord Willington

Ans A
3. Who was the prominent leader in Lucknow during the Revolt of
1857?
A. Begum Hazrat Mahal

B. Rani Laxmi Bai

C. Kuar Singh

D. Bahadur Shah Zafar

Ans. A
4. Sir Huge Rose described whom as ‘the best and bravest military
leader of the rebel’?
A. Begum Hazrat Mahal

B. Rani Laxmi Bai

C. Kuar Singh

D. Bahadur Shah Zafar

Ans. B
5. Who is the author of the book”The First Indian War of
Independence- 1857-59”?
A. Karl Marx
B. Syed Ahmad Khan

C. R. C. Mazumdar

D. S. N. Sen

Ans. A
6. Consider the following statements related to the cause of 1857
revolt and select the right one.
A. It was a great disparity in salaries between the Indian and European
soldiers.
B. The Indian sepoys were treated with contempt by their European
officers.C
C, The sepoys were sent to distant parts of the empire, but were not paid
any extra allowance.
D. All the above

Ans. D
7. Which of the following is one of the social reasons for 1857
revolt?
A. The English could not establish any social relationship with the Indians.

B. The racial arrogance of the British created a difference between the rulers
and the ruled.

C. Both A & B

D. The company’s trade policy destroyed Indian handicrafts.

Ans. C
8. Which of the following leader associated with Barout in Uttar
Pradesh during 1857 revolts?
A. Shah Mal

B. Maulavi Ahamadullah Shah

C. Tatya Tope

D. Veer Kuwar Singh

Ans. A
9. Who among the following British Officials suppressed the Revolt
of Jhansi?
A. Colin Campbell,
B. Henry Havelock

C. Henry Lawrence

D. Hugh Rose

Ans. C
10. Consider the following statement (s) related to the
administrative causes of 1857 revolt and select the correct one.
A. Deprivation of the traditional ruling classes of their luxury due to the
establishment of the company's suzerainty over the Indian states;
B. Introduction of new and revenue system which snatched the land from
cultivator and gave it to the moneylender or traitor.
C. Lord Canning's announcement to that Mughals would lose the title of King
and be mere Princess.
D. None of the above

Ans. C
The Revolt of 1857: Causes, Nature,
Importance and Outcomes
The Revolt of 1857 (Also regarded as India's First War of Independece) was
an extremely important event in Indian history. It was initiated by the sepoy
of the company which arouses the accumulated grievances of the people
against the Company’s administration and of their dislike for the foreign
regime.

The Revolt of 1857 was a prolonged period of armed uprising as well as


rebellions in Northern and Central India against British occupation of that
part of the subcontinent. Small precursors of brewing discontent involving
incidences of arson in cantonment areas began to manifest themselves in
January. Later, a large-scale rebellion broke out in May and turned into what
may be called a full-fledged war in the affected region. This war brought
about the end of the British East India Company’s rule in India, and led to
the direct rule by the British Government (British Raj) of much of the Indian
Subcontinent for the next 90 years.

Causes of 1857 Revolt


The issue of greased cartridges and military grievances has been over-
emphasized, as the factor for the Revolt of 1857. However, the recent
research has proved that the cartridge was neither the only cause nor even
the most important. In fact, the multiple causes i.e., social-religious-
political- economic worked together to produce the rebellion.
1. Social and Religious Causes: The British had abandoned its policy of
non-interference in the socio-religious life of the Indians. Abolition of Sati
(1829), Hindu Widow Remarriage Act (1856). Christian missionaries were
allowed to enter India and carry on with their mission of proselytizing. The
Religious Disabilities Act of 1850 modified the traditional Hindu law.
According to it, the change in religion would not debar a son from inheriting
the property of his heathen father.
2. Economic Causes: British rule led to breakdown of the village self-
sufficiency, commercialisation of agriculture which burdened the peasantry,
adoption of free trade imperialism from 1800, de-industrialization and drain
of wealth all of which led to overall decline of economy.
3. Military Grievances: The extension of British dominion in India had
adversely affected the service condition of the Sepoys. They were required
to serve in area away from their homes without the payment of extra
bhatta. An important cause of Military discontent was the General Service
Enlistment Act, 1856, which made it compulsory for the sepoys to cross the
seas, whenever required. The Post Office Act of 1854 withdrew the free
postage facility for them.

4. Political Causes: The last major extension of the British Indian territory
took place during the time of Dalhousie. Dalhousie announced in 1849, that
the successor of Bahadur Shah II would have to leave the Red Fort. The
annexation of Baghat and Udaipur were however, cancelled and they were
restored to their ruling houses. When Dalhousie wanted to apply the
Doctrine of Lapse to Karauli (Rajputana), he was overruled by the court of
Directors.
Different leaders Associated with the Revolt of
1857
Barrackpore Mangal Pandey
Delhi Bahadur Shah II, General Bakht Khan
Delhi Hakim Ahsanullah (Chief advisor to Bahadur
Shah II)
Lucknow Begum Hazrat Mahal, Birjis Qadir, Ahmadullah
(advisor of the ex-Nawab of Awadh)
Kanpur Nana Sahib, Rao Sahib (nephew of Nana), Tantia
Tope, Azimullah Khan (advisor of Nana Sahib)
Jhansi Rani Laxmibai
Bihar (Jagdishpur) Kunwar Singh, Amar Singh
Allahabad and Maulvi Liyakat Ali
Banaras
Faizabad Maulvi Ahmadullah (He declared the Revolt as
Jihad against English)
Farrukhabad Tufzal Hasan Khan
Bijnaur Mohammad Khan
Muradabad Abdul Ali Khan
Bareilly Khan Bahadur Khan
Mandsor Firoz Shah
Gwalior/Kanpur Tantia Tope
Assam Kandapareshwar Singh, Manirama Datta
Orissa Surendra Shahi, Ujjwal Shahi
Kullu Raja Pratap Singh
Rajasthan Jaidayal Singh and Hardayal Singh
Gorakhpur Gajadhar Singh
Mathura Sevi Singh, Kadam Singh

British Officials Associated with Revolt


General John Captured Delhi on 20th September, 1857
Nicholson (Nicholson died soon due to a mortal wound
received during the fighting).
Major Hudson Killed Bahadur Shah's sons and grandsons in Delhi.
Sir Hugh Wheeler Defence against Nana Sahib's forces till 26th June,
1857. British forces surrendered on 27th on the
promised of safe conduct to Allahabad.
General Neil Recaptured Banaras and Allahabad in June 1857.
At Kanpur, he killed Indians as revenge against the
killing of English by Nana Sahib's forces. Died at
Lucknow while fighting against the rebels.
Sir Colin Campbell Final recovery of Kanpur on 6th December, 1857.
Final reoccupation of Lucknow on 21 st March,
1858. Recapture of Bareilly on 5th May, 1858.
Henry Lawrence Chief Commissioner of Awadh. Who died during
the seizure of British residency by rebels at
Lucknow on 2nd July, 1857!
Major General Defeated the rebels (Nana Sahib's force) on 17th
Havelock July, 1857. Died at Lucknow in December 1857.
William Taylor Suppressed the revolt at Arrah in August 1857.
and Eye
Hugh Rose Suppressed the revolt at Jhansi and recaptured
Gwalior on 20th June, 1858. The whole of Central
India and Bundelkhand was brought under British
control by him.
Colonel Oncell Captured Banaras.

Causes of Failure
1. Some of the local rulers like Scidia of Gwalior, the Holkar of Indore, the
Nizam of Hyderabad, the Raja of Jodhpur, the Nawab of Bhopal, the rulers of
Patiala, Sindh and Kashmir and the Rana of Nepal provided active support to
the British.

2. The military equipment of the rebels was inferior. Comparative lack of


efficient leadership.

3. The modern intelligent Indians also didn't support the cause.

Impact of the Revolt


1. The revolt was mainly feudal in character carrying with it some nationalist
elements.

2. The control of Indian administration was passed on to the British Crown


by the Government of India Act, 1858.

3. The army was carefully reorganised to prevent the recurrence of such an


event.

The Revolt of 1857 was an extremely important event in Indian history. It


was mere a product of Sepoy but was accumulated grievances of the people
against the Company’s administration and of their dislike for the foreign
regime.

Asian Development Bank


he Asian Development Bank (ADB) was established as a financial institution
that would foster economic growth and cooperation in the Asia-Pacific
region. To increase the awareness of the aspirants of different exams, jagran
josh made this set of quiz of 9 questions. We hope that this set will be useful
for the aspirants of the different competitive exams.

1. On which day of the year 1966, Asian Development Bank was


established?
(a) 19 December

(b) 1 January

(c) 1 April

(d) 24 September

Ans. a
Explanation: ADB was established on 19 December, 1966 on the
recommendations of the economic commission for Asia and Pacific Region
(ECAFE).
2. Which of the following statement is NOT correct about the Asian
Development Bank (ADB)?
(a) ADB started working on January 1, 1967.

(b) The chairmanship of the ADB is always given to the Japanese

(c) Currently it has 53 members

(d) There are 48 members of the ADB belongs to the Asia and Pacific Region

Ans. c
Explanation: At the time of its establishment the Asian Development Bank
had 31 members. But as of now ADB has grown to encompass 68 members;
of which 49 are from within Asia and the Pacific and 19 outside.
3. Which of the following is the main aim of the ADB?
(a) To create a trade block in the Asian continent to save the environment of
the region

(b) To accelerate the social and economic development of the Asia and
Pacific Region

(c) To establish a trade block in the Asia to deal with the competition coming
from the European countries.

(d) None of the above

Ans. b
Explanation: The aim of the ADB is to accelerate the social and economic
development of the Asia and Pacific Region. ADB constituted ADB fund in
1974 which provides fund to Asian countries on concessional interest rates.
4. Which of the following is the newest member of the ADB?
(a) Armenia

(b) Turkmenistan

(c)Luxembourg

(d) Niue

Ans. d
Explanation: Niue is the newest member of the ADB which joined it in
2019. From 31 members at the time of its establishment in 1966, ADB has
grown to 68 members out of which 49 are from within Asia and the Pacific
and 19 outside.
5. Where is the headquarters of the ADB?
(a) Manila

(b) Kathmandu

(c) Jakarta

(d) Dhaka

Ans. a
Explanation: ADB is headquartered in the "Ortigas Center" located in the
city of Mandaluyong, Metro Manila, Philippines. The ADB offers "hard" loans
on commercial terms primarily to middle income countries in Asia and "soft"
loans with lower interest rates to poorer countries in the region.
6. Who is the current President of the ADB?
(a) Takehiko Nakao

(b) Haruhiko Kuroda

(c) Tadao Chino

(d) Shinzō Abe

Ans. a
Explanation: Takehiko Nakao is the current President of the ADB. From
1966 till date there are 9 presidents of the ADB had been chosen. President
of ADB is the chairperson of the Board of Directors and manages ADB. The
president is chosen for the term of five years, and may be re-elected.
Traditionally, the President of ADB has always been Japanese.
7. Which of the following country has the highest subscribed capital
of the ADB?
(a) India

(b) USA

(c) Japan

(d) China

Ans. c
Explanation: Japan is the largest contributor in the subscribed capital of the
country. currently it has 15.677% subscribed capital of the ADB followed by
the USA 15.567% and China 6.473%.India is on the 4th place having a
capital share of 6.359%.
8. Who is the India’s governor on the board of the governors of the
Asian Development Bank?
(a) Prime Minister of India

(b) Finance Minister of India

(c) Governor of Reserve Bank of India

(d) Foreign Minister

Ans. b
Explanation: Finance Minister of India is the India’s governor on the board
of the governors of the Asian Development Bank and in his absence the
finance secretary is the alternate governor.
9. Which of the following country has highest voting power in the
ADB?
(a) USA

(b) India

(c) China

(d) Japan

Ans. d
Explanation: Japan has highest voting power in the ADB. Japan has
12.840% voting power of total votes followed by the USA with 12.752% and
China has 5.477%. India (5.386) is on 4th position and Australia is on
5th position.
Questions and Answers on the Socio-
Religious Reforms Movement in India
The 19th century India witnesses the birth of new vision- a modern vision
among some enlightened sections of the Indian society. This enlightened
vision was to shape the course of events for decades to come and even
beyond. In this article we are giving,
As the Reform movements at that time related with some religious beliefs
therefore most of the movements of social reform were of a religious
character. Socio-religious reform movements arose in most of the
communities of Indian people. They not only attacked the bigotry but also
superstition and the hold of the priestly class.

1. Which of the following is not correctly matched?


A. Brahama Samaj - Reformist Movements

B. Prarthana Samaj - Reformist Movements

C. Aligarh Movement - Reformist Movements

D. Arya Samaj - Reformist Movements

Ans: D
Explanation: The correct match is given below-
Brahma samaj: Reformist movements
Prarthana samaj: Reformist movements
Aligarh movement: Reformist movements
Arya samaj: Revivalist movements
Theosophical movement: Revivalist movements
Deoband movement: Revivalist movements
Hence, D is not correctly matched.

GK Questions and Answers on the Growth of Modern Education in


India
2. Who among the following established the Tattvabodhini Sabha in
1839 at Calcutta (Now Kolkata)?
A. Keshab Chandra Sen

B. Debendra Nath Tagore

C. Rammohan Roy
D. Sivanatha Sasri

Ans: B
Explanation: Debendranath Tagore established the Tattvabodhini Sabha
(1839) at Calcutta to propagate Rammohan Roy’s ideas. Hence, B is the
correct option.
3. Which of the following is associated with the Young Bengal
Movement?
A. Derozians

B. Mitra Bandhu

C. Native marriage Act

D. Bethune School

Ans: A
Explanation: The movement started by Derozio was called the Young
Bengal Movement and his followers were known as the Derozians. They
condemned religious rites and the rituals, and pleaded for eradication of
social evils, female education and improvement in the condition of women.
Hence, A is the correct option.
4. Who among the following was popularly known as ‘Lokahitawadi'?
A. Jyotiba Phule

B. Pt. Iswar Chandra Vidhyasagar

C. Gopal Hari Deshmukh

D. Mahadev Govind Ranade

Ans: C
Explanation: Gopal Hari Deshmukh was popularly known as ‘Lokahitawadi’.
He made powerful rationalist attacks on Hindu orthodoxy and preached
religious and social equality. Hence, C is the correct option.
5. Who among the following establishes the Mukti Mission in Pune?
A. Pandita Ramabai

B. Swami Dayanand Saraswati

C. Savitribai Phule

D. Jyotiba Phule
Ans: A
Explanation: In 1889, Pandita Ramabai established the Mukti Mission, in
Pune, a refuge for young widows who had been deserted and abused by
their families. She also started Sharda Sadan which provided housing,
education, vocational training and medical services to widows, orphans and
the visually challenged. Hence, A is the correct option.
GK Questions and Answers on the History of Press in India
6. Which of the following statement(s) is/are correct with the
reference of Singh Sabha Movement?
A. It aimed for modern Western education for Sikhs

B. It was formed to counter proselytising activities in Sikhs by Hindu


revivalists

C. The Akali Movements was an off-shoot of this movement

D. All of the above

Ans: D
Explanation: The Singh Sabha Movement was a Sikh movement that began
in Punjab in the 1870s in reaction to the proselytising activities of Christians,
Brahmo Samajis, Arya Samaj, Muslim Aligarh movement and Ahmadiyah.
This movement was organised in an era when Sikh Empire had been
dissolved and annexed by the colonial British, Khalsa had lost its prestige,
and mainstream Sikhs were rapidly converting to other religions. The
movement's aims were, according to Barrier and Singh, to "propagate the
true Sikh religion and restore Sikhism to its pristine glory; to write and
distribute historical and religious books of Sikhs; to propagate Gurmukhi
Punjabi through magazines and media". It was helped by the missionaries’
activities of Mohammadens and Christians. It grew out of nowhere to
become a founding father of current SGPC and Akali party. Hence, D is the
correct option.
7. Which of the following is correctly paired?
A. Bengal Regulation banning Sati -1830

B. Sarda Act- 1939

C. Special Marriage Act -1955

D. Hindu Succession Act- 1956

Ans: D
Explanation: The Hindu Succession Act, 1956 is an Act of the Parliament of
India enacted to amend and codify the law relating to intestate or unwilled
succession, among Hindus, Buddhists, Jains, and Sikhs. The Act lays down a
uniform and comprehensive system of inheritance and succession into one
Act. Hence, D is the correct option.
GK Questions and Answers on the revolutionary movements during
British India
8. The Parsi Reform Movement Rahnumai Mazdayasnan Sabha
(Religious Reform Association) was founded in which year?
A. 1829

B. 1851

C. 1879

D. 1861

Ans: B
Explanation: The Western-educated progressive Parsis like Dadabhai
Naoroji, J.B. Wacha, S.S. Bangali and Naoroji Furdonji founded the
Rahanumai Mazdayasanan Sabha (Religious Reform Association) in 1851.
The prime objective of the association was- “the regeneration of the social
condition of the Parsis and the restoration of the Zoroastrian religion to its
pristine purity”. Rast Goftar (Voice of Truth) was its weekly organ. Hence, B
is the correct option.
9. Who among the following founded the Nirankari Movement?
A. Baba Dyal Das

B. Baba Ram Singh

Ç. Mahmud Hasan

D. Jagat Mithra

Ans: A
Explanation: Baba Dayal Das founded the Nirankari Movement. He insisted
the worship of God as Nirankar (formless). Hence, A is the correct option.
10. Who among the following founded the Namdhari Movement?
A. Baba Dyal Singh

B. Baba Ram Singh

Ç. Mahmud Hasan

D. Jagat Mithra
Ans: B
Explanation: The Namdhari Movement was founded by Baba Ram Singh.
His followers wore white clothes and gave up meat eating. Hence, B is the
correct option.
Questions and Answers on NASA Missions
NASA (National Aeronautics and Space Administration) is a United
States government agency created in 1958 with an aim to explore space and
for flight research. It is also responsible for Science and Technology related
to air and space. There are various missions of NASA both manned and
unmanned. This article deals with the NASA’s missions which will help in the
preparation of various entrance examinations and also gives knowledge
about NASA and its missions.
1. Which mission of NASA brought first human to moon?
A. Apollo program
B. Gemini Program
C. Mercury Program
D. None of the above
Ans. A
2. Name the first mission of NASA in which a spacecraft was reused?
A. Apollo-Soyuz
B. Space Shuttle
C. Skylab
D. None of the above
Ans. B
India's moon mission Chandrayaan-2 launched 22 July: All you need
to know
3. How many manned space missions are launched by NASA so far?
A. 160
B. 162
C. 164
D. 166
Ans D
4. Name the first Spacecraft to visit the Solar System?
A. Pioneer 10 and Pioneer 11
B. Ranger 1 and Ranger 2
C. Surveyor 6 and Surveyor 7
D. Viking 1 and Viking 2
Ans A
5. What was the first manned mission of NASA?
A. PQS-5
B. Apollo-Soyuz
C. STS-1
D. Bell X-1
Ans. C
6. Name the NASA’s first exploration mission beyond earth?
A. Pioneer 2
B. Pioneer 3
C. Pioneer 4
D. Pioneer P-1
Ans. C
Space Missions to the Sun: Brief Analysis
7. When NASA had launched its first mission to Sun?
A. 2016
B. 2017
C. 2018
D. None of the above
Ans. C
8. What is the goal of the Surveyor Program of NASA?
A. To demonstrate the proper landing on the Mars.
B. To demonstrate the soft landings on the Moon.
C. To check other missions of Apollo.
D. To determine the radiations in Space.
Ans. B
9. Name the first satellite which was launched specifically for the
purpose of X-ray astronomy?
A. Uhuru
B. Hinode
C. Clementine
D. WMAP
Ans. A
10. Name NASA’s Mars Exploration Rover Mission?
A. Opportunity Rover
B. Spirit Rover
C. Sojourner Rover
D. Curiosity Rover
Ans. B

India's second moon mission


Chandrayaan-2 : Important Questions and
Answers
Chandrayaan-2 is the second mission to moon after Chandrayaan-1 which
was launched in 2008. Chandrayaan-2 will carry 13 scientific satellites and
weighs around 3.8 tonnes which will be equivalent of eight elephants. No
doubt Chandrayaan-2 is the advanced version of Chandrayaan-1. Let us
solve questions related to Chandrayaan-2.
1. Which of the following statement is true for Chandrayaan-2?
A. Chandrayaan-2 is an indigenous mission.
B. Chandrayaan-2 moon mission launch date is delayed.
C. During Chandrayaan-2 launching a technical snag was observed in launch
vehicle system at T-56 minutes.
D. All the above
Ans. D
Explanation: Chandrayaan-2 is the ISRO's second moon mission and is
totally an indigenous. The launched date of Chandrayaan-2 was postponed
due to some technical problem. In the launch vehicle system a technical
error was observed at T-56 minutes. It was originally scheduled to launch on
15 July, 2019 at 2.51 am IST. On 22 July, 2019, Chandrayaan-2 successfully
launched.
2. According to ISRO, Chandrayaan-2 moon mission will explore which
area of the moon?
A. Moon’s North Pole
B. Moon's South Pole
C. Moon's North-West part.
D. Unexplored part of the Moon
Ans. B
Explanation: According to ISRO, Chandrayaan-2 moon mission will explore
South Pole of the moon.
3. Chandrayaan-2 spacecraft has how many modules?
A. 2
B. 3
C. 4
D. 5
Ans. B
Explanation: Chandrayaan-3 spacecraft has three modules.
4. What are the objectives of launching Chandrayaan-2 moon mission?
A. To map the surface of the moon.
B. Signature of water-ice on the lunar surface.
C. To collect data on minerals and formation of rocks.
D. All the above
Ans. D
Explanation: Scientific objectives are: to map the Moon's surface, its
mineral and element content, moonquakes and signatures of water-ice on
the lunar surface.
5. Chandrayaan-2 Lunar mission comprises:
A. An Orbiter
B. Lander
C. Rover
D. All the above
Ans. D
Explanation: Indian Space Research Organisation (ISRO) second moon
mission is Chandrayaan-2 which comprises an orbiter and lander and a
rover.
6. Which rocket will carry Chandrayaan-2 into the space?
A. PSLV
B. GSLV
C. GSLV Mk-III
D. RLV-TD
Ans. C
Explanation: The GSLV Mk-III will carry Chandrayaan-2 to its designated
orbit.
7. Spacecraft Lander is also known as....
A. Vikram
B. Vijay
C. Vikramaditya
D. None of the above
Ans. A
Explanation: Lander also known as Vikram. ISRO has named Lander after
the founder of ISRO and the father of Indian Space Program Vikram
Sarabhai.
8. Which of the following statements are correct about Rover?
A. Rover is also known as Pragyan.
B. On the Lunar surface it will test the mineral and chemical composition.
C. Both A and B
D. Only A
Ans. C
Explanation: Rover also known as Pragyan will have two instruments on
board. On the surface of the Moon the instrument will test the mineral and
chemical compositions and also about soil and rocks formation. Data on and
around the South Pole will be collected and sent.
9. Which of the following statement is or are correct about Orbiter?
A. Orbiter of Chandrayaan-2 will be installed at 100 km above the moon.
B. It consists of eight instruments.
C. it will bring the command sent from ISRO to the lander and the rover.
D. All the above
Ans. D
Explanation: Orbiter of Chandrayaan-2 will be installed at 100 km above
the moon and consists of eight instruments. Specifications of these
instruments are not provided that will be loaded on to the rocket. It will be
circling the information from Lander and Rover to the ISRO Centre. It will
also bring the command sent from ISRO to the lander and the rover.
10. What is the orbiter mass of Chandrayaan-2?
A. 2090 Kg
B. 2209 Kg
C. 2379 Kg
D. 2390 Kg
Ans. C
Explanation: The Chandrayaan 2 orbiter is a box-shaped craft with an
orbital mass of 2379 kg and solar arrays capable of generating 1000 W
power.
11. The Lander was designed to execute the soft-landing on the
surface of the moon and to function on one Lunar day which is equal
to 14 earth days. But unfortunately few minutes before landing, ISRO
lost contact with Vikram Lander. Scientists of ISRO are trying to
restore the contact with the Vikram Lander.

12. The Chairman of ISRO K Sivan recently told that the space
agency has found location of the Vikram Lander through a thermal
image which was clicked by the Orbiter of the Chandrayaan-2, which is
already in the intended orbit around the Moon. But there is no
communication yet. ISRO is trying to have contact and hoping for the
best.

13. Let us see what happened in the past.


14. On 2 September 2019, the Vikram lander has been placed in an
independent circular path around the moon identical to that of the
orbiter that was passing over the lunar poles at a distance of about
100 km from the surface. Chandrayaan-2 with the combined orbiter,
lander and Rover finally completed its fifth orbit-lowering manoeuvre
on 1 September, 2019 that brings it to almost circular orbit of 119 X
127 km around the moon.

15. Chandrayaan-2 successfully entered the lunar orbit on 20


August, 2019. According to ISRO, Lander (Vikram) along with a Rover
(Pragyan) has separated from the orbiter on 2 September,2019 and is
now on its way to the Moon.

16. Chandrayaan-2 on 14 August, 2019 successfully entered Lunar


Transfer Trajectory. Only three countries US, Russia and China
reached on the surface of the Moon and China being the only country
to have made it in the first attempt.

17. India's second Moon mission Chandrayaan-2 was successfully


launched at 2:43 pm on 22 July, 2019 from Sriharikota space station.
A new history is created in India. It is a proud moment for all of us.
People will be able to see the darker side of the Moon.

18. About Chandrayaan-2 India's second Moon mission


19. Chandrayaan-2 moon mission is totally an indigenous mission.
Earlier, Chandryaan-2 was decided to launch on 15 July, 2019 but due
to some technical problem it was delayed. Finally, it was launched on
22 July, 2019 via GSLV-Mk-III rocket from Satish Dhawan Space
Centre, Sriharikota. No doubt, the Chandrayaan-2 mission comes
nearly 11 years after India's first expedition to the moon in October
2008. Therefore, space agency said it is better to be delayed rather
than any complications.

20. It is one of the biggest project till date. The journey of


Chandrayaan-2 is around 3.84 lakh km to Moon. It will be injected into
an earth parking 170 x 40400 km orbit. The weight of Chandrayaan-2
spacecraft is approximately 3850 kilograms. It will collect data on
water, minerals and formations of rock.

21. Chandrayaan-2 spacecraft have three modules a Lander


(Vikram), an Orbiter and Rover (Pragyan).

22. The primary objective of ISRO to launch Chandrayaan-2 is to


demonstrate the ability to soft-land on the lunar surface and operate a
robotic rover on the surface. Scientific goals are to study lunar
topography, mineralogy, elemental abundance, the lunar exosphere
and signatures of hydroxyl and water ice.
23. Chandrayaan-2 landing near the Pole
The Lander and Rover of Chandrayaan-2 are targeted for a location of
about 600 kilometres or 375 miles approx from the South Pole. Let us
tell you that it would be the first time any mission touched down so far
from the equator. The Orbiter and Lander modules will be mechanically
interfaced and stacked together like an integrated module and
accommodated inside the GSLV MK-III launch vehicle where as the
Rover is housed inside the Lander.
After launching, the integrated module will reach the Moon orbit by
using the Orbiter propulsion module. Then, Lander will automatically
separate from the Orbiter and land at the site close to lunar South
Pole.
24. Breakdown of entire payload of Chandrayaan-2
25. 1. Rover also known as Pragyan will have two instruments on
board. On the surface of the Moon the instrument will test the mineral
and chemical compositions and also about soil and rocks formation.
Data on and around the South Pole will be collected and sent. That is it
will send information from the moon to Vikram Lander. Lander will
send data to Orbiter. Then the Orbiter will send it to the ISRO centre.
This whole process will take about 15 minutes. So, it can be said that
information sent from Pragyan Robot will take about 15 minutes to
reach the ISRO centre in India.

26.
2. Lander also known as Vikram. ISRO has named Lander after the
founder of ISRO and the father of Indian Space Program Vikram
Sarabhai. The five-legged lander will have three instruments onboard.
They are the Radio Anatomy of Moon-Bound Hypersensitive
Ionosphere and an Atmosphere Probe (Rambha) that will measure the
density of the lunar sub surface and changes around it. Further, the
Chandra Surface Thermophysical Experiment (ChaSTE) will be used to
measure the thermal temperature around the South Pole of the moon.
And thirdly, is the instrument for Lunar Seismic Activity (ILSA) which
will measure the seismicity or quake or tremor-potential of the region.

27.
It will use scientifically for 15 days. Its initial design was made by
ISRO's Space Application Centre Ahmedabad. Later, it was developed
by the URSC of Bengaluru. ISRO has made an indigenous Lander on
Russia's refusal.
28. 3. Orbiter of Chandrayaan-2 will be installed at 100 km above
the moon and consists of eight instruments. Specifications of these
instruments are not provided that will be loaded on to the rocket. But
there will be an Imaging Infra-red Spectometer (IIRS) which will try to
identify minerals and indicators of hydroxyl and water molecules. It
will operate on solar power. It will be circling the information from
Lander and Rover to the ISRO Centre. It will also bring the command
sent from ISRO to the lander and the rover. It was generated by
Hindustan Aeronautics Limited and handed it to ISRO in 2015.

29.

30. The profile of the Mission


31. As discussed above Chandrayaan-2 begin its journey to Moon on
22 July, 2019. The Lander-Orbiter pair will go into an initial elliptical
(180 X 24000 km altitude) Earth orbit, followed by a trans-lunar
injection. Both will go into an initial elliptical lunar orbit. After orbit
insertion, the Lander and the Orbiter separate. The Orbiter evolves
into a 100 km circular polar orbit and the Lander breaks from orbit and
lands on the surface in the high latitude areas near the South Pole.
The Orbiter portion of the mission is planned to last 1 year. The rover
will be deployed using a ramp shortly after the landing and is planned
for 14-15 days that is one period of lunar daylight.

32. Note: India's first lunar probe was Chandrayaan-1 mission


which was launched in October 2008. The mission included a lunar
orbiter and an impactor but did not have a rover like Chandrayaan-2.
33. Chandrayaan-2: Facts in brief
34. Operator - Indian Space research Organisation
Type of Mission - Lunar orbiter, Lander and Rover
Duration of Mission - Orbiter: 1 year
Lander: >15 Days
Rover: >15 Days
Launch Mass - aprox. 3850 Kg
Mass of Payload - Orbiter: approx 2,379 Kg
Lander: approx. 1,471 Kg
Rover: approx. 27 Kg
Date of Launch - Not yet announced by ISRO
Rocket - GSLV Mk III
Launch Site - Satish Dhawan Space Centre
35. Launch Date: 22 July, 2019

S-ar putea să vă placă și